Download as pdf or txt
Download as pdf or txt
You are on page 1of 106

CHAPTER FOUR

Determinants

EVALUATION OF DETERMINANTS MINORS AND COFACTORS


A determinant of order two is written as Minor
a11 a12 Let A = (aij)n × n be a square matrix of order n. Then the minor
(ai j ΠC " i, j)
a21 a22 Mij of the element aij of the matrix A is the determinant of
the square sub-matrix of order (n – 1) obtained by deleting
and is equal to a11 a22 – a12 a21.
ith row and jth column of matrix A.
A determinant of order three is written as
a11 a12 a13 Illustration 1
a21 a22 a23 (ai j ΠC " i, j) Minor of element a23 in the determinant
a31 a32 a33 a11 a12 a13
and is equal to a21 a22 a23
a31 a32 a33
a22 a23 a a23 a a22 is
a11 - a12 21 + a13 21
a32 a33 a31 a33 a31 a32 a a
M23 = 11 12
= a11 (a22 a33 – a23 a32) – a12 (a21 a33 – a23 a31) a31 a32

+ a13(a21 a32 – a22 a31) Cofactor


= a11 a22 a33 + a12 a23 a31 + a13 a32 a21 Let A = (aij)n ¥ n be a square matrix of order n. Then the
– a13 a31 a22 – a32 a23 a11 – a12 a21 a33 cofactor of the element aij of the matrix A is denoted by Cij
and is equal to (- 1)i + j Mij where Mij is the minor of the
A determinant of order 3 can also be evaluated by using element aij of the matrix A.
the following diagram, due to Sarrus: Note that
a11 a12 a13
a21 a22 a23 = a11 M11 – a12 M12 + a13 M13
a a a = a11 C11 + a12C12 + a13C13
31 32 33

a11 a12 a13


The product of the three terms on each of the three single If D = a21 a22 a23
arrows are prefixed by a positive sign and the product of the
three terms on each of the three double arrows are prefixed a31 a32 a33
by a negative sign. then D = ai1 Ci1 + ai2 Ci2 + ai3 Ci3 i = 1, 2, 3
= a1j C1j + a2j C2j + a3j C3j j = 1, 2, 3
Remark
Note
This method does not work for higher order determinants.
ai1 Cj1 + ai2 Cj2 + ai3 Cj3 = 0 iπj
and a1i C1j + a2i C2j + a3i C3j = 0 iπj
4.2 Complete Mathematics—JEE Main

Remark Illustration 2
The above results remain true for determinants of every To evaluate
order. 1+ a b c
D= a 1+ b c
PROPERTIES OF DETERMINANTS a b 1+ c
write D = D1 + aD2 where
1. Reflection Property 1 b c
The determinant remains unaltered if its rows are changed D1 = 0 1 + b c
into columns and the columns into rows. 0 b 1+ c
In other words, if A is a square matrix, then |A| = |A¢| where
A¢ is transpose of A. = (1 + b) (1 + c) – bc = 1 + b + c
and
2. All-zero Property 1 b c
If all the elements of a row (column) are zero, then the de- D2 = 1 1 + b c
terminant is zero. 1 b 1+ c
Using C2 Æ C2 – bC1 and C3 Æ C3 – cC1,
3. Proportionality [Repetition] Property
we get
If the elements of a row (column) are proportional [identi- 1 0 0
cal] to the element of the some other row (column), then the D2 = 1 1 0 = 1
determinant is zero. 1 0 1
4. Switching Property \ D =1 + a + b + c

The interchange of any two rows (columns) of the determi- 8. Factor Property
nant changes its sign.
If a determinant D becomes zero when we put x = a, then
5. Scalar Multiple Property (x – a) is a factor of D.

If all the elements of a row (column) of a determinant are 9. Triangle Property


multiplied by a non-zero constant, then the determinant gets
If all the elements of a determinant above or below the main
multiplied by the same constant.
diagonal consists of zeros, then the determinant is equal to
6. Property of Invariance the product of diagonal elements. That is,

a1 b1 c1 a1 + a b1 + b c1 b1 c1 a1 a2 a3 a1 0 0
a2 b2 c2 = a2 + a b2 + b c2 b2 c2 0 b2 b3 = a2 b2 0 = a1 b2 c3
a3 b3 c3 a3 + a b3 + b c3 b3 c3 0 0 c3 a3 b3 c3

That is, a determinant remains unaltered under an opera- 10. Product of Two Determinants
tion of the form Ci Æ Ci + a Cj + b Ck, where j, k π i, or an
a1 b1 c1 a1 b1 g1
operation of the form Ri Æ Ri + a Rj + b Rk, where j, k π i.
a2 b2 c2 a2 b2 g2
7. Sum Property a3 b3 c3 a3 b3 g3
a1 + b1 c1 d1 a1 c1 d1 b1 c1 d1 a1a1 + b1b1 + c1g 1 a1a 2 + b1b2 + c1g 2
a2 + b2 c2 d2 = a2 c2 d2 + b2 c2 d2 = a2a1 + b2 b1 + c2g 1 a2a 2 + b2 b2 + c2g 2
a3 + b3 c3 d3 a3 c3 d3 b3 c3 d3 a3a1 + b3 b1 + c3g 1 a3a 2 + b3 b2 + c3g 2

a1a 3 + b1b3 + c1g 3


Remark
a2a 3 + b2 b3 + c2g 3
It is one of the most under used property. But evaluation of a3a 3 + b3 b3 + c3g 3
some of the determinants beame very easy when we use it.
Determinants 4.3
Here we have multiplied rows by rows. We can also mul- where a21, a22, a23, a31, a32 and a33 are constants, then
tiply rows by columns, or columns by rows, or columns by
columns. ¢ ( x ) a12
a11 ¢ ( x ) a13
¢ ( x)
D ¢(x) = a21 a22 a23 and
11. Conjugate of a Determinant a31 a32 a33
If ai, bi, ci ΠC (i = 1, 2, 3), and
a1 b1 c1 a1 b1 c1
Ú a11 ( x)d x Ú a12 ( x)d x Ú a13 ( x)d x
Z = a2 b2 c2 then Z = a2 b2 c2 Ú D( x)dx = a21 a22 a23
a31 a32 a33
a3 b3 c3 a3 b3 c3
In general, for any positive integer m
12. Differentiation of a Determinant
m m m
a11 ( x ) a12 ( x ) a13 ( x)
If each ai(x) is differentiable function m
D (x) = a21 a22 a23
a1 ( x ) a2 ( x )
and D(x) = a31 a32 a33
a3 ( x ) a4 ( x )
13. Determinant of Cofactor Matrix
a1¢ ( x ) a2 ( x ) a ( x ) a2¢ ( x )
then D¢(x) = + 1 a11 a12 a13 C11 C12 C13
a3¢ ( x ) a4 ( x ) a3 ( x ) a4¢ ( x )
If D = a21 a22 a23 then D1 = C21 C22 C23 = D2
If we write D(x) = [C1, C2], where Ci denotes ith column,
then a31 a32 a33 C31 C32 C33
D¢(x) = [C 1¢ , C2] + [C1, C 2¢ ] where C 1¢ denotes the column where Ci j denotes the co-factor of the element ai j in D.
which contains the derivative of all the functions in the ith
column Ci. Similarly, if SOME TIPS FOR QUICK EVALUATION
OF DETERMINANTS
ÈR ˘ È R¢ ˘ È R ˘
D(x) = Í 1 ˙ then D¢(x) = Í 1 ˙ + Í 1 ˙ 1. If D is a skew symmetric determinant of odd order,
R
Î 2˚ Î R2 ˚ Î R2¢ ˚
then D = 0
a11 ( x ) a12 ( x ) a13 ( x )
Next, if D(x) = a21 ( x ) a22 ( x ) a23 ( x ) Illustration 3
then D¢(x) a31 ( x ) a32 ( x ) a33 ( x )
0 a b
¢ ( x ) a12 ( x ) a13 ( x )
a11 ¢ ( x ) a13 ( x )
a11 ( x ) a12 Let D = -a 0 c
= ¢ ( x ) a22 ( x ) a23 ( x ) + a21 ( x ) a22
a21 ¢ ( x ) a23 ( x ) -b -c 0
¢ ( x ) a32 ( x ) a33 ( x )
a31 ¢ ( x ) a33 ( x )
a31 ( x ) a32 Using the reflection property, write
¢ ( x)
a11 ( x ) a12 ( x ) a13 0 - a -b
¢ ( x)
+ a21 ( x ) a22 ( x ) a23 D = a 0 -c
¢ ( x)
a31 ( x ) a32 ( x ) a33 b c 0
Taking –1 common from R1, R2 and R3 we get
= [C 1¢ , C2, C3] + [C1, C 2¢ , C3] + [C1, C2, C ¢3]
0 a b
Similarly, if D = (–1)3 - a 0 c = –D
È R1 ˘ È R1¢ ˘ È R1 ˘ È R1 ˘ -b -c 0
D(x) = Í R2 ˙ then D¢(x) = Í R2 ˙ + Í R2¢ ˙ + Í R2 ˙ fi 2D = 0 fi D = 0
Í ˙ Í ˙ Í ˙ Í ˙
ÎÍ R3 ˙˚ ÍÎ R3 ˙˚ ÍÎ R3 ˙˚ ÍÎ R3¢ ˙˚ 2. If a1, a2, a3 are in A.P.; b1, b2, b3 are in A.P. and c1, c2,
Corollary (Differentiation and Integration of Determinant) c3 are also in A.P. Then
a1 a2 a3
a11 ( x ) a12 ( x ) a13 ( x ) D = b1 b2 b3 = 0
If D(x) = a21 a22 a23 c1 c2 c3
a31 a32 a33 Use C1 Æ C1 + C3 – 2C2
4.4 Complete Mathematics—JEE Main
3. If a1, a2, a3 are in G.P.; b1, b2, b3 are also in G.P., with a1 b1 c1
the same common ratio, then
D = a2 b2 c2
a1 a2 a3
a3 b3 c3
D = b1 b2 b3 = 0
c1 c2 c3 If D π 0, then the only solution of the above system of
equations is x = 0, y = 0 and z = 0.
[c1, c2, c3 can be any three complex numbers.]
Corollary If at least one of x, y, z is non-zero and x, y and
Some Frequently used Determinants z are connected by the three given equations, then the elimi-
1 1 1 1 1 1 nation of x, y and z leads to the relation
1. a b c = a b c a1 b1 c1
a2 b2 c 2 bc ca ab a2 b2 c2 = 0
= (a – b) (b – c) (c – a) a3 b3 c3
1 1 1
2. a b c = (a – b) (b – c) (c – a) (a + b + c) CRAMER’S RULE
3 3 3
a b c
a1 b1 c1
1 1 1 If D = a2 b2 c2 π 0
3. a 2 b2 c 2 = (a – b) (b – c) (c – a) (bc + ca + ab) a3 b3 c3
3 3
a b c3 then the solution of the system of linear equations
a b c a1 x + b1 y + c1 z = d1
4. b c a = 3abc – a3 – b3 – c3 a2 x + b2 y + c2 z = d2
c a b a3 x + b3 y + c3 z = d3
= (a + b + c) (bc + ca + ab – a2 – b2 – c2) D D D
1 is given by x = 1 , y = 2 , z = 3
= - (a + b + c)[(b - c)2 + (c - a)2 + (a - b)2 ] D D D
2 where
LINEAR EQUATIONS d1 b1 c1 a1 d1 c1
The system of linear homogeneous equations D1 = d2 b2 c2 , D 2 = a2 d2 c2 and
a1 x + b1 y + c1 z = 0 d3 b3 c3 a3 d3 c3
a2 x + b2 y + c2 z = 0 a1 b1 d1
a3 x + b3 y + c3 z = 0 D 3 = a2 b2 d2
has a non-trivial solution (i.e., at least one of the x, y, z is a3 b3 d3
different from zero) if and only if D = 0, where

SOLVED EXAMPLES
Concept-based
Straight Objective Type Questions

Example 1: Let Then D(–3, 2) equals


1 x y (a) 13 (b) – 6
D(x, y) = 1 x + y y (c) 12 (d) – 5
1 x x+y Ans. (b)
Determinants 4.5
Solution: Using R2 Æ R2 – R1, R3 Æ R3 – R1 we get Example 4: Let
1 x y 0 b-a c-a
D(x, y) = 0 y 0 = xy D = a-b 0 c - b , then
0 0 x a-c b-c 0
\ D(–3, 2) = – 6 D equals:
(a) 0 (b) abc
Example 2: Suppose a, b, c are distinct real numbers and (c) a2 + b2 + c2 (d) bc + ca + ab
a a2 b+c Ans. (a)

D= b b 2
c+a =0 Solution: Interchanging the rows and columns, we get
2 0 a-b a-c
c c a+b
D = b-a 0 b-c
Then a + b + c equals
(a) –1 (b) 2 c-a c-b 0
(c) 0 (d) –5 Taking –1 common from each of R1, R2, R3 we get
Ans. (b) 0 b-a c-a
Solution: Using C3 Æ C3 + C1 and taking (a + b + c) D = (–1)3 a - b 0 c-b
common from C3 we get a-c b-c 0
D = (a + b + c) D1 (1) fi D = –D fi 2D = 0 or D = 0
where
Alternative Solution
a a2 1 D is a skew symmetric determinant of odd order,
D1 = b b 2 1 therefore D = 0
c c2 1 Example 5: Let A = (aij)3 × 3, where aij Œ C the set of
Using R1 Æ R1 – R2 and R2 Æ R2 – R3, we get complex numbers. If det (A) = 2 – 3i, then det (A–1) equals:
a - b a 2 - b2 0 1 1
(a) (2 – 3i) (b) (2 + 3i)
13 13
D1 = b - c b2 - c 2 0 (c) 2 – 3i (d) 2 + 3i
2
c c 1 Ans. (b)
1 1
1 a+b 0 Solution: det (A–1) = =
det ( A) 2 - 3i
= (a – b) (b – c) 1 b + c 0
2 + 3i 1
c c2 1 = = (2 + 3i )
22 + 32 13
1 a+b Example 6: In a triangle ABC, if
= (a – b) (b – c)
1 b+c 1 a b
[Expand along C3] D= 1 c a =0
= (a – b) (b – c) (c – a) (2)
1 b c
From (1) and (2)
D = (a + b + c) (a – b) (b – c) (c – a) then sin A + sin B + sin2C is:
2 2

As D = 0 and a, b, c are distinct, 3 3 5


(a) (b)
we get a+b+c=0 2 4
Example 3: Suppose A = (aij)3 × 3, where aij Œ R. If 9
(c) (d) 2
det (adj A) = 25, then |det (A)| equals: 4
(a) 5 (b) 12.5 Ans. (a)
(c) 5 5 (d) 52/3 Solution: Evaluating along C1, we get
Ans. (a) D = (c2 – ab) + (b2 – ac) + (a2 – bc) = 0
fi (a – b)2 + (b – c)2 + (c – a)2 = 0
Solution: Using det (adj A) = (det (A))2, we get fi a = b = c fi A = B = C = p /3
(det (A))2 = 25 fi |det (A)| = 5
3 3
\ sin2A + sin2B + sin2C = .
2
4.6 Complete Mathematics—JEE Main
Example 7: If a - b a 2 - b2 0
2
x +x x +1 x-2 D1 = b - c 2
b -c 2
0
2 x2 + 3x - 1 3x 3 x - 3 = Ax + B, c c 2
1
x2 + 2 x + 3 2x - 1 2x - 1 1 a+b
= (a - b)(b - c)
then A is equal to: 1 b+c
(a) 12 (b) 18 = (a – b) (b – c) (c – a)
(c) 24 (d) 30 As a < b < c, D1 π 0. Therefore,
Ans. (c) D = 0 fi p + abc = 0
Solution: Applying R2 Æ R2 – R1 – R3, we get fi p = –abc
As p is prime and a, b, c are integers such that a < b < c, we
x2 + x x +1 x-2 must have – a = 1, b = 1, c = p.
Ax + B = -4 0 0 fi a = –1, b = 1, c = p.
x2 + 2 x + 3 2 x - 1 2 x - 1 Example 9: Suppose
Expanding along R2, we get x -51 -71
x +1 x - 2 P(x)= 51 x -73
Ax + B = (-1)(- 4)
2x - 1 2x - 1 71 73 x
Product of zeros of P(x) is
3 x-2
=4 (a) 0 (b) 195
0 2x - 1 (c) –195 (d) –264333
= 4(3) (2x – 1) = 24x – 12 Ans. (a)
Thus, A = 24 Solution:
Example 8: Suppose a, b, c are three integers such a < b
< c and p is a prime number. TIP
2 3
a a p+a Suppose P(x) = x3 + ax2 + bx + c, then product of zeros
Let D= b b 2
p+b 3 of P(x) is – c = –P(0).
2
c c p + c3 0 -51 -71
If D = 0, then which one of the following is not true Now, – c = - P(0) = - 51 0 -73
(a) a = –1, b = 1 (b) b = 1, c = p 71 73 0
=0
(c) a = 0, c = p (d) abc + p = 0
[ Skew symmetric determinant of odd order]
Ans. (c)
Example 10: Let
Solution: Write D = pD1 + D2
where x -3 + 4i 3 - 4i
a a2 1 a a2 a3 P(x) = x -7i 5 + 6i
D1 = b b 2 1 and D2 = b b 2 b3 - x 7 - 2i -7 - 2i

c c2 1 c c2 c3 The number of values of x for which P(x) = 0 is


(a) 0 (b) 1
write (c) 2 (d) 3
1 a a2 a a2 1 Ans. (b)
D2 = abc 1 b b2 = abc(-1)2 b b2 1 Solution: Using R2 Æ R2 – R1, R3 Æ R3 + R1,
2 2 we get
1 c c c c 1 x -3 + 4i 3 - 4i
= abc D1 P(x) = 0 3 - 11i 2 + 10i
Thus, D = (p + abc) D1. 0 4 + 2i -4 - 6i
Using R2 Æ R2 – R3, R1 Æ R1 – R2, we get As P(x) is a linear polynomial, P(x) = 0 for exactly one
value of x.
Determinants 4.7
Example 11: Let Thus, D(x) = x2 + (a2 + a4) x2 – x3
1 sin q 1 = (1 + a2 + a4) x2 – x3
D (q) = - sin q 1 sin q , 0 £ q £ 2p 1
As x π 0, D(x) = 0 fi x=.
-1 - sin q 1 1 + a + a2
Solution of D(q) = 3 is Thus, D(x) = 0 for exactly one value of x.
(a) { }
p 3p
,
2 2
(b) { p 3p 5p 7p
, , ,
4 4 4 4 } Example 13: If

{ } { }
p 3p p p 3p a2 b2 c2 a2 b2 c2
(c) , (d) , , ,p 2 2 2
4 4 4 2 4 (a + l ) (b + l ) (c + l ) = k l a b c
Ans. (b) (a - l ) 2
( b - l )2 ( c - l )2 1 1 1
Solution: Using C1 Æ C1 + C3, we get
l π 0, then k is equal to:
2 sin q 1 (a) 4 labc (b) – 4 labc
D(q) = 0 1 sin q (c) 4 l2 (d) – 4 l2
0 - sin q 1 Ans. (c)
= 2 ( 1 + sin2q ) Solution: Using R3 Æ R3 – R2 and R2 Æ R2 – R1,
\ D(q) = 3 fi 2 sin2q = 1 we get
1 p 3p 5p 7p a2 b2 c2
fi sin q = ± fi q = , , ,
2 4 4 4 4 D = 2 al + l 2 2
2 bl + l 2cl + l 2
Example 12: Suppose a Œ R and x π 0. Let - 4 al - 4 bl - 4cl
1- x a a2 Take – 4l common from R3, and applying R2 Æ R2 – 2lR3,
D(x) = a a -x2
a3 we get
a 2 b2 c 2
a2 a3 a4 - x
D = - 4l 1 1 1
3
Number of values of x for which D(x) = 0 is
(a) 0 (b) 1 a b c
(c) 2 (d) 3 a2 b2 c2
Ans. (b) = l (4l 2 ) a b c
Solution: Using the sum property, write 1 1 1
D(x) = D1 –xD2
\ k = 4l2
where
1 a a2 Example 14: Let
D1 = a a -x2
a3 1 cos q -1
2 3 4 f (q) = - sin q 1 - cos q
a a a -x
-1 sin q 1
1 a a2 Suppose A and B are respectively maximum and minimum
and D2 = 0 a - x 2
a3 value of f (q). Then (A, B) is equal to:
(a) (2, 1) (b) (2, 0)
0 a3 a4 - x
(c) ( 2 , 1) (d) ÊÁ 2, 1 ˆ˜
= (a2 – x) (a4 – x) – a6 Ë
Ans. (b) 2¯
= – (a2 + a4)x + x2
Solution: Using C1 Æ C1 + C3, we get
In D1, use C2 Æ C2 – aC1, C3 Æ C3 – a2C1 to obtain
0 cos q -1
1 0 0
f (q) = - (sin q + cos q ) 1 - cos q
D1 = a -x 0 = x2
0 sin q 1
a2 0 -x Evaluating along C1, we get
4.8 Complete Mathematics—JEE Main

cos q -1 has a non-trivial solution, then ab + bc + ca equals:


f (q) = (sin q + cos q ) (a) a + b + c (b) a b c
sin q 1
= (sin q + cos q)2 (c) 1 (d) –1
2
Ans. (b)
= È 2 ÊÁ 1 sin q + 1 cos q ˆ˜ ˘ Solution: As the given system of equations has a non-
ÎÍ Ë 2 2 ¯ ˚˙
trivial solution,
Êp ˆ a - 1 -1 -1
= 2 sin 2 Ë + q ¯
4 D = -1 b - 1 -1 = 0
Êp ˆ -1 -1 c -1
As 0 £ sin 2 Ë + q ¯ £ 1, we get
4 Write
0 £ f (q) £ 2 a -1 -1 1 -1 -1
p D = 0 b - 1 -1 - 1 b - 1 -1
A = 2 for q =
4 0 -1 c - 1 1 -1 c - 1
p
and B = 0 for q = - 1 0 0
4
Thus, (A, B) = (2, 0) = a [(b - 1)(c - 1) - 1] - 1 b 0
1 0 c
Example 15: If a, b, c are non-zero real numbers and if [use C2 Æ C2 + C1
the system of equations C 3 Æ C 2 + C 1]
(a – 1)x = y + z,
= a(bc – b – c) – bc
(a – 1)y = z + x,
As D = 0, we get
(a – 1)z = x + y, ab + bc + ca = abc

LEVEL 1

Straight Objective Type Questions

Example 16: Let coefficient of x 2


7 6 x - 10 =– = 30.
coefficient of x 3
P(x) = 2 x - 10 5
Example 17: Let
x - 10 3 4
sum of zeros of P(x) is x 2 - 13 4 2
(a) 30 (b) 28 P(x) = 3 x 2 - 13 7
(c) 27 (d) 25 2
Ans. (a) 6 5 x - 13

Solution: Using R1 ´ R3, write If x = – 2 is a zero of P(x), then sum of the remaining five
zeros is
x - 10 3 4 (a) –2 (b) 0
P(x) = – 2 x - 10 5 (c) 2 (d) 3
7 6 x - 10 Ans. (c)
Solution:
TIP
TIP
Observe P(x) is of the form
P(x) = – (x –10)3 + a (x – 10) + b Observe
where a, b are some real numbers. P(x) = (x2 –13)3 + a (x2 – 13) + b.
where a, b are some real numbers.
\ sum of zeros of P(x) As coefficient of x5 in P(x) is 0, sum of six zeros of P(x) is 0.
fi sum of the remaining five zeros + (–2) = 0
Determinants 4.9
fi sum of the remaining five zeros = 2. 1
= a (a + b + c) D1 , where
Example 18: Suppose a, b are two real numbers and a
f(n) = a n + b n. Let a a+c a-b
3 1 + f (1) 1 + f (2) D1 = b b a+b
D = 1 + f (1) 1 + f (2) 1 + f (3) c c-a c
1 + f (2 ) 1 + f (3) 1 + f (4) Using C2 Æ C2 – C1, C3 Æ C3 – C1, we get
If D = k (a – 1)2 (b – 1)2 (a – b)2, then k is equal to a c -b
(a) 1 (b) 4 ab D1 = b 0 2 2 2
a = a (a + b + c )
(c) 9 (d) a2 b2
c -a 0
Ans. (a)
Solution: \ D = (a + b + c) (a2 + b2 + c2) = 0
As a, b, c are distinct real numbers, a2 + b2 + c2 π 0, therefore
1+1+1 1+a + b 1+ a2 + b2 a + b + c = 0.
D = 1+a + b 1+ a2 + b2 1 + a3 + b3 fi the line a (x – 5) + b(y – 2) + c = 0 passes through (6, 3).
1+ a2 + b2 1 + a3 + b3 1 + a 4 + b4 Example 20: Suppose a, b, c and x are real numbers.
1 1 1 1 1 1 Let
= 1 a b 1 a b = D12 1 + a 1 + ax 1 + ax 2
2 2 2 2
1 a b 1 a b D = 1 + b 1 + bx 1 + bx 2
1 1 1 1 + c 1 + cx 1 + cx 2
where D1 = 1 a b Then D is independent of
1 a2 b2 (a) a, b, c (b) x
Applying C3 Æ C3 – C2 and C2 Æ C2 – C1, we get (c) a, b, c, x (d) none of these
1 0 0 Ans. (c)
D1 = 1 a -1 b -a Solution: Write D = D1 + D2, where
2
1 a -1 b -a 2 2 1 1 + ax 1 + ax 2

1 1 D1 = 1 1 + bx 1 + bx 2
= (a - 1) ( b - a )
a +1 b +a 1 1 + cx 1 + cx 2
= (a –1) (b –1) (b – a) a ax ax 2
Thus, D = (a –1)2 (b –1)2 (a – b)2
and D2 = b bx bx 2 = 0
\ k =1
c cx 2
cx
Example 19: Suppose a, b and c are distinct real
[ C1 and C2 are propotional]
numbers. Let
In D1, use C2 Æ C2 – C1, C3 Æ C3 – C1 to obtain
a a+c a-b
D = b-c 1 ax ax 2
b a+b = 0
c+b c-a c D1 = 1 bx bx 2 = 0
Then the straight line a(x – 5) + b(y – 2) + c = 0 passes 1 cx cx 2
through the fixed point [ C2 and C3 are proportional]
(a) (5, 2) (b) (6, 2) Thus, D = 0 and hence independent of a, b, c, x.
(c) (6, 3) (d) (5, 3)
Ans. (c) Example 21: Suppose a, b, c > 1 and

Solution: Applying C1 Æ aC1 + bC2 + cC3, we get a- x ax x


a (a + b + c) a + c a - b f (x)= b -3 x b 3x
3x3 , x ΠR
1 -5 x 5x
Let D = b (a + b + c) b a+b c c 5x5
a
c (a + b + c) c - a c
4.10 Complete Mathematics—JEE Main
then f is 1 b c
(a) a constant function D1 = 0 c - b a - c
(b) a polynomial of degree 5
(c) an odd function 0 a-b b-c
(d) an even function = – (b – c)2 – (a – b) (a – c)
Ans. (d) = – (a2 + b2 + c2 – bc – ca – ab)
Solution: 1
fi D1 = – [(b – c)2 + (c – a)2 + (a – b)2] < 0
2
ax a- x -x
As a + b + c > 0, we get
f (–x) = b3 x -3 x
b -3 x 3 D = (a + b + c) D1 < 0
5x -5 x
c c -5 x 5 Example 24: Suppose A, B, C are angles of a triangle,
-x x
and let
a a x e2iA e-iC e-iB
= (-1)(-1) b -3 x b3 x 3x3 D = e -iC e2iB e-iA
-5 x 5x 5
c c 5x e-iB e-iA e2iC
= f(x) Then value of D is
Thus, f is an even function. (a) – 1 (b) – 4
Example 22: Suppose n, m are natural numbers and (c) 0 (d) 4
Ans. (b)
1 (1 + x )m (1 + mx )mn Solution: Taking eiA, common from R1, eiB from R2 and
iC
f(x) = (1 + mx )n 1 (1 + nx )mn e from R3, we get
(1 + nx )m (1 + x )n 1 D = ei(A + B + C) D1
where
constant term of the polynomial f (x) is:
eiA e-i( A + C ) e-i( A + B)
(a) 1 (b) m + n
(c) m – n (d) 0 D1 = e - i ( B + C ) eiB e-i( A + B)
Ans. (d) e-i( B + C ) e-i( A + C ) eiC
i(A + B + C) ip
Solution: Constant term of polynomial f (x) is f (0), and But A + B + C = p, so that e =e
1 1 1 = cos p + i sin p = –1. Also,
f (0) = 1 1 1 = 0 A + C = p – B fi e–i(A + C) = e–pi eiB = –eiB.
1 1 1 eiA -eiB -eiC
- iA iB
Example 23: Suppose a, b, c are sides of a scalene Thus, D1 = - e e -eiC
triangle. Let
-eiA -eiB eiC
a b c
D= b c a 1 -1 -1
= ei ( A+ B +C ) -1 1 -1
c a b
Then -1 -1 1
(a) D £ 0 (b) D < 0 Using C1 Æ C1 + C2, we get
(c) D > 0 (d) D ≥ 0 0 -1 -1
Ans. (b) D1 = (-1) 0 1 -1 = (–1) (–2) (2) = 4
Solution: Using C1 Æ C1 + C2 + C3, we get -2 -1 1
D = (a + b + c) D1 Therefore, D = (–1) D1 = – 4
where Example 25: Suppose x1, x2, x3 are real numbers such
1 b c
that x1 x2 x3 π 0. Let
D1 = 1 c a x1 + a1b1 a1b2 a1b3
1 a b D= a b x + a b a
2 1 2 2 2 2 b3
Applying R2 Æ R2 – R1, R3 Æ R3 – R1, we get a3 b1 a3 b2 x3 + a3 b3
Determinants 4.11

D 1 -4 20
Then - 1 equals:
x1 x2 x3 Example 27: Let D = 1 - 2 5 . Solution set of

a1b1 a2 b2 a3 b3 D = 0 is 1 2x 5x2
(a) + +
x1 x2 x3 (a) {– 2, 3} (b) {– 3, 4}
(b) – 1 (c) {4, – 6} (d) {– 2, – 1}
Ans. (d)
a1a2 a3 + b1b2 b3
(c) Solution: Applying R2 Æ R2 – R1, R3 Æ R3 – R1 we get
x1 x2 x3
1 -4 20
(d) 0 1 - 15
D= 0 2 - 15 = 2(x + 2)
Ans. (a) 1 5 ( x - 2)
0 2 ( x + 2) 5 ( x 2 - 4)
Solution: Using the sum property, write
D = x1 D1 + b1 D2 = 10(x + 2) (x + 1)
where Now, D = 0 fi x = – 2, – 1.
1 a1b2 a1b3
D1 = 0 x2 + a2 b2 a2 b3 1 a a 2 - bc
0 a3 b2 x3 + a3 b3 Example 28: Let D = 1 b b2 - ca , then D is equal to
= (x2 + a2 b2) (x3 + a3 b3) – a3 b2 a2 b3 1 c c 2 - ab
= x2 x3 + x2 a3 b3 + x3 a2 b2 (a) 0 (b) a + b + c
1 2
a1 a1b2 a1b3 (c) (a + b2 + c2) (d) none of these
and D2 = a2 2
x2 + a2 b2 a2 b3 Ans. (a)
a3 a3 b2 x3 + a3 b3 Solution: Applying R2 Æ R2 – R1, R3 Æ R3 – R1, we get
Using C2 Æ C2 – b2 C1, C3 Æ C3 – b3 C1, we get
1 a a 2 - bc
a1 0 0 D = (b – a) (c – a) 0 1 a + b + c = 0
D2 = a2 x2 0 = a1 x2 x3 0 1 a+b+c
a3 0 x3
[ R2 and R3 are identical]
Thus,
D = x1[x2 x3 + x2 a3 b3 + x3 a2 b2] + a1 b1 x2 x3 Example 29: Suppose a, b, c > 0 and a, b, c are the
pth, qth, rth terms of a G.P. Let
D ab a b ab 1 p log a
fi -1 = 1 1 + 2 2 + 3 3
x1 x2 x3 x1 x2 x3 D= 1 q log b
4 3 2 1 r log c
Example 26: Let pl + ql + rl + sl + t
l 2 + 3l l -1 l - 3 then numerical value of D is
(a) – 1 (b) 2
= l -1 - 2l l - 4 (c) 0 (d) none of these
l -3 l + 4 3l Ans. (c)
where p, q, r, s and t are constants. Then value of t is Solution: Let a = ARp – 1, b = AR q – 1 and c = ARr – 1
(a) 0 (b) – 1 fi log a = a + (p – 1)b, log b = a + (q – 1)b and
(c) 2 (d) 3 log c = a + (r – 1)b
Ans. (a) where a = log a, b = log R.
Solution: Putting l = 0, we obtain Now,
1 p a + ( p - 1) b
0 -1 3
D = 1 q a + (q - 1) b
t= 1 0 -4 = 0
1 r a + (r - 1) b
-3 4 0
as it is a skew symmetric determinant of odd order. Using C3 Æ C3 – (a – b)C1 – bC3, we get D = 0.
4.12 Complete Mathematics—JEE Main
Solution: If n is a multiple of 3, we get each element
Example 30: Let w =
1
2
(
– 1 + 3i and ) of D becomes 1.
\ D = 0.
1 1 1 If n = 3k + 1, then
D = 1 - 1 - w2 w 2 then D equals 1 w w2
2 4
1 w w D= w w2 1 = 0 [use C1 Æ C1 + C2 + C3]
(a) 3w (b) 3w (w – 1) w 2
1 w
(c) 3w2 (d) 3w (1 – w)
Ans. (b) If n = 3k + 2, then
Solution: Using w3 = 1 and 1 + w + w2 = 0, and apply- 1 w2 w
ing C1 Æ C1 + C2 + C3, we get D = w2 w 1 = 0 [use C1 Æ C1 + C2 + C3]
3 1 1 w 1 w2
2 2
D= 0 w w = 3(w – w) = 3w (w – 1)
Example 33: Let
2
0 w w b+c q+r y+z x a p
Example 31: Let a, b, and c be respectively the pth, qth D = c+a r+ p z + x and D1 = y b q
and rth terms of a harmonic progression and
a+b p+q x+y z c r,
1 1 1 then
D= p q r (a) D = 2D1 (b) D = – 2D1
bc ca ab (c) D = 4D1 (d) D = – 4D1
Ans. (a)
Then numerical value of D is
Solution: Using R1 Æ R1 + R2 + R3, we get
(a) 0 (b) – 1
(c) 1 (d) none of these 2(a + b + c) 2( p + q + r ) 2( x + y + z )
Ans. (a)
D= c+a r+p z+x
1 1 1 a+b p+q x+y
Solution: , , are the pth, qth, rth terms of an A.P.
a b c
Taking 2 common from R1 and applying R2 Æ R2 – R1,
1 1 R3 Æ R3 – R1, we obtain
Let = A + (p – 1)D, = A + (q – 1)D,
a b a+b+c p+q+r x+y+z
1 D =2 -b -q -y
and = A + (r – 1)D.
c
-c -r -z
Now
1 1 1 Applying R1 Æ R1 + R2 + R3, we get
D = abc p q r a p x a x p
A + ( p - 1)D A + (q - 1)D A + (r - 1)D D = 2(– 1) (– 1) b q y =–2 b y q
Using R3 Æ R3 – (A – D)R1 – DR2, we get c r z c z r
D = 0.
x a p
Example 32: Let w π 1 be complex cube root of unity =2 y b q = 2D1
and n be a natural number and
z c r
1 wn w 2n
x+y x x
D = wn w 2n 1
Example 34: If x = – 2, and D = 5 x + 4 y 4 x 2 x
w 2n 1 wn
then numerical value of D is 10 x + 8 y 8 x 3 x
Then D equals
(a) 0 (b) 1 (a) 8 (b) – 8
(c) w (d) w2 (c) 4 (d) – 4
Ans. (a) Ans. (b)
Determinants 4.13
Solution: Taking x common from C2 and C3 we obtain Solution: Using change of base formula
1 1 1 log b
x+y 1 1 logab = , a, b > 0, a, b π 1,
log a
D = x2 5 x + 4 y 4 2 = x3 5 4 2
10 8 3 we can write
10 x + 8 y 8 3 log x log y log z
[using C1 Æ C1 – yC2] 1
D= log x log y log z = 0.
Next using C1 Æ C1 – C2, C2 Æ C2 – C3, we obtain log x log y log z
log x log y log z
0 0 1
1 2 Example 37: Let w π 1 be a cube root of unity and
D = x3 1 2 2 = x3 = x3
2 5
2 5 3 1 - w - w2 2 2
As x = – 2, D = – 8. D= 2w w -w -1 2
2w
Example 35: If a = w π 1, is a cube root of unity, b = 2w 2 2w 2 w2 - 1 - w
– 785, c = 2008 i, and then D equals
a a+b a+b+c (a) – w (b) 3w (1 – w)
D = 2 a 3a + 2b 4 a + 3b + 2c (c) 0 (d) 1 – w2
3a 6a + 3b 10a + 6b + 3c Ans. (c)
then D equals Solution: Using R1 Æ R1 + R2 + R3 and 1 + w + w2 =
(a) – i (b) i 0, we obtain
(c) 1 (d) 1 – wi D = 0.
Ans. (c)
Solution: Write
Example 38: Suppose x = –
1
3
(1 + 7i and )
1 a+b a+b+c p p
y = cos + isin .
D = a 2 3a + 2b 4a + 3b + 2c 4 4
3 6a + 3b 10a + 6b + 3c 1 x x
and apply C2 Æ C2 – bC1, C3 Æ C3 – cC1 to obtain Let D= 1 x + y y ,
1 a a+b 1 x x+y
D = a 2 3a 4a + 3b then D equals
3 6a 10 a + 6c (a) - 7 (b) 7
(c) i (d) – 1.
Take a common from C2 and apply C3 Æ C3 – bC2 to obtain Ans. (c)
1 1 a 1 1 1 Solution: Using R2 Æ R2 – R1 and R3 Æ R3 – R1, we get
D = a2 2 3 4 a = a3 2 3 4
1 x x
3 6 10a 3 6 10 p p
D= 0 y y - x = y2 = cos + i sin = 0 + i(1) = i
Apply C3 Æ C3 – C2, C2 Æ C2 – C1 to obtain 2 2
0 0 y
1 0 0 [Using De Moivre’s Theorem]
D = a3 2 1 1 = a3 = w3 = 1.
p p
3 3 4 Example 39: Let x = cos + i sin and
3 3
Example 36: Let x, y, z be positive and x, y, z π 1. Let
1 x x2
1 log x y log x z
D = x2 1 x
D = log y x 1 log y z ,
2
log z x log z y 1 1 x 1
then numerical value of D is
then numerical value of D is
(a) 0 (b) – 1
(a) – 1 (b) 0
(c) 8 (d) – 4
(c) 1 (d) none of these
Ans. (c)
Ans. (b)
4.14 Complete Mathematics—JEE Main
Solution: Use C3 Æ C3 – xC2 and C2 Æ C2 – xC1. Example 42: Let f : N Æ N be defined by
2
1 0 0
f (x) = ( x + 1)2 + x - ÈÎ ( x + 1)2 + ( x + 1) ˘˚
D = x2 1 – x3 0 ([ ] denotes the greatest integer function). Suppose a, b, c
1 x – x 1 – x3
2 are three distinct natural numbers. Let
= (1 – x3)2 = [1 – cosp – isinp]2 = 8 f (a) a 2 a
[De Moivre’s Theorem] D = f (b ) b 2 b
Let f(x) = [2 - x [2 x ]], f (c ) c 2
2 2
Example 40: x ΠR. ([ ] denotes c
the greatest integer function.) Let x1 = 0, x2 = log23 and x3 = The D is equal to:
2 . Suppose 0 < x4 < 1. (a) – (a + b + c) (b) a + b + c
f ( x1 ) f ( x2 ) f ( x3 ) (c) –1 (d) 0
Ans. (d)
D = f ( x4 ) f ( x4 ) f ( x2 ) , then D is equal to:
Solution: If n ΠN, then
f ( x2 ) f ( x3 ) f ( x1 )
n2 < n2 + n < (n + 1)2
(a) –1 (b) 0
(c) 1 (d) 2 log23 fi n< n2 + n < n + 1
Ans. (b)
2
fi [ n2 + n ] = n
x2 x2 [2 x ]
Solution: 0 £ [2 ] £ 2 fi 0 £ 2
£ 1. Therefore, fi ÈÎ ( x + 1)2 + ( x + 1) ˘˚ = x + 1 " x Œ N.
2x
range f (x) is {0, 1}
2 2 2 \ f (x) = (x + 1)2 + x Р(x + 1)2 = x " x ΠN.
Also [2x ] = 2x ¤ 2x is a positive integer.
Thus,
Thus, f (x1) = 1, f (x2) = 1, f (x3) = 1 and f (x4) = 0.
a a2 a
Therefore,
D= b b 2
b =0
1 1 1 2
c c c
D= 0 0 1 = 0
[ C1 and C3 are identical]
1 1 1
Example 41: If w π 1 is a cube root of unity and x 3 7
Example 43: If x = – 9 is a root of 2 x 2 = 0, then
x + w2 w 1
the other two roots are 7 6 x
D= w w2 1 + x = 0, then value of x is
(a) 3, 7 (b) 2, 7
1 x +w w2 (c) 3, 6 (d) 2, 6
(a) 0 (b) 1 Ans. (b)
(c) – 1 (d) none of these Solution: Applying R1 Æ R1 + R2 + R3, we get
Ans. (a) x+9 x+9 x+9
Solution: Applying C1 Æ C1 + C2 + C3, we get D= 2 x 2
x + w2 + w +1 w 1 7 6 x
D = w +w +1+ x 2
w 2
1+ x Taking x + 9 common from R1 and applying C2 Æ C2 – C1,
C3 Æ C3 – C1 we get
1+ x + w + w2 x +w w2
1 0 0
x w 1 D = (x + 9) 2 x - 2 0
= x w2 1 + x using 1 + w + w2 = 0 7 -1 x - 7
x x +w w2 = (x + 9) (x – 2) (x – 7)
Thus, D = 0 fi x = – 9, 2, 7
D is clearly equal to 0 for x = 0.
\ the other two roots are 2 and 7.
Determinants 4.15

x b b Solution: Multiplying R3 of D1 by xyz, we get


x b
Example 44: If D1 = a x b and D2 = , then ax by cz a b c
a x
a a x 1 xyz
D1 = x2 y2
z2 = x y z = D2
d xyz xyz
(a) D1 = 3 (D2)2 (b) (D1) = 3D2 xyz xyz xyz yz z x xy
dx
d [taking x, y, z common from C1, C2, C3 respectively]
(c) (D1) = 3D22 (d) D1 = 3D3/2
2 fi D1 – D2 = 0.
dx
Ans. (b) Example 47: If
Solution: We have
1 x x +1
1 b b x 0 b x b 0
d D1 D(x) = 2x x ( x - 1) x ( x + 1)
= 0 x b + a 1 b + a x 0
dx
0 a x a 0 x a b 1
3 x ( x - 1) x ( x - 1) ( x - 2 ) x x 2 - 1 ( )
then D(100) equals
x b x b x b
= + + (a) 0 (b) – 100
a x a x a x (c) 100! (d) – 100!
= 3D2. Ans. (a)
Example 45: If x e R and n e I, then the determinant Solution: Taking x common from C2, x + 1 from C3 and
sin ( np ) sin x - cos x log tan x x – 1 from R3, we get
D = cos x - sin x cos [(2n + 1) p / 2 ] log cot x 1 1 1
log cot x log tan x tan ( np ) D(x) = x(x + 1) (x – 1) 2 x x - 1 x
equals 3x x - 2 x
(a) 0 (b) log tan x – log cot x
Applying C1 Æ C1 – C3, C2 Æ C2 – C3, we get
(c) tan (p/4 – x) (d) none of these
Ans. (a) 0 0 1
Solution: We can write D as D(x) = x(x + 1) (x – 1) x -1 x = 0
0 sin x - cos x log tan x 2 x -2 x
D = - (sin x - cos x ) 0 - log tan x [ C1 and C2 are proportional]
- log tan x log tan x 0 Thus, D(100) = 0
0 - (sin x - cos x ) - log tan x
= (– 1)3 sin x - cos x 0 log tan x 1 1 1
log tan x - log tan x 0 Example 48: If D(x) = (e + e ) (p + p )
x -x 2 x -x 2
2
[taking – 1 common from R1, R2 and R3]
(e x - e- x ) (p x - p - x )
2 2
= – D [using the reflection property] -2
fi 2D = 0 fi D = 0. then D(x) equals
(a) x2 (b) x2 – 1
ax by cz
2 2

Example 46: Let D1 = x 2 y2 z 2 and (c) e x - p x (d) 0


1 1 1 Ans. (d)
Solution: Using R2 Æ R2 – R3 and (ax + a–x )2 – (ax – a–x )2
a b c
= 4ax a–x = 4, we get
D2 = x y z , then D1 – D2 equals
yz z x xy 1 1 1
(a) (x – 1) (y – 1) (z – 1) D(x) = 4 4 4 =0
(b) (x – y) (y – z) (z – x)
(e x - e- x ) (p x - p - x )
2 2
-2
(c) abc (x – y) (y – z) (z – x)
(d) 0 [since R1 and R2 are proportional]
Ans. (d)
4.16 Complete Mathematics—JEE Main

1 cos x 1 - cos x (a) – sin q – cos q (b) sin 2q


(c) 1 + sin 2q – cos 2q (d) 0
Example 49: If D(x) = 1 + sin x cos x 1 + sin x - cos x ,
Ans. (d)
sin x sin x 1
p /2 Solution: Applying R2 Æ R2 + R3, and using
then Ú D( x ) dx equals
0 sin (A + B) + sin(A – B) = 2 sin A cos B and
(a) 1/4 (b) 1/2 cos (A + B) + cos (A – B) = 2 cos A cos B, we get
(c) 0 (d) – 1/2 sin q cos q
Ans. (d)
D = 2 sin q cos (2p / 3) 2 cos q cos (2p / 3)
Solution: Using C1 Æ C1 – C2 – C3, we get
sin (q - 2p / 3) cos (q - 2p / 3)
0 cos x 1 - cos x
sin 2q
D(x) = 0 cos x 1 + sin x - cos x
2 sin 2q cos ( 4p / 3)
-1 sin x 1
sin (2q - 4p / 3)
cos x 1 - cos x
= (– 1) sin q cos q sin 2q
cos x 1 + sin x - cos x
D = - sin q - cos q - sin 2q
= (– 1) cos x [1 + sin x – cos x – 1 + cos x]
sin (q - 2p / 3) cos (q - 2p / 3) sin (2q - 4p /3)
1
= – cos x sin x = - (sin 2x) since cos (2p/3) = cos (p – p/3) = – cos (p/3) = – 1/2
2
p /2
and cos (4p/3) = cos (p + p/3) = – cos (p/3) = – 1/2
p /2 1 ( -1) ˘ fi D=0
\ Ú0 D(x) dx = -
2 2
cos 2x ˙
˚0
[ R1 and R2 are proportional.]
Example 52: If a, b, g are the roots of x3 + px2 + q = 0,
1 1 where q π 0, and
= (cos p – cos 0) = -
4 2 1a 1b 1g
Example 50: The determinant D= 1 b 1 g 1 a
a b aa + b 1g 1a 1b
D= b c ba + c equals zero, if then D equals
(a) – p/q (b) 1/q
a a + b ba + c 0
(c) p2/q (d) 0
(a) a, b, c are in A.P. Ans. (d)
(b) a, b, c are in H.P.
Solution: We have bg + ga + ab = 0. We can write D as
(c) x – a is a factor of ax2 + 2bx + c
(d) x – a is a factor of ax2 + bx + c bg ga ab
Ans. (c) 1
D = ga ab bg
Solution: Applying C3 Æ C3 – aC1 – C2, we get a 3 b 3g 3
ab bg ga
a b 0 bg + ga + ab ga ab
1
D = b c 0 = ga + ab + bg ab bg
a 3 b 3g 3
(
a a + b ba + c - aa 2 + 2ba + c ) ab + bg + ga bg ga

a b [using C1 Æ C1 + C2 + C3]
= – (aa2 + 2ba + c) [expanding along C3] 0 ga ab
b c 1
2
= – (aa + 2ba + c) (ac – b ) 2 = 0 ab bg = 0 [all zero property]
a 3 b 3g 3
= 0 if x – a is a factor of ax2 + 2bx + c. 0 bg ga
Example 51: If q e R, then the determinant Example 53: If A, B, C are the angles of a triangle, and
sin q cos q sin 2q -1 cos C cos B
D = sin (q + 2p / 3) cos (q + 2p / 3) sin (2q + 4p / 3) D = cos C -1 cos A
sin (q - 2p / 3) cos (q - 2p / 3) sin (2q - 4p / 3) cos B cos A -1
equals then D equals
Determinants 4.17
(a) 0 (b) – 1 Ans. (a)
(c) 2 cos A cos B cos C (d) none of these Solution: Applying R3 Æ R3 – cos j R1 + sin j R2, we
Ans. (a) get
Solution: Applying C1 Æ aC1 + bC2 + cC3, we get cos q - sin q 1
- a + b cos C + c cos B cos C cos B D = sin q cos q 1
1 0 0 sin j - cos f
D= a cos C - b + c cos A -1 cos A
a
a cos B + b cos A - c coss A -1 = (sin j – cos j) (cos2 q + sin2 q)

=
1
0 cos C cos B
0 -1 cos A = 0
= 2 { 1
2
sin j -
1
2 }
cos j = 2 sin (f – p/4)
a
0 cos A -1 As – 1 £ sin (f – p/4) £ 1, - 2 £ 2 sin (j – p/4) £ 2
[using projection formulae and all zero property.] or - 2 £ D £ 2 .
Example 54: If a, b, c are three complex numbers such b+c a-b a
that a2 + b2 + c2 = 0 and Example 56: If D1 = c + a b - c b and
b2 + c2 ab ac a+b c-a c
D= ab c2 + a2 bc = ka2 b2 c2, a b c
ac bc a +b
2 2 D2 = b c a , then D1 – D2 equal
c a b
then the value of k is
(a) 1 (b) 2 (a) 0 (b) 3abc
(c) – 2 (d) 4 (c) 6abc (d) 2(a3 + b3 + c3)
Ans. (d) Ans. (a)
Solution: Using a2 + b2 + c2 = 0, we can write D as Solution: Using C1 Æ C1 + C3 and C2 Æ C2 – C3 in
D1, we get
-a2 ab ac -a a a a + b + c -b a
D= ab -b2 bc = abc b -b b D1 = a + b + c - c b
ac bc -c2 c c -c a + b + c -a c
[taking a, b, c common from C1, C2, C3 respectively] Using C1 Æ C1 + C2 – C3, we get
c -b a a -b c a b c
-1 1 1
D1 = a - c b = - b - c a = b c a
= a2 b2 c2 1 -1 1
b -a c c -a b c a b
1 1 -1
[taking a, b, c common from R1, R2, R3 respectively] fi D1 = D2 fi D1 – D2 = 0.
0 2 1 Example 57: If x, y, z are different from zero and
2 1 a b-y c-z
= a2 b2 c2 2 0 1 = – 2a2 b2 c2 = 4a2 b2 c2
0 0 -1 0 -1 D= a- x b c - z = 0,
[applying C1 Æ C1 + C3 and C3 Æ C2 + C3] a-x b-y c
Thus, k = 4. a b c
then the value of the expression + + is
Example 55: If q, f e R, then the determinant x y z
cos q - sin q 1 (a) 0 (b) –1
D= sin q cos q 1 (c) 1 (d) 2
Ans. (d)
cos (q + f ) - sin (q + f ) 0
lies in the interval Solution: Applying R2 Æ R2 – R1, R3 Æ R3 – R1, we get
a b-y c-z
(a) ÈÎ- 2 , 2 ˘˚ (b) [– 1, 1]
D = -x y 0 =0
(c) ÈÎ- 2 , 1˘˚ (d) ÈÎ-1, 2 ˘˚ -x 0 z
4.18 Complete Mathematics—JEE Main
Expanding along C3, we get 21 31
(a) (b)
-x y a b-y 10 10
(c – z) +z =0 (c) – 5 (d) 4
-x 0 -x y
Ans. (a)
fi (c – z) (xy) + z(ay + bx – xy) = 0
Solution:
fi cxy – xyz + ayz + bxz – xyz = 0
From (1) and (2), 2x + z = 0 fi 2x = – z.
fi ayz + bzx + cxy = 2xyz
From (3), we get 3y = 5z
a b c
fi + + =2 1 5
x y z \ x= – z, y = z
2 3
Example 58: If p + q + r = a + b + c = 0, then the
Substituting this in (1) we get
determinant
pa qb rc 1 5k
– z+ z – 3z = 0. fi Ê 5k - 7 ˆ z = 0
2 3 Ë 3 2¯
D = qc ra pb equals
rb pc qa 21
Now, z π 0 fi k= .
(a) 0 (b) 1 10
(c) pa + qb + rc (d) none of these Example 61: If a ΠR and the system of equations x +
Ans. (a) ay = 0, az + y = 0, ax + z = 0 has infinite number of solution
Solution: We have then the value of a is
(a) – 1 (b) 1
D = pqr (a3 + b3 + c3) – abc (p3 + q3 + r3)
(c) 0 (d) no real value
But a + b + c = 0 fi (a + b)3 = – c3
Ans. (a)
fi a3 + b3 + 3ab(a + b) + c3 = 0
Solution: We have x = – ay, therefore
fi a3 + b3 + c3 = – 3ab(– c) = 3abc – a2y + z = 0 fi z = a2y.
Similarly, p3 + q3 + r3 = 3pqr Thus, az + y = 0 fi (a3 + 1)z = 0.
Thus, D = pqr (3abc) – abc (3pqr) = 0 For the system of equations to have infinite number of solu-
Example 59: The system of equations tions a3 + 1 = 0
lx + y + z = 0 fi a=–1 [\ a is real]
– x + ly + z = 0 Example 62: Given 2x – y + 2z = 2,
– x – y + lz = 0 x – 2y + z = – 4,
will have a non-trivial solution if real values of l are x + y + lz = 4
(a) 0, 1 (b) 0, – 1 then the value of l such that the given system of equations
(c) 0, 2 (d) 0 has no solution, is
Ans. (d) (a) 3 (b) 1
Solution: The system of equation will have a non-triv- (c) 0 (d) – 3
ial solution if and only if Ans. (b)
D = 0 where Solution: From the first two equations, we get
x + y= 6 – z
l 1 1
Putting this value in the last equation we get
D = - 1 l 1 = l3 + 3l = l (l2 + 1)
6 + (– 1 + l) z = 4 = (l – 1)z = – 2.
-1 -1 l For the system of equations to have no solutions, l = 1
Thus, D = 0, l ΠR fi l = 0
Example 63: If the system of linear equations
Example 60: The values of k for which the system of x + 2ay + az = 0
equations x + 3by + bz = 0
x + ky – 3z = 0 (1)
x + 4cy + cz = 0
3x + ky – 2z = 0 (2)
has a non-zero solution, then a, b, c
2x + 3y – 4z = 0 (3) (a) are in G.P.
has a non-trivial solution is (are) (b) are in H.P.
Determinants 4.19
(c) satisfy a + 2b + 3c = 0 (a) – 2 (b) 1
(d) are in A.P. (c) – 2 (d) either – 2 or 1
Ans. (b) Ans. (a)
Solution: The system will have a non-zero solution if Solution: Let
1 2a a a 1 1
D = 1 3b b = 0 D= 1 a 1
1 4c c 1 1 a
Using C2 Æ C2 – 2C3, we get Using C1 Æ C1 + C2 + C3, we get
1 0 a 1 1 1 1 1 1
D = 1 b b =0 D = (a + 2) 1 a 1 = (a + 2) 0 a - 1 0
1 2c c 1 1 a 0 1 a -1
fi 1(bc – 2bc) + a(2c – b) = 0 fi 2ac = bc + ab [using R2 Æ R2 – R1, R3 Æ R3 – R1]
2ac \ D = (a + 2) (a – 1)2
fi = b fi a, b, c are in H.P. For no solution, D = 0 fi a = – 2, 1.
a+c
For a = 1, the system of equations has infinite number of
Example 64: The system of homogenous equations solutions.
(a – 1)x + (a + 2)y + az = 0 For a = – 2, on adding the three equations we obtain
0= – 9
(a + 1)x + ay + (a + 2)z = 0
Thus, system of equations has no solution for a = – 2.
ax + (a + 1)y + (a – 1)z = 0
Example 66: Suppose a, b, c, a Œ R and abc a π 0. If
has a non-trivial solution if a equals
the system of equations:
1 1 (a + a)x + ay + az = 0 (1)
(a) (b) –
2 2 ax + (b + a)y + az = 0 (2)
(c) 2 (d) – 1. ax + ay + (a + c)z = 0 (3)
Ans. (b) Ê 1 1 1ˆ
has a non-trivial solution, then a Ë + + ¯ is equal to
Solution: The system of equations will have a non- a b c
trivial solution if D = 0 where (a) – 1 (b) 0
a -1 a + 2 a (c) abc (d) bc + ca + ab
Ans. (a)
D = a +1 a a+2
Solution: From (1) and (2)
a a +1 a -1
ax – by = 0
Using C1 Æ C1 – C3 and C2 Æ C2 – C3, we obtain and from (2) and (3)
-1 2 a ax – cz = 0
D = -1 - 2 a + 2 \ ax = by = cz
1 2 a -1 x y z
fi = =
1/ a 1/ b 1/ c
Using R3 Æ R3 + R2, we get
Putting in (1) we get
-1 2 a a +a a a
+ + =0
D = - 1 - 2 a + 2 = (2a + 1) (2 + 2) = 4(2a + 1) a b c
0 0 2a + 1 Ê 1 1 1ˆ
fi a Ë + + ¯ = –1
Now, D = 0 fi a = – 1/2. a b c
Example 67: Suppose a, b, Œ R and a, b π 1. If the
Example 65: The system of equations system of equations:
ax + y + z = a – 1 ax + y + z = 0 (1)
x + ay + z = a – 1 x + by + z = 0 (2)
x + y + az = a – 1 x + y + 2z = 0 (3)
has no solution, if a equals has a non-trivial solution, then
4.20 Complete Mathematics—JEE Main
(a) a + b = 2 (b) a + b = ab (a) 0 (b) p/48
1 p p
(c) a + = 2 (d) a + b = 0 (c) - - (d) none of these
b 2 15 2
Ans. (a) Ans. (d)
Solution: From (1) and (3) Solution: Applying R2 Æ R2 – R3, we get
(a – 1)x – z = 0
sec x cos x sec 2 x + cot x cosec 2 x
and from (2) and (3)
f(x) = - sin 2 x 0 0
(b – 1)y = z
2
x y z 1 cos x cosec 2 x
\ = =
1 1 1 cos x sec 2 x + cot x cosec 2 x
(1 - a ) (1 - b) = – (– sin2 x)
cos2 x cosec 2 x
Putting in (1) we get 2 2
= sin x [cos x cosec x
a 1 – cos2 x (sec2 x + cot x cosec2 x)]
+ +1 = 0
1- a 1- b
cos3 x
a 1 = cos x – sin2 x –
fi =- sin x
1- a 1- b
fi 1 – b = –1 + a 1 Ê 1 ˆ
= cos x – (1 – cos 2x) – Á - sin x˜ cos x
2 Ë sin x ¯
fi a + b =2
p /2 p /2 1 p /2
Example 68: If a2 + b2 + c2 = – 2 and Thus, Úp / 4 f(x) dx = Úp / 4 cosx dx –
2 Úp / 4 dx
1 + a2 x (1 + b2 ) x (1 + c 2 ) x
1 p /2 p /2 Ê 1 ˆ
f(x) = (1 + a 2 ) x 1 + b2 x (1 + c 2 ) x +
2 Úp / 4 cos 2x dx – Úp / 4 ÁË sin x - sin x˜¯ cos xdx
(1 + a 2 ) x (1 + b2 ) x 1 + c2 x 1
1 1Êp pˆ 1 Ê t2 ˆ ˘
then f(x) is a polynomial of degree = 1- - Á - ˜ + (0 - 1) - Á log t - ˜ ˙
(a) 3 (b) 2 2 2 Ë 2 4¯ 4 Ë 2¯ ˚1
2
(c) 1 (d) 0 where t = sin x
Ans. (b) 1 p 1
= 1- - - log 2
Solution: Applying C1 Æ C1 + C2 + C3, we get 2 8 2

1 (1 + b2 ) x (1 + c 2 ) x Example 70: If a + b + c = 0, then a root of the equation


f(x) = 1 1+ b x2 2
(1 + c ) x a-x c b
1 (1 + b2 ) x 1 + c2 x D= c b-x a = 0 is
b a c-x
[using a2 + b2 + c2 = – 2]
Applying R2 Æ R2 – R1 and R3 Æ R3 – R1, we get (a) 1 (b) – 1
(c) a2 + b2 + c2 (d) 0
1 (1 + b2 ) x (1 + c 2 ) x Ans. (d)
f(x) = 0 1- x 0 = (1 – x)2 Solution: Applying C1 Æ C1 + C2 + C3, we get
0 0 1- x
a+b+c-x c b
which is a polynomial of degree 2.
D = a+b+c-x b-x a
Example 69: a+b+c-x a c-x
sec x cos x sec 2 x + cot x cosec 2 x -x c b
Let f(x) = cos2 x cos2 x cosec 2 x = -x b - x a [ a + b + c = 0]
1 cos2 x cosec 2 x -x a c-x
p /2 D clearly equals 0 when x = 0.
then value of Úp / 4 f(x)dx is It is unnecessary to evaluate the determinant further.
Determinants 4.21
Example 71: A root of the equation cos2 A + sin 2 A cos A cos B + sin A sin B
0 x-a x-b = cos A cos B + sin A sin B cos2 B + sin 2 B
D= x+a 0 x - c = 0 is
cos C cos A + sin C sin A cos B cos C + sin B sin C
x+b x+c 0
cos C cos A + sin A sin C
1
(a) (a + b + c) (b) 0 cos B cos C + sin B sin C
2
(c) –1 (d) 1 cos2 C + sin 2 C
Ans. (b) cos A sin A 0 cos A sin A 0
Solution: When we substitute x = 0, D becomes = cos B sin B 0 cos B sin B 0 = (0) (0) = 0.
0 - a -b cos C sin C 0 cos C sin C 0
a 0 -c
Example 73: If a, b, c are the sides of a D ABC opposite
b c 0
angles A, B, C respectively, and
which is equal to 0 as D is skew symmetric determinant of a2 b sin A c sin A
odd order. D = b sin A 1 cos ( B - C ) , then D equals
Alternative Solution c sin A cos ( B - C ) 1
Evaluating D along R1, we get (a) sin A – sin C sin B (b) abc
(c) 1 (d) 0
x+a x-c x+a 0 Ans. (d)
– (x – a) + (x – b) =0
x+b 0 x+b x+c Solution: By the law of sines
fi (x – a) (x + b) (x – c) + (x – b) (x + a) (x + c) = 0 a b c
= = = k (say)
fi x3 + (– a + b – c)x2 + (– ab + ac – bc)x + abc + x3 sin A sin B sin C
+ (a – b + c)x2 + (– ab + ac – bc) – abc = 0
fi a = k sin A, b = k sin B, c = k sin C. Now
fi 2x3 – 2(ab – ac + bc)x = 0
a2 ab / k ac / k
fi 2x[x2 – (ab – ac + bc)] = 0
D = ab / k 1 cos ( B - C )
fi x = 0 or x = ± ab - ac + bc ac / k cos ( B - C ) 1
\ one of the roots of the equation is 0.
1 sin B sin C
Example 72: If a, b, g are three real numbers such that = a2 sin B 1 cos ( B - C )
a + b + g = 0, then sin C cos ( B - C ) 1
1 cos g cos b 1 sin ( A + C ) sin ( A + B )
D = cos g 1 cos a equals = a2 sin ( A + C ) 1 cos ( B - C )
cos b cos a 1 sin ( A + B ) cos ( B - C ) 1
(a) – 1 (b) 0
(c) 1 (d) cos a cos b cos g sin A cos A 0 sin A cos A 0
Ans. (b) = a2 cos C sin C 0 cos C sin C 0
Solution: Let A, B and C be three real numbers such cos B sin B 0 cos B sin B 0
that a = B – C, b = C – A and g = A – B, clearly a + b + g = a2(0) = 0
= 0. We have
1 cos ( A - B ) cos (C - A) 1 1 1
Example 74: If a, b, c are distinct, and a b c
D = cos ( A - B ) 1 cos ( B - C )
cos (C - A) cos ( B - C ) 1 = (b – c) (c – a) (a – b) (a + b + c) a3 b3 c3
4.22 Complete Mathematics—JEE Main

1 1 1 a b g
then D = ( x - a)2
( x - b)2
( x - c)2 1-a 1- b 1- g
D= equals
( x - b) ( x - c) ( x - c) ( x - a ) ( x - a ) ( x - b) a b g
vanishes if a2 b2 g2
1
(a) x = (a + b + c) 25 d 25 d
3 (a) (b)
2a a
2
(b) x = (a + b + c) -25d
3 (c) (d) none of these
(c) x = a + b + c a+b+c+d
(d) none of these Ans. (d)
Ans. (a) Solution: Taking a, b, g common from C1, C2, C3
Solution: Multiplying C1 by (x – a), C2 by (x – b) and C3 by respectively, we get
(x – c), we get 1 1 1
A B C 1-a 1- b 1- g
1 D = abg
D= A3 B3 C3 1 1 1
ABC
ABC ABC ABC a b g
where A = x – a, B = x – b, C = x – c. 1 1 1 1 1
- -
A B C 1 1 1 1-a 1- b 1-a 1- g 1-a
= abg
3
B3 C 3 = (– 1) (– 1) A 1 0 0
D= A B C
1 1 1 a b -a g -a
A3 B3 C 3
[using C2 Æ C2 – C1, and C3 Æ C3 – C1]
= (B – C) (C – A) (A – B) (A + B + C)
= (c – b) (a – b) (b – a) [3x – (a + b + c)] abg ( -1) ( b - a ) (g - a ) 1 - g 1- b
=
1
Note that D become 0 when x = (a + b + c) (1 - a ) (1 - b ) (1 - g ) 1 1
3
abg (a - b ) ( b - g ) (g - a )
Example 75: The equation =
(1 - a ) (1 - b ) (1 - g )
x-a x-b x-c
D= x-b x-c x-a =0 As a, b, g are the roots of ax3 + bx2 + cx + d = 0,
x-c x-a x-b ax3 + bx2 + cx + d = a(x – a) (x – b) (x – g)
is satisfied when and abg = – d/a
1 1 (- d / a )(25 / 2 ) 25 d
(a) x = (a + b + c) (b) x = (a + b + c) Thus, D= = -
3 2 (a + b + c + d ) / a 2 (a + b + c + d )
(c) x = a + b + c (d) x = 0
Example 77: Let P = [aij] be a 3 ¥ 3 matrix and Q = [bij],
Ans. (a)
where bij = 2 i + j aij for l £ i, j £ 3. If the determinant of P is
Solution: Using C1 Æ C1 + C2 + C3, we get 2, then the determinant of the matrix Q is
3 x - (a + b + c ) x - b x-c (a) 210 (b) 211
D = 3 x - (a + b + c ) x - c x-a (c) 212 (d) 213
3 x - (a + b + c ) x - a x-b Ans. (d)
1 Solution:
Note that D becomes 0 when x = (a + b + c).
3 22 a11 23 a12 2 4 a13
Example 76: If a, b, g are different from 1 and are the det (Q)= 2 a213 4
2 a22 25 a23
roots of ax3 + bx2 + cx + d = 0 and (b – g) (g – a) (a – b) =
2 4 a31 25 a32 26 a33
25/2, then the determinant
Determinants 4.23

a11 a12 a13 a + 1 a - 1 (- 1)n + 2 a


2 3 4
= (2 )(2 )(2 ) 2a21 2a22 2a23 = b + 1 b - 1 (- 1)n + 1 b
2 2 2
2 a31 2 a32 2 a33 c -1 c +1 (- 1)n c

a11 a12 a13 [using the reflection property]


= 29 (2) (22) a21 a22 a23 = 212 det (P) = 213 (- 1)n + 2 a a + 1 a - 1
a31 a32 a33 = (–1)2 (- 1)n + 1 b b +1 b -1
Example 78: If x is a positive integer, and D(x) = (- 1)n c c -1 c +1
x! ( x + 1)! ( x + 2)!
Thus,
( x + 1)! ( x + 2)! ( x + 3)! , then D(x) is equal to
a + (- 1)n + 2 a a +1 a -1
( x + 2)! ( x + 3)! ( x + 4)!
n +1
D = - b + (- 1) b b +1 b -1
(a) 2x! (x + 1)!
(b) 2x! (x + 1)! (x + 2)! c + (- 1)n c c -1 c +1
(b) 2x! (x + 3)! The first column consists of all 0’s if n is any odd integer.
(d) 2(x + 1)! (x + 2)! (x + 3)! \ D = 0 if n is any odd integer.
Ans. (b) 2p
Example 80: Let w be the complex number cos
Solution: Taking x! common from R1, (x + 1)! from R2 3
and (x + 2)! from R3 to obtain 2p
D(x) = x! (x + 1)! (x + 2)! D1(x) where + i sin . Then the number of distinct complex number z
3
1 x + 1 ( x + 1) ( x + 2) z +1 w w2
D1(x) = 1 x + 2 ( x + 2) ( x + 3)
satisfying w z + w2 1 = 0 is
1 x + 3 ( x + 3) ( x + 4) 2
w 1 z +w
Applying R3 Æ R3 – R2, R2 Æ R2 – R1, we get
(a) 1 (b) 0
1 x + 1 ( x + 1) ( x + 2) (c) 2 (d) 3
D1(x) = 0 1 2 ( x + 2) =2 Ans. (a)
0 1 2 ( x + 3) Solution: Denote the given determinant by D.
Example 79: Let a, b, c be such that b(a + c) π 0. Using C1 Æ C1 + C2 + C3 and 1 + w + w2 = 0, we get
a a +1 a -1 1 w w2
If D = - b b + 1 b - 1
D = z 1 z + w2 1
c c -1 c +1
1 1 z +w
a +1 b +1 c -1
Applying R2 Æ R2 – R1 and R3 Æ R3 – R1, we get
+ a -1 b -1 c +1 = 0
1 w w2
(-1)n + 2 a (-1)n + 1 b (-1)n c
2
D = z 0 z +w -w 1 - w2
Then the value of n is
(a) any odd integer 0 1-w z + w - w2
(b) any integer
= z[(z + w2 – w) (z + w – w2) – (1 – w) (1 – w2)]
(c) zero
(d) any even integer = z[z2 – (w2 – w)2 – (1 – w – w2 + 1)]
Ans. (a) = z[z2 – (w4 + w2 – 2w3) – 3] = z(z2) = z3
3
a +1 b +1 c -1 Thus, z = 0 fi z = 0
Solution: Let D2 = a -1 b -1 c +1 Thus, there is just one value of z, satisfying the given
n+2 n +1 n equation.
(- 1) a (- 1) b (- 1) c
4.24 Complete Mathematics—JEE Main

Assertion-Reason Type Questions

an an + 3 an + 6 Example 83: Suppose x > 0, y > 0, z > 0 and

Example 81: Let Dn = an + 1 an + 4 an + 7 x log 2 3 15 + log (a x )

an + 2 an + 5 an + 8 D(a, b, c) = y log 3 5 25 + log (b y )

Statement-1: If ak > 0 " k ≥ 1 and a1, a2, a3, . . . are in G.P. z log 5 7 35 + log (c z )
then Statement-1: D(8, 27, 125) = 0
Dn = 0 " n ≥ 1. Ê 1 1 1ˆ
Statement-2: D Á , , ˜ = 0
Statement-2: If a1, a2, a3 . . . are in A.P. then Ë 2 3 5¯
Dn = 0 " n ≥ 1. Ans. (b)
Ans. (b) Solution: Using log (bc) = c log b and applying C3 Æ
Solution: Let ak = ar k–1
" k ≥ 1, then C3 – 5C2 we get
1 1 1 x log 2 3 x log a
D n = a na n + 3 a n + 6 r r r =0 D(a, b, c) = y log 3 5 y log b
2 2 2 z log 5 7 z log c
r r r
Next, if ak = b + (k – 1)d, then using C2 Æ C3 – C1 and D(8, 27, 125) = D(23, 33, 53) = 0 as in this case C1 and C3 are
C2 Æ C2 – C1, we get proportional. Similarly, D(1/2, 1/3, 1/5) = D(2–1, 3–1, 5–1) = 0.

an 3d 3d Example 84: Let a π 0, p π 0 and


Dn = an + 1 3d 3d = 0 a b c
D= 0 p q
an + 2 3d 3d
p q 0
[ C2 and C3 are identical]
Statement-1: If the equations
2
0 cos x - sin x ax2 + bx + c = 0 and px + q = 0
Example 82: Let f(x) = sin x 0 cos x . have a common root, then D = 0.
cos x sin x 0 Statement-2: If D = 0, then the equations
Statement-1: If sin 2x = 1, then f(x) = 2/3 ax2 + bx + c = 0 and px + q = 0
Statement-2: f(x) = 0 if sin x = cos x have a common root.
Ans. (d) Ans. (b)
Solution: Multiplication of two determinants leads us Solution: If l is a common root of
1 -y y ax2 + bx + c = 0 and px + q = 0, then
f(x) = - y 1 y al2 + bl + c = 0, pl + q = 0 and pl2 + ql = 0
y y 1 Eliminating l, we obtained D = 0.
For statement-2, expanding D along C1 we obtain
where y = sin x cos x
– aq2 + p(bq – cp) = 0
Using C1 Æ C1 – C2, C2 Æ C2 + C3, we get 2
Ê qˆ Ê qˆ
1 0 y 1 0 y or aÁ - ˜ + bÁ - ˜ + c = 0
Ë p¯ Ë p¯
f(x) = (1 + y)2 -1 1 y = (1 + y)2 0 1 2 y
Thus, ax2 + bx + c = 0 and px + q = 0 have a common root.
0 1 1 0 1 1
Example 85: Statement-1:
= (1 + y)2 (1 – 2y)
When sin 2x = 1, y = 1/2 and f(x) = 0 sin p cos( x + p / 4) tan( x - p / 4)
When sin x = cos x, 1 – 2y = 1 – 2 sin2x = cos 2x = 0 D = sin( x - p / 4) - cos(p / 2) log( x / y) =0
\ f(x) = 0. cot(p / 4 + x ) log( y / x ) tan p
Determinants 4.25
Statement-2: A skew symmetric determinant of odd order 0 -1 3
equals 0. Solution: e = f (0) = 1 0 - 4
Ans. (a)
-3 4 0
Solution: For statement-2, see theory. =0
Now, using [skew symmetric determinant of odd order]
Ê pˆ Èp Ê p ˆ˘ \ Statement-2 is true.
cos Á x + ˜ = cos Í - Á - x˜ ˙
Ë 4¯ Î2 Ë 4 ¯˚ 1
To obtain a, replace x by , so that
x
Êp ˆ Ê pˆ
= sin Á - x˜ = - sin Á x - ˜ ;
Ë4 ¯ Ë 4¯ Ê 1ˆ
2
3 1 1
Ë x¯ + x x
-1
x
+3
Êp ˆ Èp Ê p ˆ˘
cot Á + x˜ = cot Í - Á - x˜ ˙ 1 2 1
Ë4 ¯ Î2 Ë 4 ¯˚ +1 - -4
x x x
p p
= tan ÊÁ - xˆ˜ = - tan ÊÁ x - ˆ˜ , 1 1 3
Ë4 ¯ Ë 4¯ -3 +4
x x x
and log (x/y) = – log (y/x) 4 3 2
we find D is a skew symmetric determinant of odd order. Ê 1ˆ Ê 1ˆ Ê 1ˆ Ê 1ˆ
aË ¯ + bË ¯ + cË ¯ + d Ë ¯ + e
x x x x
Example 86: Let f : Q Æ [–1, 1] by Take x2 common from C1, x from C2 and C3
f (x) = sin x 1 + 3x 1 - x 1 + 3x
Let x1, x2, x3 be three distinct rational numbers. Let a = f(x1), 1
b = f(x2), c = f(x3). = 4 x + x2 -2 1 - 4x
x
Statement-1: x - 3x2 1 + 4 x 3x
1 a a2 1
[a + bx + cx 2 + dx 3 + ex 4 ]
=
D= 1 b b 2 π0 x4
Cancel 1/x4 and put x = 0 to obtain
1 c c2 1 1 1
Statement-2: f is a one-to-one function.
0 -2 1 = a
Ans. (a)
0 1 0
Solution: Let x1, x2 Œ Q be such that fi a = –1
f(x1) = f(x2)
Example 88: Statement-1: The system of line equations
fi sin x1 = sin x2 x + (sin a) y + (cos a) z = 0
fi x1 = np + (–1)n x2, n Œ I x + (cos a) y + (sin a) z = 0
n+1
fi x1 + (–1) x2 = np, n Œ I. x – (sin a) y – (cos a) z = 0
But LHS is rational and RHS is irrational except when n = 0. has a non-trivial solution for only one value of a lying in
\ x1 = x2 the interval (0, p /2).
Thus, f is one-to-one. Statement-2: The equation in a
Also, D = (a – b) (b – c) (c – a) π 0 cos a sin a cos a
as x1, x2, x3 are distinct and hence a, b, c are distinct. D = sin a cos a sin a = 0
Example 87: Let cos a - sin a - cos a
has only one solution lying in the interval (0, p /2).
x2 + 3x x -1 x + 3 Ans. (b)
f (x) = x + 1 -2 x x - 4
Solution: Using C1 Æ C1 – C3 in D, we get
x-3 x + 4 3x
0 sin a cos a
= ax4 + bx3 + cx2 + dx + e
D= 0 cos a sin a
Statement-1: a = – 1
2 cos a - sin a - cos a
Statement-2: e = f (0)
= 2 cos a (sin2 a – cos2 a)
Ans. (a)
4.26 Complete Mathematics—JEE Main
= –2 cos a cos 2 a 2 0 0
D = 0 for a = p/4 Π(0, p /2). D1 = 1 cos a sin a = 0
This is the only value of a lying in (0, p /2) for which D = 0. 1 - sin a - cos a
The system of linear equations will have a non-trivial solu-
tion if and only if fi 2 (– cos2 a + sin2 a) = 0
fi –2 cos 2a = 0
1 sin a cos a
This is true for only one value of a Π(0, p /2)
D1 = 1 cos a sin a = 0 viz, a = p /4.
1 - sin a - cos a Thus, statement-1 is also true. However, statement-2 is not
Using R2 Æ R2 + R1, we get a correct reason for statement-1.

LEVEL 2

Straight Objective Type Questions

Example 89: Consider the system of equations in x, y, z: Example 90:


(sin 3q) x – y + z = 0 3 + 2 sin 4 x 2 cos4 x sin 2 2 x
(cos 2q) x + 4y + 3z = 0 Let D(x) = 2 sin x4
3 + 2 cos x 4
sin 2 2 x
2x + 7y + 7z = 0
2 sin 4 x 2 cos4 x 3 + sin 2 2 x
The values of q for which the system has a non-trivial solu-
p /2
tion are given by then Ú- p / 2 x D( x)dx equals
Ê 1 ˆ 1
(a) p Á n + (- 1)n ˜ (b) p ÊÁ n + (- 1)n ˆ˜ (a) p 2
(b) p (p – 1)
Ë 3 ¯ Ë 4 ¯
(c) 1 (d) 0
Ê 1 ˆ np Ans. (d)
(b) p Á n + (- 1)n ˜ (d)
Ë 6 ¯ 2 Solution: Using C1 Æ C1 + C2 + C3, we get
where n is an integer 1 2 cos4 x sin 2 2 x
Ans. (c)
D(x) = f(x) 1 3 + 2 cos4 x sin 2 2 x
Solution: For the system of equations to have a non- 4
trivial solution we must have 1 2 cos x 3 + sin 2 2 x
sin 3q - 1 1 where f(x) = 3 + 2 sin4x + 2 cos4x + sin2 2x
D = cos 2q 4 3 =0 = 3 + 2 (sin4x + cos4x + 2 sin2x cos2 x)
2 7 7 = 3 + 2 (cos2x + sin2x)2 = 5
Using R2 Æ R2 + 4R1 and R3 Æ R3 + 7R1 we get Applying C2 Æ C2 – (2 cos4 x) C1 and C3 Æ C3 –
sin 3q -1 1 (sin2 2x) C1, we get
1 0 0
D = cos 2 q + 4 sin 3q 0 7 =0
D(x) = 5 1 3 0 = 45
2 + 7 sin 3q 0 14
1 0 3
fi 2(cos 2q + 4 sin 3q) – (2 + 7 sin 3q) = 0
p /2 p /2
fi sin 3q + 2 cos 2q – 2 = 0 Thus, Ú- p / 2 x D( x)dx = 45Ú- p / 2 xdx = 0
fi 3 sin q – 4 sin3 q – 4 sin2 q = 0
Example 91: If f (x), g(x)and h(x) are three polynomials
fi sin q(2 sin q + 3) (2 sin q – 1) = 0 of degree 3 then
fi sin q = 0 or sin q = 1/2 [ |sin q | £ 1] f ¢( x ) g¢ (x) h¢ (x)
Ê (- 1)n ˆ f(x) = f ¢¢ ( x ) g ¢¢ ( x ) h ¢¢ ( x )
fi q = m p or q = n Á p + where m
Ë 6 ˜¯ f ¢¢¢ ( x ) g ¢¢¢ ( x ) h ¢¢¢ ( x )
and n are integers
Determinants 4.27
is a polynomial of degree Solution: The function D(x) is continuous on [0, p/2]
(a) 3 (b) 4 and differentiable on (0, p/2). Also D(0) = 0 and D(p/2) = 0.
(c) 5 (d) none of these Thus, by the Rolle’s theorem there exists at least one
Ans. (d) c e (0, p/2) such that D¢(c) = 0.
Solution: We have cos2 x cos x sin x - sin x
f ¢¢( x ) g ¢¢ ( x ) h ¢¢ ( x ) Example 94: Let D(x) = cos x sin x sin 2 x cos x
f¢(x) = f ¢¢ ( x ) g ¢¢ ( x ) h ¢¢ ( x ) + sin x - cos x 0
f ¢¢¢ ( x ) g ¢¢¢ ( x ) h ¢¢¢ ( x ) p /2

f ¢( x ) g¢ (x) h¢ (x) f ¢( x ) g ¢ ( x ) h ¢ ( x )
then Ú0 [D(x) + D¢(x)] dx equals

f ¢¢¢ ( x ) g ¢¢¢ ( x ) h ¢¢¢ ( x ) + f ¢¢ ( x ) g ¢¢ ( x ) h ¢¢ ( x ) (a) p/3 (b) p/2


(c) 2p (d) 3p/2
f ¢¢¢ ( x ) g ¢¢¢ ( x ) h ¢¢¢ ( x ) f iv ( x ) giv ( x ) hiv ( x ) Ans. (b)
f ¢( x ) g ¢ ( x ) h ¢ ( x ) Solution: Applying C1 Æ C1 – sin x C3 and C2 Æ C2 +
= 0 + 0 + ¢¢ ( x ) g ¢¢ ( x ) h ¢¢ ( x ) = 0
f cos x C3, we get
0 0 0 1 0 - sin x
D(x) = 0 1 cos x
since f, g, h are polynomials of degree 3, f iv(x) = giv(x) =
hiv(x) = 0 sin x - cos x 0
fi f(x) must be a constant. Applying R3 Æ R3 – sin x R1 + cosx R2, we get
Example 92: The determinant 1 0 - sin x
a a + b a + 2b
D(x) = 0 1 cos x =1
D = a + 2b a a+b
0 0 cos2 x + sin 2 x
a + b a + 2b a fi D¢(x) = 0
equals
(a) 9b2(a + b) (b) 9a2(a + b) p /2 p /2 p
(c) 9(a + b)3 (d) 9ab(a + b)
Thus, Ú0 [D(x) + D¢(x)] dx = Ú0 dx =
2
.
Ans. (a) Example 95: The determinant
Solution: Applying C1 Æ C1 + C2 + C3, we get 13 + 3 2 5 5
1 a + b a + 2b D= 15 + 26 5 10 equals
D = 3(a + b) 1 a a+b 3 + 65 15 5
1 a + 2b a
(a) 15 2 - 25 3 (b) 25 3 - 15 2
Applying R2 Æ R2 – R1 and R3 Æ R3 – R1,
(c) 3 5 (d) -15 2 + 7 3
1 a + b a + 2b Ans. (a)
D = 3(a + b) 0 -b -b
Solution: Taking 5 common from C2 and C3, we get
0 b -2b
13 + 3 2 1
-b -b
( )
2
= 3(a + b) = 9b2(a + b) D= 5 15 + 26 5 2
b -2b
Example 93: Let 3 + 65 3 5
sin x cos x sin 2 x + cos 2 x Applying C1 Æ C1 – 13 C3 – 3 C2, we get
D(x) = 0 1 1
- 3 2 1
1 0 -1
then D¢(x) vanishes at least once in
D = ( 5) 0 5 (
2 = 5 - 3 5- 6 )( )
(a) (0, p/2) (b) (p/2, p) 0 3 5
(c) (0, p/4) (d) (– p/2, 0)
Ans. (a) = 5 ( )
18 - 25 3 = 15 2 - 25 3
4.28 Complete Mathematics—JEE Main
Example 96: The values of l for which the system of Example 98: If a + b + c π 0, the system of equations
equations (b + c) (y + z) – ax = b – c
x + y – 3= 0 (c + a) (z + x) – by = c – a
(1 + l)x + (2 + l)y – 8 = 0 (a + b) (x + y) – cz = a – b
x – (1 + l)y + (2 + l) = 0 has
(a) a unique solution
has a non-trivial solution, are
(b) no solution
(a) – 5/3, 1 (b) 2/3, – 3
(c) infinite number of solutions
(c) – 1/3, – 3 (d) 0
(d) finitely many solutions
Ans. (a)
Ans. (a)
Solution: The given system of equations has a non
Solution: We can write the above system of equations
trivial solution if
(a + b + c) (y + z) – a(x + y + z) = b – c
1 1 -3
(a + b + c) (z + x) – b(x + y + z) = c – a
D = 1+ l 2+l -8 = 0
(a + b + c) (x + y) – c(x + y + z) = a – b
1 - (1 + l ) 2 + l
Adding the above equations, we obtain
Applying C2 Æ C2 – C1 and C3 Æ C3 + 3C1, we get 2(a + b + c) (x + y + z) – (a + b + c) (x + y + z) = 0
1 0 0
fi (a + b + c) (x + y + z) = 0
D = 1+ l 1 -5 + 3l = 0 fi x+y+z =0 [ a + b + c π 0]
1 -2 - l 5+l fi y+z =–x
fi (5 + l) + (2 + l) (3l – 5) = 0 c-b
\ (b + c) (– x) – ax = b – c fi x =
fi 5 + l + 6l – 10 + 3l2 – 5l = 0 a+b+c
fi 3l2 + 2l – 5 = 0 fi (3l + 5) (l – 1) = 0
a-c b-a
fi l = – 5/3 or l = 1. Similarly, y = ,z= .
a+b+c a+b+c
Example 97: The values of m for which the system
Example 99: Let a, b, c be positive real numbers. The
of equations 3x + my = m and 2x – 5y = 20 has a solution
following system of equations in x, y and z.
satisfying the conditions x > 0, y > 0 are given by the set
(a) {m : m < – 13/2} x2 y2 z2 x2 y2 z2 - x2 y2 z2
+ - = 1, - + = 1, + + =1
(b) {m : m > 17/2} a2 b2 c2 a2 b2 c2 a2 b2 c2
(c) {m : m < – 13/2 or m > 17/2} has
(d) none of these (a) no solution
Ans. (d) (b) unique solution
(c) infinitely many solutions
3 m (d) finitely many solutions
Solution: Here D = = – 15 – 2m
2 -5 Ans. (d)
m m Solution: Adding all the equations, we obtain
Dx = = – 25 m
20 -5 x2 y2 z2
2
+ 2
+ =3
3 m a b c2
and Dy = = 60 – 2m
2 20 2z2
Subtracting first equation from it we get = 2 fi z2 = c2
If D = 0, then m = – 15/2. But for this value of m, Dx π 0 c2
and Dy π 0. Thus, in this case, the system of equations is not fi z = ± c. Similarly, x = ± a, y = ± b. Thus, the given system
consistent. of equations has eight solutions.
25m 2m - 60 Example 100: If the system of equations
If D π 0, then x = and y = , by the Cramer’s
2m + 15 2m + 15 x – ky – z = 0, kx – y – z = 0, x + y – z = 0
rule. has a non-zero solution, then the possible values of k are
Now, x > 0, y > 0 ¤ 25m > 0, 2m – 60 > 0, 2m + 15 > 0 (a) – 1, 2 (b) 1, 2
or 25m < 0, 2m – 60 < 0, 2m + 15 < 0 (c) 0, 1 (d) – 1, 1
fi m > 30 or m < – 15/2. Ans. (d)
Determinants 4.29
Solution: As the given system has a non-zero solution, p a- p a- p
1 - k -1 1+ k - k - 1 -1 D= b q-b 0 =0
0 = k -1 -1 = 1 + k -2 -1 c 0 r-c
1 1 -1 0 0 -1 Expanding along C3, we get
b q-b p a- p
[using C1 Æ C1 – C2, C2 Æ C2 + C3] (a – p) + (r – c) =0
c 0 b q-b
fi 0 = (– 1) [(1 + k) (– 2) – (1 + k) (– k – 1)]
fi (a – p) (– c) (q – b) + (r – c) {p(q – b) – b(a – p)} = 0
fi 0 = (1 + k) (– 2 + k + 1) fi k = – 1, 1
fi (p – a) (q – b) c + p (r – c) (q – b) + b (r – c) (p – a) = 0
Example 101: If the system of equations Dividing by (p – a) (q – b) (r – c) we get
lx1 + x2 + x3 = 1, x1 + lx2 + x3 = 1, x1 + x2 + lx3 = 1 c p b
is inconsistent, then l equals + + =0
r-c p-a q-b
(a) 5 (b) – 2/3
(c) – 3 (d) – 2 p q r q-b r-c
fi + + = + =2
Ans. (d) p-a q-b r-c q-b r-c
l 1 1 l +2 1 1 Example 103: Let l and a be real. Relation between l
Solution: Let D = 1 l 1 = l + 2 l 1 and a for which the system of equations
1 1 l l +2 1 l lx + (sin a)y + (cos a)z = 0
x + (cos a)y + (sin a)z = 0
[C1 Æ C1 + C2 + C3] – x + (sin a)y – (cos a)z = 0
1 1 1 1 0 0 has a non-trivial solution is
= (l + 2) 1 l 1 = (l + 2) 1 l - 1 0 (a) l = sin 2a + cos 2a
(b) l = |sin 2a|
1 1 l 1 0 l -1 (c) l = |sin 2a – cos 2a|
[using C2 Æ C2 – C1 and C3 Æ C3 – C1] (d) l = cos 2a
= (l + 2) (l – 1)2 Ans. (a)
If D = 0, then l = – 2 or l = 1. Solution: The system of equations will have a non-
But when l = 1, the system of equation becomes x1 + x2 + x3 trivial solution if and only if
= 1 which has infinite number of solutions. When l = – 2, by l sin a cos a
adding three equations, we obtain 0 = 3 and thus, the system 0 = 1 cos a sin a
of equations is inconsistent. -1 sin a - cos a
Example 102: If p π a, q π b, r π c and the system of 0 (l + 1)sin a (1 - l )cos a
equations = 0 cos a + sin a sin a - cos a
px + ay + az = 0
-1 sin a - cos a
bx + qy + bz = 0
[using R1 Æ R1 + lR3, R2 Æ R2 + R3]
cx + cy + rz = 0
= (l + 1) sin a (sin a – cos a)
has a non-trivial solution, then the value of
+ (l – 1) cos a(cos a + sin a)
p q r
+ + is = l(sin2a + cos2a) + [sin2a – cos2 a
p-a q-b r-c
– 2 sin a cos a]
(a) – 1 (b) 0 Thus, l = sin 2a + cos 2a
(c) 1 (d) 2
Example 104: If fr(x), gr(x), hr(x), r = 1, 2, 3 are
Ans. (d)
polynomials in x such that fr(a) = gr(a) = hr(a), r = 1, 2, 3
Solution: As the given system of equations has a non- f1 ( x ) f2 ( x ) f3 ( x )
trivial solution
and F (x) = g1 ( x ) g2 ( x ) g3 ( x )
p a a
h1 ( x ) h2 ( x ) h3 ( x )
D= b q b =0 then F ¢(a) is
c c r (a) – 1 (b) a
Applying C3 Æ C3 – C2 and C2 Æ C2 – C1, we get (c) 0 (d) none of these
Ans. (c)
4.30 Complete Mathematics—JEE Main
Solution: Differentiating w.r.t. x we get F ¢(x) 1 1
(c) x = (y – w), z = (y + w), y ≥ 0
f1¢( x ) f2¢ ( x ) f3¢ ( x ) f1 ( x ) f2 ( x ) f3 ( x ) 7 3
= g1 ( x ) g2 ( x ) g3 ( x ) + g1¢ ( x ) g2¢ ( x ) g3¢ ( x ) (d) x = 1, y = 0, z = 0, w = 0.
h1 ( x ) h2 ( x ) h3 ( x ) h1 ( x ) h2 ( x ) h3 ( x ) Ans. (d)
Solution: We have
f1 ( x ) f2 ( x ) f3 ( x ) 1 – (x + 2y) = z ≥ 0
+ g1 ( x ) g2 ( x ) g3 ( x ) and 2x – 3y – 2 = – w £ 0.
h1¢ ( x ) h2¢ ( x ) h3¢ ( x ) fi x + 2y £ 1 and 2x – 3y £ 2, x ≥ 0, y ≥ 0
Since fr(a) = gr(a) = hr(a) for r = 1, 2, 3, we get each of the The only point where the two shaded region intersect is
three determinants on the right side becomes zero when x (1, 0). Thus, x = 1, y = 0 for these values z = 0, w = 0.
is replaced by a. (In each case two rows become identical.)
y
Thus, F ¢(a) = 0. 1
Example 105: The number of real values of a for which 1
x+
the system of equations 2 2y =2
=1 – 3y
x + ay – z = 0, 2x – y + az = 0, ax + y + 2z = 0 2x
has a non-trivial solution, is O 1 x
(a) 3 (b) 1
(c) 0 (d) infinite –2
Ans. (a) 3

Solution: Since the given system of equations has a


non-trivial solution, Fig. 4.1
1 a -1
D = 2 -1 a = 0 Example 107: The number of values of k for which the
system of equations
a 1 2
(k + 1)x + 8y = 4k
Using C1 Æ C1 + C3, C2 Æ C2 + aC3, we get
kx + (k + 3)y = 3k – 1
0 0 -1 has infinitely many solutions is
D = 2 + a -1 + a 2 a =0 (a) 0 (b) 1
2 + a 1 + 2a 2 (c) 2 (d) infinite
Ans. (b)
2 + a -1 + a 2 k +1 8
fi (-1) =0 Solution: Let D =
2 + a 1 + 2a k k +3
fi (2 + a) (1 + 2a + 1 – a2) = 0 = k2 – 4k + 3 = (k – 1) (k – 3)
fi a = – 2, 1 ± 3 . If D π 0, the system of equations has a unique solution.
Thus, there are three real values of a for which the system For the system of equations to have an infinite number of
of equations has a non-trivial solution. solutions
D = 0 fi k = 3, 1
Example 106: The solution set of
For k = 3, the system equations becomes
x + 2y + z = 1
4x + 8y = 12 and 3x + 6y = 8
2x – 3y + w = 2
8
subject x ≥ 0, y ≥ 0, z ≥ 0, w ≥ 0 is fi x + 2y = 3, x + 2y =
3
1 Thus, in this case the system of equations has no solution.
(a) x = (y – w), y ≥ w ≥ 0, z ≥ 0
7 For k = 1, the system of equations becomes
1 1 2x + 8y = 4 and x + 4y = 2
(b) x = (y + w), z= (y – w), y ≥ w ≥ 0
8 3 fi x + 4y = 2, x + 4y = 2
This system has infinite number of solutions.
Determinants 4.31
Example 108: Suppose a, b, c ΠR and let (k Р1)x Р2y = 0.
0 a-x b-x If k = – 1, we can choose y in infinite number of ways,
f(x) = - a - x 0 c-x corresponding to which we can choose x and z in infinite
number of ways.
-b-x -c-x 0
2 If k = 1, then y = 0 and x and hence z can be chosen in infinite
Then coefficient of x in f(x) is number of ways.
(a) – (a + b + c) (b) a + b + c
Example 110: Let a2, a3 ΠR be such that | a2 Рa3 | = 6.
(c) 0 (d) ab + bc + ca Let
Ans. (c)
1 a3 a2
Solution: Expanding along C1 f(x) = 1 a3 2a2 - x , x ΠR
a-x b-x a-x b-x 1 2a3 - x a2
f(x) = (a + x) - (b + x )
-c-x 0 0 c-x The maximum value of f (x) is
= (a + x) (b – x) (c + x) – (b + x) (a – x) (c – x) (a) 6 (b) 9
= – [(x + a) (x – b) (x + c) + (x – a) (x + b) (x – c)] (c) 12 (d) 36
Ans. (b)
3 2
= – [2x + x (a – b + c – a + b – c) + ...] Solution: Using R2 Æ R2 – R1, R3 Æ R3 – R1, we get
= – 2x3 + 0x2 + ...
\ coefficient of x2 in f(x) is 0. 1 a3 a2
Example 109: If the system of equations f(x) = 0 0 a2 - x
x – ky – z = 0, kx – y – z = 0, x + y – z = 0 0 a3 - x 0
has a non-zero solution, then possible values of k are = – (a2 – x) (a3 – x) = – [x2 – (a2 + a3)x + a2a3]
(a) – 1, 2 (b) 1, 2
(c) 0, 1 (d) – 1, 1 1 a + a3 ˆ 2
= (a2 – a3)2 – Ê x - 2 £9
Ans. (d) 4 Ë 2 ¯
Solution: Subtracting the last equation from the first f(x) attains maximum value 9 when
two equations, we get
1
– (k + 1)y = 0 x= (a2 + a3).
2

EXERCISES
Concept-based
Straight Objective Type Questions

1. Suppose A = (aij)n × n, where aij Œ R. (c) p3 – 2 abc (d) 0


–1
If det (adj(A)A ) = 3, then det (adj(A)) equals: 3. Suppose x π 1 and

(a) 3 (b) 3 1 x x2

(c) 3 3 (d) 9 D = x2 1 x
2
x x 1
2. If a, b, c are in A.P. and p is a real number, and
then D = 0
p+c p+2 p+a
(a) for exactly two distinct complex numbers
D = p+b p+5 p+b (b) for exactly four distinct complex numbers
p+a p+8 p+c (c) for exactly two distinct real numbers
then D equals:
(d) none of these
(a) – p3 (b) p3
4.32 Complete Mathematics—JEE Main

10! 11! 12! 10. Suppose a, b, c ΠR. Let


D
4. Let D = 11! 12! 13! , then – 260 equals: (a + 2016)2 (b + 2016)2 (a + 2016)2
(10!)3
12! 13! 14! D = (a - 2016)2 (b - 2016)2 (c - 2016)2
(a) 1 (b) 2
a2 b2 c2
(c) 3 (d) 4 3
If D = k (2016) (a – b) (b – c)(c – a), then k is
5. Let equal to:
x +1 2 3 (a) – 1 (b) – 4
P(x) = 1 x+2 3 , (c) 4 (d) 1
1 2 x+3 x -6 -1
the product of zeros of P(x) is 11. Let P(x) = 2 -3 x x + 3 ,
(a) 0 (b) 6 -3 2 x x + 2
(c) – 6 (d) 12 sum of the zeros of P(x) is
6. Let A(x1, y1), B(x2, y2) and C(x3, y3) be vertices of (a) – 6 (b) – 7
an equilateral triangle whose side is 4 units. Let (c) 13/5 (d) –12/5
x1 y1 1 12. Let n be an integer and x, y, z > 1. Suppose
D = x2 y2 1 , x n +1 x n+2 x n +3
n +1 n+2
x3 y3 1 D= y y y n +3
2
then D is equal to z n +1 z n + 2 z n +3
(a) 64 (b) 128 If D = (x – y) (y – z)(z – x) x2 y2 z2, then n is equal
(c) 192 (d) 256 to:
(a) –1 (b) 0
7. Suppose a, b, c are in A.P. If p, q, r are also in (c) 1 (d) 2
A.P., then value of
13. Suppose a, b, c are distinct real numbers. Let
x2 + a x+p c
0 x3 - a x4 - b
D = x2 + b x+q b ,
3
P(x) = x + a 0 x5 + c
x2 + c x+r a
is dependent on x4 + b x5 - c 0
(a) x (b) a, b, c A value of x satisfying P(x) = 0 is
(c) p, q, r (d) none of these (a) –(a + b + c) (b) a + b + c
(c) a + b – c (d) 0
8. Suppose a, b are two non-zero numbers. Let
14. Suppose a, b, c Œ R and abc π 0. Let
2 a+b a 2 + b2 1+ a 1 1
D= a+b a 2 + b2 a 3 + b3 , D = 1 + b 1 + 2b 1
a 2 + b 2 a 3 + b3 a 4 + b 4 1 + c 1 + c 1 + 3c
then D is equal to: 1 1 1
(a) 0 (b) ab If D = 0, then + + is equal to:
a b c
(c) a6 + b6 (d) a3b5 + a5b3 (a) 0 (b) –1
9. Suppose a, b, c > 1. Let (c) –2 (d) –3
log a log b log c 15. Suppose n and m are natural numbers such that
D = log (2007a) log (2007 b) log (2007 c) xm x m +2 x2m
log (2017 a) log (20117 b) log (2017 c) D= 1 x n
2n
then D is equal to:
x m + 5 x n +6 x 2 m + 5
(a) 0 (b) log (4024 abc)
Then a possible relationship between n and m is
Ê 2017 ˆ (a) n = m + 2 (b) n = m + 1
(c) log Ë (d) none of these
2007 ¯ (c) n = m (d) n = m – 1
Determinants 4.33

LEVEL 1

Straight Objective Type Questions

16. Suppose a, b, c Œ R and a + b + c π 0. Let 21. Suppose point (x, y, z) in space satisfies the equation
b+c c+a a+b
x2 + 1 xy xz
D= c+a a+b b+c 2
yx y +1 yz =5
a+b b+c c+a
If D = 0, then zx zy z2 + 1
(a) a = b = c (b) a3 + b3 – c3 = 0 Then (x, y, z) lies on a
(c) a = b + c (d) a = b = c = 0 (a) plane (b) straight line
17. Distance of line (c) sphere (d) none of these
x +1 x x 22. Suppose A, B, and C are angles of a triangle. Let
y= x x+2 x -1 cos C cos B
x x x+3 D = cos C -1 cos A
from the origin is cos B cos A -1
6 7
(a) (b) then D equals:
11 13
(a) 0 (b) –1
6 7 (c) –2 (d) –3
(c) (d)
122 122 23. Let
18. Suppose a, b, c ΠR. Let
a x x
3a -a + b -a + c
D= x b x
D = -b + a 3b -b + c
x x c
-c + a -c + b 3c and f(x) = (x – a) (x – b) (x – c)
Then D equals
Determinant D is equal to:
(a) 3 (a + b + c) (bc + ca + ab)
(b) a+b+c (a) f(x) – x3 (b) f ¢(x)
(c) 3 (a + b + c) (a2 + b2 + c2) (c) xf ¢(x) – f(x) (d) f ¢(x) – x f ≤(x)
(d) 0 24. Straight line
19. Suppose a, b, c are in A.P. Let 2-x-y 4 4
p + 2 n +1 + 2a q 2 + 2 n + 2 + 3b r 2 + a
2 2x x-y-2 2x =0
D= 2n + a 2 n +1 + b 2b 2y 2y y-2- x
2
p +2 +a n
q + 2 + 2b r - c 2 n +1 2 passes through the fixed point
Then D equals: (a) (–2, –2) (b) (–2, 0)
(a) –1 (b) 0 (c) (0, –2) (d) ( –1, –1)
(c) p2 q2 r2 Р3abc (d) p2 q2 r2 Р4 (a + b + c) 25. Suppose a ΠR. Let
20. Suppose a, b, c, d, e and f are in G.P. with common x+a x x
ratio > 1. Let p, q, r be three real numbers. Let
f (x) = x x+a x
a2 d2 p x x x+a
2 2
D= b e q Then f (2x) – f (x) is equal to
c2 f 2 r (a) 3xa2 (b) 3x2a
2
(c) xa (d) a2x
Then D depends on
(a) a, b, c (b) d, e, f 26. If a, b, g are the roots of x3 + ax2 + b = 0, then
(c) p, q, r (d) none of these the determinant
4.34 Complete Mathematics—JEE Main

a b g 32. If x π 0, the determinant


D = b g a equals a0 a1 a2
g a b D = -x x 0
0 -x x
(a) – a3 (b) a3 – 3b
(c) a2 – 3b (d) a3 vanishes if
(a) a0 + a1 + a2 = 0 (b) a0 + a1 = 2a2
27. If a, b, g are the roots of x3 + bx + c = 0, then the
(c) a0 + a2 = 2a1 (d) none of these
determinant
33. If x ΠR, the determinant
a b g
1 cos x 0
D = b g a equals
D = -1 1 - cos x sin x + cos x equals
g a b
0 -1 1 - 2 sin ( x + p / 4 )
(a) – b3 (b) b3 – 3c
(c) b2 – 3c (d) 0 (a) 0 (b) –1
28. If a, b, c are distinct and different from zero and (c) 1 (d) none of these
bc ca ab x a b
D = ca ab bc = 0, then 34. The factors of D = a x b are
ab bc ca a b x
–1 –1 –1 (a) x – a, x – b and x + a + b
(a) a +b +c =0
(b) x – a, x – b and x – a – b
(b) a–1 + b–1 – c–1 = 0 (c) x + a, x + b and x – a – b
(c) a–1 – b–1 + c–1 = 0 (d) none of these
(d) a–1 – b–1 – c–1 = 0 35. If q e R, maximum value of
1 1 1
l a l 2 + a2 1
D = 1 1 + sin q 1 is
29. The determinant D = l b l 2 + b2 1 equals
1 1 1 + cos q
lc l 2 + c2 1
(a) 1/2 (b) 3 /2
(a) l(a – b) (b – c) (c – a)
(b) l(a2 + b2 + c2) (c) 2 (d) 3 2 /4
(c) l(a + b + c) x+a b c
(d) l2(a – b) (b – c) (c – a) 36. If D = a x+b c = 0, then x equals
30. If a, b, g are real numbers, then the determinant a b x+c
(a) a + b + c (b) – (a + b + c)
sin 2 a cos 2a cos2 a
(c) 0, a + b + c (d) 0, – (a + b + c)
2
D = sin b cos 2b cos2 b equals 37. The determinant
sin 2 g cos 2g cos2 g sec 2 q tan 2 q 1
2 2
(a) 0 tan q sec q -1 equals
(b) – 1 12 10 2
(c) sin2 a + sin2 b + sin2 g
(d) none of these (a) 2 sin2q
31. If bc + ca + ab = 18, and (b) 12 sec2 q – 10 tan2 q
1 a2 a3 1 1 1 (c) 12 sec2 q – 10 tan2 q + 5
(d) 0
1 b 2
b3 = l a b c - a 2b 0
1 c2 c3 a2 b2 c2 38. If D = 0 - a 2b = 0, then
the value of l is 2b 0 - a
(a) – 1 (b) 0 (a) 1/b is a cube root of unity
(c) 9 (d) 18 (b) a is one of the cube roots of unity
Determinants 4.35
(c) b is one of the cube roots of 8 6i -3i w
(d) a/b is a cube root of 8 45. If D = 4 3i - w = x + iy, then
39. The determinant
20 3 iw
1 1+ i i
D = 1+ i i 1 equals (a) x = 3, y = 1 (b) x = 1, y = 3
(c) x = 0, y = 3 (d) x = 0, y = 0
i 1 1+ i
p p
(a) 7 + 4i (b) – 7 + 4i 46. If – £x£ , the number of distinct real roots
4 4
(c) – 7 – 4i (d) 2(i – 1) of D = 0
40. If a, b, c are non-zero real numbers, then
sin x cos x cos x
1 1
1 ab + D = cos x sin x cos x is
a b
cos x cos x sin x
1 1
D = 1 bc + equals (a) 0 (b) 2
b c
(c) 1 (d) 3
1 1
1 ca + 47. If w π 1 is a complex cube root of unity, and
c a
1 i -w
(a) 0 (b) bc + ca + ab
x + iy = -i 1 w2 then
(c) a–1 + b–1 + c–1 (d) abc – 1
w -w 2 1
log a ( abc ) log a b log a c
(a) x = – 1, y = 0 (b) x = 1, y = – 1
41. If a, b, c > 1, then D = logb ( abc ) 1 logb c (c) x = 1, y = 1 (d) none of these
logc ( abc ) logc b 1
equals 48. If eix = cosx + isinx and
1 ep i / 4 ep i / 3
(a) 0
(b) loga b + logb c + logc a x + iy = e-p i / 4 1 e2p i / 3 , then
(c) logabc (a + b + c) e -p i / 3 e-2p i / 3 e-2p i / 3
(d) none of these
(a) x = – 1, y = 2
a + bx c + dx p + qx a c p (b) x = 1, y = - 2
42. If D = ax + b cx + d px + q = l b d q
(c) x = - 2 , y = 2
u v w u v w
(d) none of these
then l equals 49. If a, b, c e R, the number of real roots of the
(a) 0 (b) 1 equation
(c) x (d) 1 – x2 x c -b
1 sin q 1 -c x a = 0, is
43. Let D = - sin q 1 sin q , 0 £ q £ 2p.The b -a x
-1 - sin q 1 (a) 0 (b) 1
(c) 2 (d) 3
(a) D = 0 (b) D Œ (2, •)
(c) D Π(2, 4) (d) D Π[2, 4] 1 x x2
50. If D = x x2 1 = – 7 and
b2 - ab b - c bc - ac
x2 1 x
44. The determinant D = ab - a 2 a - b b2 - ab
bc - ac c - a ab - a 2 x3 - 1 0 x - x4
equals
D1 = 0 x - x4 x 3 - 1 , then
(a) (b – c) (c – a) (a – b) 4 3
x-x x -1 0
(b) abc (b – c) (c – a) (a – b)
(c) (a + b + c) (b – c) (c – a) (a – b) (a) D = 7 (b) D = 343
(c) D = – 49 (d) D = 49
(d) 0
4.36 Complete Mathematics—JEE Main

sin a cos a sin a + cos b 57. For a fixed positive integer n, if


51. If D = sin b cos a sin b + cos b then D equals n! (n + 1)! (n + 2)!
sin g cos a sin g + cos b D = ( n + 1)! ( n + 2 )! ( n + 3)!
(a) sin a sin b sin g (n + 2)! (n + 3)! (n + 4)!
(b) cos a sec b tan g D
(c) sin a sin b sin g + cos a cos b cos g then is equal to
n! ( n + 1)! ( n + 2 )!
(d) 0
52. Suppose a, b, c ΠR and a, b, c > 0, (a) Р4 (b) Р2
(c) 2 (d) 4
log a log b log c
x+2 x+7 a
Let D = log (7 a) log (49 b) log (343 c) 58. If a, b, c are in A.P., and D = x + 5 x + 11 b
log (3 a) log (9 b) log (27 c) x + 8 x + 15 c
then D is equals to then D equals
(a) 0 (b) –1 (a) 0 (b) 1
(c) 1 (d) 30 (c) – (a + b + c) (d) a + b + c.
53. The values of q lying between q = 0 and q = p/2 1+ y 1- y 1- y
and satisfying the equation
59. If D = 1 - y 1 + y 1 - y = 0, then value of y are
1 + cos2 q sin 2 q 4 sin 4q 1- y 1- y 1+ y
2
D= cos q 1 + sin 2 q 4 sin 4q =0 (a) 0, 3 (b) 2, – 1
2 2
cos q sin q 1 + 4 sin 4q (c) – 1, 3 (d) 0, 2
are given by 60. The determinant
(a) p/24, 5p/24 (b) 7p/24, 11p/24 al + a ¢l ¢ am + a ¢m ¢ an + a ¢n ¢
(c) 5p/24, 7p/24 (d) 11p/24, p/24 D = bl + b ¢l ¢ bm + b ¢m ¢ bn + b ¢n ¢
54. The number of distinct roots of the equation cl + c ¢l ¢ cm + c ¢m ¢ cn + c ¢n ¢
x2 - 1 x2 + 2 x + 1 2 x2 + 3x + 1 is equal to
P(x) = 2 x 2 + x - 1 2 x 2 + 5 x - 3 4 x2 + 4 x - 3 = 0 (a) (abc + a¢b¢c¢) (lmn + l¢m¢n¢)
(b) abc lmn + a¢b¢c¢ l¢m¢n¢
6 x - x - 2 6 x - 7 x + 2 12 x 2 - 5 x - 2
2 2
(c) (a2 + b2 + c2) (l2 + m2 + n2)
is + (a¢2 + b¢2 + c¢2) (l¢2 + m¢2 + n¢2)
(a) 6 (b) 5 (d) 0
(c) 3 (d) 4 61. If a = i, b = w, c = w2 where w is complex cube
2rp 2rp root of unity, then
55. If ar = cos + i sin , then value of the
9 9 a a+b a+b+c
determinant D = 3a 4 a + 3b 5a + 4b + 3c is equal to
1 a8 a7 6 a 9a + 6b 11a + 9b + 6c
D = a3 a2 a1 is
(a) – w (b) – w2
a6 a5 a4 (c) i (d) – i
(a) – 1 (b) 1 1 1 1
(c) 0 (d) – 2
56. If a π p, b π q, c π r and the system of equations 62. If D = m
C1 m +3
C1 m +6
C1 = 2a 3b 5g, then
px + by + cz = 0 C2 m + 3 C2 m + 6 C2
m

ax + qy + cz = 0 a + b + g is equal
ax + by + rz = 0 (a) 3 (b) 5
has a non-zero solution, then value of (c) 7 (d) none of these
p+a q+b r+c 63. Suppose a, b, c, x, y ΠR. Let
+ + is
p-a q-b r-c 1 2 + ax 3 + ay
(a) 2 (b) – 2 D = 1 2 + bx 3 + by
(c) 1 (d) 1 1 2 + cx 3 + cy
Determinants 4.37
Then D is independent of 70. The values of l for which the system of equations
(a) a, b, c (b) x, y (l + 5)x + (l – 4)y + z = 0
(c) a, b, c, y (d) a, b, c, x, y (l – 2)x + (l + 3)y + z = 0
64. If A, B and C are the angles of a triangle, then the lx + ly + z = 0
determinant has a non-trivial solution is (are)
0 cos C cos B (a) – 1, 2 (b) 0, – 1
D = cos C -1 cos A is equal to (c) 0 (d) none of these
cos B cos A -1 71. Given the system of equations
(b + c) (y + z) – ax = b – c
(a) sin2A (b) sin2B (c + a) (z + x) – by = c – a
(c) sin2C (d) 0 (a + b) (x + y) – cz = a – b
cos x x 1 (where a + b + c π 0); then x : y : z is given by
f (x) (a) b – c : c – a : a – b (b) b + c : c + a : a + b
65. Let f(x) = 2 sin x x2 2 x , then lim 2 is
xÆ0 x a b c
tan x x 1 (c) a : b : c (d) : :
given by b c a
(a) 0 (b) – 1 72. If a, b, c e R and a + b + c π 0 and the system of
(c) 2 (d) 3 equations
66. If w is a complex cube root of unity, then value of ax + by + cz = 0
bx + cy + az = 0
a1 + b1w a1w 2 + b1 c1 + b1w
cx + ay + bz = 0
D = a2 + b2w a2w 2 + b2 c2 + b2w has a non-zero solution, then a : b : c is given by
a3 + b3w a3w 2 + b3 c3 + b3w (a) 1 : a : b
is where a, b are roots of ax2 + bx + c = 0
(a) 0 (b) – 1 (b) 1 : r : r2 where r is some positive real number
(c) 2 (d) none of these (c) 1 : k : 2k where k is some positive real number
67. If pqr π 0 and the system of equations (d) none of these
(p + a)x + by + cz = 0 73. If p is a constant and
ax + (q + b)y + cz = 0 x3 x3 3x 2
ax + by + (r + c)z = 0 f(x) = 1 -6 4 , then f ¢≤(x) is
a b c
has a non-trivial solution, then value of + + p p2 p3
p q r 3
is (a) proportional to x (b) proportional to x2
(a) – 1 (b) 0 (c) proportional to x (d) a constant
(c) 1 (d) 2
a1 b1 c1
68. The system of equations
ax + by + (aa + b)z = 0 74. If D = a2 b2 c2 and
bx + cy + (ba + c)z = 0 a3 b3 c3
(aa + b)x + (ba + c)y = 0 a1 + pb1 b1 + qc1 c1 + ra1
has a non-zero solutions if a, b, c are in D = a2 + pb2 b2 + qc2 c2 + ra2 then
(a) A.P. (b) G.P. a3 + pb3 b3 + qc3 c3 + ra3
(c) H.P. (d) A.G.P. (a) D = D (b) D = D (1 – pqr)
69. If the system of equations (c) D = D(1 + pqr) (d) D = D(1 + p + q + r)
ax + ay – z = 0 75. Number of real values of l for which the system
bx – y + bz = 0 of equations
– x + cy + cz = 0 (l + 3)x + (l + 2)y + z = 0
(where a, b, c π – 1) has a non-trivial solution, then 3x + (l + 3)y + z = 0
1 1 1 2x + 3y + z = 0
value of + + is
1+ a 1+ b 1+ c has a non-trivial solutions is
(a) 2 (b) – 1 (a) 0 (b) 1
(c) – 2 (d) 0 (c) 2 (d) infinite
4.38 Complete Mathematics—JEE Main

Assertion-Reason Type Questions

76. Statement-1: If ax + by + cz = 0
Ê pˆ Ê pˆ bx + cy + az = 0
sin x - cos x sin Á x - ˜ cos Á x - ˜
Ë 3¯ Ë 3¯ cx + ay + bz = 0
Êp ˆ Ê pˆ Ê pˆ has infinite number of solutions.
D(x) = sin Á - x˜ tan Á x - ˜ sec Á x - ˜
Ë3 ¯ Ë 4¯ Ë 4¯ Statement-2: If |A| = 0, the system of equations
AX = B has infinite number of solutions.
Ê 2p ˆ Ê 2p ˆ Ê pˆ
cos Á + x˜ sec Á + x˜ cot Á x + ˜ 81. Suppose a, b, c Œ R. Consider the system of linear
Ë 3 ¯ Ë 3 ¯ Ë 4¯
equations:
Êpˆ ax + by + cz = 0
then D Á ˜ = 0
Ë 4¯ bx + cy + az = 0
Statement-2: If A is a skew symmetric matrix of cx + ay + bz = 0
odd order, then |A| = 0.
Statement-1: If a + b + c π 0 and a2 + b2 + c2
77. Statement-1: Let p < 0 and a1, a2, . . . a9 be the = bc + ca + ab, then the system of equations has
nine roots of x9 = p, then infinite number of solutions.
Statement-2: If a + b + c = 0 and a2 + b2 + c2
a1 a2 a3
π bc + ca + ab, then the system of equations has
D = a4 a5 a6 = 0 unique solutions.
a7 a8 a 9 82. Suppose a1, a2, a3, b1, b2, b3 ΠR.
Statement-2: If two rows of a determinant are Let
identical, then determinant equals zero. a1 b1 1
78. Statement-1: D1 = a2 b2 1
6 2i 3 + 6i a3 b3 1
w = 12 3 + 8i 3 2 + 6i 2 a1b1 a1b2 + a2 b1 a1b3 + a3 b1
18 2 + 12i 27 + 2i D2 = a1b2 + a2 b1 2 a2 b2 a2 b3 + a3 b2
is a purely imaginary number. a1b3 + a3 b1 a2 b3 + a3 b2 2a3b3
Statement-2: |z| = | z | for each complex number z.
Statement-1: D2 π 0 for some values of a1, a2, a3,
79. Statement-1: If ai, bi ΠN, for i = 1, 2, 3 and b1, b2, b3 ΠR.
(1 + x )a1 b1 (1 + x )a1 b2 (1 + x )a1 b3 Statement-2: If (a1, b1), (a2, b2), (a3, b3) are non-
a2 b1 a2 b2 a2 b3
collinear than D1 π 0.
D(x) = (1 + x ) (1 + x ) (1 + x ) ,
83. Suppose a1, b1, c1, d1 a2, b2, c2, d2 are eight integers.
a3 b1 a3 b2 a3 b3
(1 + x ) (1 + x ) (1 + x )
Statement-1: ( a12 + b12 + c12 + d12 ) ( a22 + b22 + c22 + d22 )
then coefficient of x in expansion of D(x) is 0. can be written in the form a2 + b2 + c2 + d2 where
Statement-2: If P(x) = (1 + x)n, n ΠN then coef- a, b, c, d are some integers.
ficient of x in the expansion of P(x) is P¢(0). Statement-2: If a, b, c, d Œ Z, then
80. Statement-1: If a + b + c = 0 and a2 + b2 + c2 a + ib c + id
π bc + ca + ab, then the system of homogenous a2 + b2 + c2 + d2 =
equations -c + id a - ib
Determinants 4.39

LEVEL 2

Straight Objective Type Questions

84. If l12 + m12 + n12 = 1 etc., and l1l2 + m1m2 + n1n2 89. Suppose a, b, g, q ΠR and
= 0 etc., and cos (a + q ) sin (a + q ) 1
l1 m1 n1 A(a, b, g, q) = cos (b + q ) sin (b + q ) 1
D = l2 m2 n2 cos (g + q ) sin (g + q ) 1
l3 m3 n3
p p 2p
then Numerical value of A Ê - , 0, , ˆ is
Ë 2 2 13 ¯
(a) | D | = 3 (b) | D | = 2 (a) 0 (b) – 1
(c) | D | = 1 (d) D = 0 (c) 2 (d) none of these
85. If a, b, c are non-zero real number and 90. If a, b, c are positive integers such that a > b > c
and
b2 c 2 bc b+c 1 1 1
D = c2 a2 ca c+a a b c =–2
a 2 b2 ab a + b a 2 b2 c 2
then D equals then 3a + 7b – 10c equals
(a) abc (b) a2 b2 c2 (a) 10 (b) 11
(c) bc + ca + ab (d) 0 (c) 12 (d) 13
x2 - 5x + 3 2x - 5 3 91. If A, B, C, P, Q, R ΠR, and
86. Let D(x) = 3x2 + x + 9 6x + 1 9 cos ( A + P ) cos ( A + Q ) cos ( A + R)
2
7 x - 6 x + 9 14 x - 6 21 D = cos ( B + P ) cos ( B + Q) cos ( B + R)
= ax3 + bx2 + cx + d, cos (C + P ) cos (C + Q) cos (C + R)
then a equals (a) D depends on P, Q, R
(a) – 1 (b) 0 (b) D depends on A, B, C
(c) 2 (d) none of these (c) D depends on A, B, C, P, Q, R
87. The number of distinct values of t for which the (d) none of these
system 2 cos x 1 0
(a + t)x + by + cz = 0 92. Let f (x) = 1 2 cos x 1 then
ax + (b + t)y + cz = 0
0 1 2 cos x
ax + by + (c + t)z = 0
has a non-trivial solution is (a) f Ê p ˆ = 1 (b) f ¢ Ê p ˆ = - 3
Ë 3¯ Ë 3¯
(a) 1 (b) 2
(c) 3 (d) none of these p
88. If a2 + b2 + c2 = 1, then (c) f Ê ˆ = – 1 (d) none of these
Ë2 ¯
2 2 2
a + (b + c ) cos q ab (1 - cos q ) 93. Consider the set A consisting of all determinants
ba (1 - cos q ) 2 2
b + (c + a ) cos q2 of order 3 with entries 0 and 1 only. Let B be the
subset of A consisting of all the determinants with
ca(1 - cos q ) cb(1 - cos q ) value 1. Let C be the subset of A consisting of all
the determinants with value –1. Then
ac (1 - cos q ) (a) C = f
bc (1 - cos q ) equals (b) B has as many elements as C
2 2
c + (a + b )cos q 2 (c) A = B « C
(d) A = B » C
(a) cos2q (b) 0 94. let w = e2pi/3 and consider the system of linear
(c) 1 (d) sin2q equations
4.40 Complete Mathematics—JEE Main
x+y+z=a 96. If the adjoint of a 3 ¥ 3 matrix P is
x + w y +w2 z = b Ê1 4 4ˆ
x + w2 y + w z = c Á2 1 7˜ then
If x, y, z is a solution of the above system of equa- Á ˜
Ë1 1 3¯
| a |2 + | b |2 + | c |2
tions, then value of is the possible value(s) of determinant P is (are)
| x |2 + | y |2 + | z |2 (a) ± 2 (b) ± 3
(a) 9 (b) 6 (d) ± 1 (d) 0
(c) 3 (d) 1 1 -3 4
a a 2 a3 - 1 97. If -5 x + 2 2 = 0, then x equals
95. If a, b, c are distinct and b b2 b3 - 1 =0 4 1 x-6
2 3
then abc equals c c c -1 (a) 17, 21 (b) 0,19
(a) 0 (b) 1 (c) 0, 35 (d) 21, 35
(c) –1 (d) –2

Previous Years' AIEEE/JEE Main Questions

1. If the equation ax2 + 2bx + c = 0 has equal roots (c) satisfy a + 2b + 3c = 0


then the determinant (d) are in A.P. [2003]
a b ax + b 5. If a1, a2, a3, ... , an, ... are in G.P., then the value
D= of the determinant
b c bx + c
ax + b bx + c 0 log an log an +1 log an + 2
is log an + 1 log an + 2 log an + 3
(a) positive (b) negative log an + 2 log an + 3 log an + 4
(c) 0 (d) dependent on a.
[2002] is
2. If l, m, n > 0 and l, m, n are the pth, qth, rth terms (a) 2 (b) 1
of a G.P., then determinant (c) 0 (d) – 2 [2004, 2005]
6. If a2 + b2 + c2 = – 2 and
log l p 1
D = log m q 1 equals [2002] 1 + a2 x (1 + b2 ) x (1 + c 2 ) x
log n r 1 f(x) = (1 + a 2 ) x 1 + b2 x (1 + c 2 ) x
2 2
(a) 0 (b) – 1 (1 + a ) x (1 + b ) x 1 + c2 x
(c) p + q + r (d) none of these
then f(x) is a polynomial of degree
3. If 1, w, w2 are the cube roots of unity then
(a) 3 (b) 2
1 w n w 2n (1) 1 (d) 0 [2005]
D = wn w 2n 1 equals 7. The system of equations
ax + y + z = a – 1
w 2n 1 wn
(a) 1 (b) 2 x + ay + z = a – 1
(c) w2 (d) 0 [2003] x + y + az = a – 1
4. If the system of linear equations has no solution, if a is
x + 2ay + az = 0, x + 3by + bz = 0, x + 4cy + cz (a) not – 2
= 0 has a non-zero solution, then a, b, c (b) 1
(a) are in G.P (c) –2
(b) are in H.P. (d) either – 2 or 1 [2005]
Determinants 4.41

1 1 1 1
(a) (b) 1
8. If D = 1 1 + x 1 for x π 0, y π 0, then D is ab
1 1 1+ y (c) –1 (d) ab
[2014]
(a)divisible by neither x nor y
13. If a, b, c are non-zero real numbers and if the
(b)divisible by both x and y
system of equations
(c)divisible by x but not y
(d)divisible by y but not x [2007] (a – 1)x = y + z,
9. Leta, b, c be such that b(a + c) π 0. If (b – 1)y = z + x,
a a +1 a -1 (c – 1)z = x + y,
D = -b b +1 b -1 has a non-trivial solution, then ab + bc + ca equals:
c c -1 c +1 (a) a + b + c (b) abc
(c) 1 (d) –1 [2014 online]
a +1 b +1 c -1 14. Let for i = 1, 2, 3, pi(x) be a polynomial of degree
+ a -1 b -1 c +1 =0 2 in x, pi¢(x) and pi≤(x) be the first and second order
(-1)n + 2 a (-1)n + 1 b (-1)n c derivatives of pi(x) respectively. Let,
then the value of n is È p1 ( x ) p1¢ ( x ) p1¢¢( x ) ˘
(a) any odd integer A(x) = Í p2 ( x ) p2¢ ( x ) p2¢¢( x )˙
Í ˙
(b) any integer ÍÎ p3 ( x ) p3¢ ( x ) p3¢¢( x ) ˙˚
(c) zero
(d) any even integer and B(x) = [A(x)]T A(x). Then determinant of B(x).
[2009] (a) is a polynomial of degree 6 in x
10. If a, b, c are sides of a scalene triangle, then the (b) is a polynomial of degree 3 in x
a b c
(c) is a polynomial of degree 2 in x
value of D = b c a is:
(d) does not depend on x
c a b [2014 online]
(a) non-negative 15. If
(b) negative
a2 b2 c2
a 2 b2 c 2
(c) positive
2 2 2
(d) non-positive (a + l ) (b + l ) (c + l ) = kl a b c , l π 0,
[2013 online] ( a - l )2 ( b - l )2 ( c - l )2 1 1 1
11. Statement-1: The system of linear equations
x + (sin a)y + (cos a)z = 0 then k is equal to:
x + (cos a)y + (sin a)z = 0 (a) 4labc (b) – 4labc
x – (sin a)y – (cos a)z = 0 (c) 4l2 (d) – 4l2
[2014 online]
has a non-trivial solution for only one value of a
lying in the interval (0, p /2). r 2r - 1 3r - 2
Statement-2: The equation in a n
16. If Dr = n -1 a
cos a sin a cos a 2
D = sin a cos a sin a = 0 1 1
n(n - 1) (n - 1)2 (n - 1)(3n + 4)
2 2
cos a - sin a - cos a
n -1
has only one solution lyining in the interval
(0, p /2). [2013 online]
then the value of  Dr :
r =1
12. If a, b π 0, and f(n) = a n + b n and
(a) depends only on a
3 1 + f (1) 1 + f (2) (b) depends only on n
1 + f (1) 1 + f (2) 1 + f (3) (c) depends both on a and n
1 + f (2) 1 + f (3) 1 + f (4) (d) is independent of both a and n.
= K(1 – a)2 (1– b)2 (a – b)2, then K is equal to: [2014 online]
4.42 Complete Mathematics—JEE Main
17. The set of all values of l for which the system of (a) -2 2 (b) -16 2
linear equations: (c) – 8 (d) – 1 [2015 online]
2x1 – 2x2 + x3 = lx1 20. The system of linear equations
2x1 – 3x2 + 2x3 = lx2 x + ly – z = 0
– x1 + 2x2 = lx3 lx – y – z = 0
has a non-trivial solution, x + y – lz = 0
(a) is an empty set has a non-trivial solution for
(b) is a singleton l
(c) contains two elements (b) exactly one value of l
(d) contains more than two elements [2015] (c) exactly two values of l
x2 + x x +1 x-2 (d) exactly three values of l [2016]
21. The number of distinct real roots of the equation.
18. If D = 2 x2 + 3x - 1 3x 3 x - 3 = ax - 12,
cos x sin x sin x
2
x + 2x + 3 2x - 1 2x - 1
D = sin x cos x sin x = 0
then a is equal to: sin x sin x cos x
(a) 12 (b) 24
(c) – 12 (d) – 24 [2015 online] È p p˘
in the interval Í - , ˙ is:
*
19 . The least value of the product xyz for which the Î 4 4˚
x 1 1 (a) 1 (b) 4
determinant D = 1 y 1 is non-negative, is: (c) 2 (d) 3 [2016 online]
1 1 z

Previous Years’ B-Architecture Entrance


Examination Questions

1. If the system of equations n


x + y + z= 0 then the value of  D k depends
k =1
ax + by + z = 0
(a) only on a and b not on g
bx + y + z = 0 (b) on all a, b and g
has a non-trivial solution, then (c) on none of a, b and g
(a) b2 = 2b + 1 (b) b2 = 2b – 1 (d) only on a, not on b and g [2007]
(c) b – a = 0 (d) b2 = 2b [2006] 4. Let A = (aij) be a 3 × 3 matrix whose determinant
2. The system of linear equations is 5. The determinant of the matrix B = (2i – j aij) is
(l + 3)x +(l + 2)y + z = 0 (a) 5 (b) 10
3x + (l + 3)y + z = 0 (c) 20 (d) 40 [2008]
2x + 3y + z = 0 2r -1 2(3r -1 ) 4(5r -1 )
5. Let Dr = a b g , for
has a non-trivial solution
(a) if l = 1 (b) if l = –1 2n - 1 3n - 1 5n - 1
(c) for no real value of l n
(d) if l = 0 [2007] r = 1, 2,…, n. Then  D r is
r =1
3. If (a) independent of a, b, g and n
2(3k -1 ) 3(4 k -1 ) 4(5k -1 ) (b) independent of n only
Dk = a b g (c) depends on a, b, g and n
3n - 1 4n - 1 5n - 1 (d) independent of a, b, g only [2010]
*
Question is incorrect as xyz can take any real value.
Determinants 4.43
6. If x1, x2, x3,…, x13 are in A.P. then the value of
Answers
e x1 e x4 e x7
Concept-based
e x4 e x7 e x10 is:
x7 x10 x13 1. (d) 2. (d) 3. (a) 4. (d)
e e e
5. (a) 6. (c) 7. (d) 8. (a)
(a) 27 (b) 0 9. (a) 10. (b) 11. (d) 12. (c)
(c) 1 (d) 9 [2011] 13. (d) 14. (d) 15. (a)
7. The value of the determinant
Level 1
13 + 3 2 5 5
16. (a) 17. (c) 18. (a) 19. (b)
15 + 26 5 10 is equal to: 20. (d) 21. (c) 22. (a) 23. (c)
3 + 65 15 5 24. (d) 25. (a) 26. (d) 27. (d)
28. (a) 29. (a) 30. (a) 31. (d)
(a) 5 3 ( 6 - 5) (b) 5 3 ( 6 - 5 ) 32. (a) 33. (c) 34. (a) 35. (a)
(c) 5 ( 6 - 5) (d) 3 ( 6 - 5 ) [2012] 36. (d) 37. (d) 38. (d) 39. (d)
40. (a) 41. (a) 42. (d) 43. (d)
8. If the system of linear equations, x + 2ay + az =
44. (d) 45. (d) 46. (c) 47. (a)
0, x + 3by + bz = 0 and x + 4cy + cz = 0 has a
48. (d) 49. (b) 50. (d) 51. (d)
non-zero solution, then a, b, c satisfy:
52. (a) 53. (b) 54. (d) 55. (c)
(a) 2b = a + c (b) b2 = ac
56. (d) 57. (c) 58. (a) 59. (a)
(c) 2ac = ab + bc (d) 2ab = ac + bc [2013] 60. (d) 61. (d) 62. (a) 63. (d)
9. In a DABC, if 64. (a) 65. (b) 66. (a) 67. (a)
1 a b 68. (b) 69. (a) 70. (d) 71. (a)
1 c a = 0, 72. (d) 73. (d) 74. (c) 75. (a)
76. (a) 77. (d) 78. (b) 79. (a)
1 b c 80. (c) 81. (c) 82. (d) 83. (a)
then sin2A + sin2B + sin2C is
Level 2
3 9
(a) 3 (b) 84. (c) 85. (d) 86. (b) 87. (b)
2 4
88. (a) 89. (c) 90. (d) 91. (d)
5
(c) (d) 2 [2014] 92. (b) 93. (b) 94. (c) 95. (b)
4
96. (a) 97. (c)
10. The system of linear equations
x – y + z= 1 Previous Years’ AIEEE/JEE Main Questions
x + y – z= 3 1. (c) 2. (a) 3. (d) 4. (b)
x – 4y + 4z = a has: 5. (c) 6. (b) 7. (c) 8. (b)
(a) a unique solution when a = 2 9. (a) 10. (b) 11. (b) 12. (b)
(b) a unique solution when a π – 2 13. (b) 14. (d) 15. (c) 16. (d)
(c) an infinite number of solutions, when a = 2 17. (c) 18. (b) 19. (?) 20. (d)
(d) an infinite number of solutions, when a = –2 21. (c)
[2015]
Previous Years’ B-Architecture Entrance
Ê pˆ
11. For all values of q Œ Á 0, ˜ , the determinant of the Examination Questions
Ë 2¯
1. (b) 2. (c) 3. (c) 4. (a)
È -2 tan q + sec2 q 3 ˘ 5. (a) 6. (b) 7. (c) 8. (c)
Í ˙ 9. (b) 10. (d) 11. (a)
matrix D = Í- sin q cos q sin q ˙ always
Í -3 -4 3 ˙˚
Î
Hints and Solutions
lies in the interval:
(a) [3, 5] (b) (4, 6) Concept-based
Ê 5 19 ˆ È 7 21 ˘ 1. det (adj (A)) A–1) = det (adj(A)) det(A–1)
(c) Á , ˜ (d) Í , ˙ [2016]
Ë2 4 ¯ Î2 4 ˚ 1
= (det(A))2 = det(A)
det( A)
4.44 Complete Mathematics—JEE Main
2. Use R1 Æ R1 + R3 – 2R2 (a + 2016)2 (b + 2016)2 (c + 2016)2
3. Using R2 Æ R2 – x2R1 and R3 Æ R3 – xR1, we get D = - 4(2016) a b c
1 x a2 b2 c2
D = 0 1 - x3 x(1 - x 3 ) Applying R1 Æ R1 – 2 (2016) R2 – R3, we get
0 0 1 - x3 1 1 1
= (1 – x3)2 = (1 – x)2 (1 + x + x2)2 D = - 4 (2016)3 a b c
As x π 1, D = 0 fi 1 + x + x2 = 0 fi x = w, w2 a2 b2 c2
4. Taking 10!, 11! and 12! common from C1, C2, C3 = – 4 (2016)3 (a – b) (b – c) (c – a)
respectively, we get 11. Using R2 Æ R2 + R1, R3 Æ R3 – R1, we get
D = (10! 11! 12!) D1 x -6 -1
where P(x) = x + 2 -3( x + 2) x + 2
1 1 1 -3 - x 2( x + 3) x + 3
D1 = 11 12 13 x -6 -1
(11)(12) (12)(13) (13)(14) = (x + 2) (x + 3) 1 -3 1
Using C3 Æ C3 – C2 and C2 Æ C2 – C1, we get -1 2 1
1 0 0 Using R1 Æ R1 + R3, R2 Æ R2 – R3, we get
D1 = 11 1 1 =2 x -1 -4 0
(11)(12) 2(12) 2(13) P(x) = (x + 2) (x + 3) x -5 0
D -1 2 1
Thus, = 2(11) (12) = 264
(10!)3 = (x + 2) (x + 3) (– 5x + 13)
sum of zeros of P(x) is –2 – 3 + 13/5 = –12/5.
5. Product of zeros = –P(0) = 0
12. Write D as
1 3 2
6. D = area of triangle = ( 4) = 4 3 1 x x2
2 4
D = xn+1 yn+1 zn+1 1 y y2
fi |D| = 8 3 fi D2 = 192
1 z z2
7. Use R1 Æ R1 + R3 – 2R2. = (xyz) n+1
(x – y) (y – z)(z – x)
8. Write \ n+1 = 2 fi n = 1.
1+1 a+b a +b2 2 13. When x = 0, P(0) is a skew-symmetric determinant
of odd order.
D= a+b a 2 + b2 a 3 + b3 14. Write D = D1 + aD2 where
a 2 + b2 a 3 + b3 a 4 + b4 1 1 1
1 1 0 1 1 0 D1 = 1 + b 1 + 2 b 1
= a b 0 a b 0 =0 1 + c 1 + c 1 + 3c
a2 b2 0 a2 b2 0 1 0 0
and D2 = 1 + b 1 + 2b 1
9. Use R2 Æ R2 – R1, R3 Æ R3 – R1 and
1 + c 1 + c 1 + 3c
log x – log y = log (x/y) to obtain
= (1 + 2b) (1 + 3c) – (1 + c)
log a log b log c = 2 (b + c + 3c)
D = log (2007) log (2007) log (2007) = 0 In D1, apply C2 Æ C2 – C1 and C3 Æ C3 – C1 to
log (2017) log (2017) log (2017) obtain
[ R2 and R3 are proportional] 1 0 0
10. Using R2 Æ R2 – R1, we get D1 = 1 + b b -b = 2bc
1 + c 0 2c
Determinants 4.45
Thus, D = 2(bc + ac + ab + 3abc) 19. Using R1 Æ R1 – R3 – R2 we get R1 consists of all
1 1 1 zeros.
\ D = 0 fi + + = –3
a b c 20. If R is the common ratio of the G.P., then b = aR,
15. Taking xm common from R1 and xm+5 from R3, we c = aR2, d = aR3, e = aR4, f = aR5, and
get 1 1 p
1 x2 xm D = (a 2 )(a 2 R6 ) R 2 R2 q =0
D = x 2m +5 1 xn 2n
R4 R4 r
n-m
1 x xm [ C1 and C2 are identical]
If n – m = 2, then R1 and R3 are identical and hence 21. Multiply C1 by x, C2 by y and C3 by z to obtain
D = 0.
( x 2 + 1) x xy 2 xz 2
Level 1 1 2 2
5= yx ( y + 1) y yz 2
16. Using C1 Æ C1 + C2 + C3, we get xyz
zx 2 zy 2 z( z 2 + 1)
1 c+a a+b Taking x common from R1, y from R2 and z from
D = 2(a + b + c) 1 a + b b + c R3, we get
1 b+c c+a x2 + 1 y2 z2
Using R2 Æ R2 – R1, R3 Æ R3 – R1, we get xyz
5= x2 y2 + 1 z 2
xyz
1 c+a a+b x2 y2 z2
D = 2(a + b + c) 0 b - c c - a Using C1 Æ C1 + C2 + C3, we get
0 b-a c-b
5 = (x2 + y2 + z2 + 1) D1
= – 2(a + b + c) [(b – c)2 + (b – a) (c – a)]
= – (a + b + c) [(b – c)2 + (c – a)2 + (a – b)2] 1 y2 z2
Now, use D = 0, a + b + c π 0. where D1 = 1 y 2 + 1 z2
2 2
17. Write y = D1 + D2 where 1 y z +1
1 x x Applying R2 Æ R2 – R1, R3 Æ R3 – R1, we get
D1 = 0 x + 2 x = 5x + 6
1 y2 z2
0 x x+3
D1 = 0 1 0 =1
1 x x 1 0 0 0 0 1
and D2 = x 1 x + 2 x = x 1 2 0 = 6x
Thus, x2 + y2 + z2 = 4, which represents a sphere.
1 x x+3 1 0 3
Thus, y = 11x + 6 22. Apply R1 Æ aR1 + bR2 + cR3 and use a = b cos C
6 + c cos B etc.
Its distance from the origin =
122 23. Write D = D1 + xD2 where
18. Using C1 Æ C1 + C2 + C3, we get a-x x x
1 -a + b -a + c D1 = 0 b x = (a - x )(bc - x 2 )
D = (a + b + c) 1 3b -b + c 0 x c
1 -c + b 3c 1 x x
Now, use R2 Æ R2 – R1, R3 Æ R3 – R1, we get and D2 = 1 b x
1 -a + b -a + c 1 x c
D = ( a + b + c ) 0 2b + a - b + a 1 x x
0 - c + a a + 2c = 0 b-x 0 [use R2 Æ R2 – R1,
D = (a+b+c) [(a + 2b) (a + 2c) – (a – c) (a – b)] 0 0 c-x R 3 Æ R 3 – R 1]
= 3(a + b + c) (bc + ca + ab) = (b – x) (c – x)
4.46 Complete Mathematics—JEE Main
Thus, D = (a – x) (bc – x2) + x(b – x) (c – x) = – a[b g + g a + a b – (a 2 + b 2 + g 2)] (2)
3 2
= 2x – (a + b + c)x + abc = – a[0 – a2] = a3
Also, x f ¢(x) – f(x) 27. In this case a + b + g = 0.
Thus, from (1) in solution to question 26, we get
= x [(x – b) (x – c) + (x – a) (x – c)
D = 0.
+ (x – a) (x – b)] – f (x)
28. We can write
= 2x3 – (a + b + c)x2 + abc a b g
24. Applying R1 Æ R1 + R2 + R3, we get D = (abc)2 b g a
1 1 1 g a b
0 = ( x + y + 2) 2 x x-y-2 2x 1 1 1
where a= ,b= ,g=
2y 2y y-2- x a b c
Using C2 Æ C2 – C1, C3 Æ C3 – C1, we get From (2) in solution question 26, we have
D = (abc)2 (a + b + g) [(b – g)2 + (g – a)2
1 0 0
+ (a – b)2]
0 = ( x + y + 2) 2 x - ( x + y + 2 ) 0
fi a + b + g = 0 fi a + b + c–1 = 0
–1 –1

2y 0 - ( x + y + 2) 29. Using C2 Æ C2 – l2C3, we can write


fi 0 = (x + y + 2)3
a a2 1
fi x+y+2=0
D = l b b2 1
It passes through (–1, –1).
25. Write f(x) = D1 + xD2 where c2 1
c
Using R3 Æ R3 – R2, R2 Æ R2 – R1, we get
a x x
D1 = 0 x+a x = a[(x + a)2 – x2] a a2 1
0 x x+a D= l b-a b -a 2 2
0
= 2 xa2 + a3 c-b c 2 - b2 0
1 x x 1 b+a
and D2 = 1 x + a = l(b – a) (c – b)
x 1 c+b
1 x x+a = l(b – a) (c – b) (c – a)
1 x x = l(a – b) (b – c) (c – a)
= 0 a 0 = a2 30. Use C2 Æ C2 – C3 + C1 to show that C2 consists
of all zeros.
0 0 a 31. We know
Thus, f (x) = 2xa2 + a3 + xa2 1 1 1
= 3xa2 + a3 a b c = (a – b) (b – c) (c – a)
\ f(2x) – f(x) = 3xa2 a2 b2 c2
2
26. a + b + g = – a, b g + g a + a b = 0, a b g = – b. 1 a a3
Applying C1 Æ C1 + C2 + C3, we get Let D1 = 1 b2 b3
1 b g 1 c 2
c3
D = (a + b + g ) 1 g a (1) Using R3 Æ R3 – R2 and R2 Æ R2 – R1, we get
1 a b 1 a2 a3
Using R3 Æ R3 – R2 and R2 Æ R2 – R1, we get D1 = 0 b 2 - a 2 b3 - a 3
1 b g 0 c -b 2 2
c 3 - b3
D= – a 0 g -b a -g
b + a b2 + a 2 + ba
0 a -g b -a = (b – a) (c – b)
c+b c 2 + b2 + cb
= – a[(g – b ) (b – a) – (a – g )2]
Determinants 4.47
3
Applying C2 Æ C2 – bC1, we get Thus, D = 0 fi Ê a ˆ = 8
Ë b¯
b + a a2 fi a/b is a cube root of 8.
D1 = (b – a) (c – b)
c+b c2 39. Using C1 Æ C1 + C2 + C3, we can write D =
2(1 + i)D1 where
b+a a2
= (b – a) (c – b) 1 1+ i i
c - a c - a2
2 1 1+ i i
D1 = 1 i 1 = 0 -1 1- i = i
[using R2 Æ R2 – R1]
1 1 1+ i 0 -i 1
b + a a2
= (b – a) (c – b) (c – a) [using R2 Æ R2 – R1 and R3 Æ R3 – R1]
1 c+a
Thus, D = 2(i – 1)
= (a – b) (b – c) (c – a) (bc + ca + ab)
Thus, l = 18 1 1 1
1 +
32. Show that D = (a0 + a1 + a2)x2 c a b
33. Using R2 Æ R2 + R1, we get 1 1 1
40. Write D = abc 1 +
a b c
1 cos x 0
1 1 1
D= 0 1 sin + cos x 1 +
b c a
0 - 1 1 - (sin x + cos x )
and use C2 Æ C2 + C1, to show that D = 0
Using R3 Æ R3 + R2, we get 41. Applying C1 Æ C1 – C2 – C3 and reduce D to
1 cos x 0 determinant in Example 36.
D= 0 1 sin + cos x = 1 42. Applying R1 Æ R1 – xR2 we get
0 0 1 a (1 - x 2 ) c (1 - x 2 ) p (1 - x 2 )
34. Using C1 Æ C1 + C2 + C3, we get D = ax + b cx + d px + q
1 a b u v w
D = (x + a + b) 1 x b Take 1 – x2 common from R1 and apply R2 Æ R2
1 b x – xR1 to obtain
a c p
Using R3 Æ R3 – R2, R2 Æ R2 – R1, we get
D = (1 – x2) b d q
1 a b
u v w
D = (x + a + b) 0 x - a 0
43. Applying R1 Æ R1 + R3 to obtain
0 b-a x-b
= (x + a + b) (x – a) ( x – b) 0 0 2
35. Use R2 Æ R2 – R1, R3 Æ R3 – R1to obtain D= - sin q 1 sin q = 2(sin2q + 1)
1 -1 - sin q 1
D = sinq cosq = sin(2q)
2 As 0 £ sin2q £ 1, we get D Œ [2, 4].
36. Use C1 Æ C1 + C2 + C3 to obtain 44. Write D = D1 – D2 where
1 b c
b2 - ab b bc - ac
D = (x + a + b + c) 1 x + b c
D1 = ab - a 2 a b2 - ab and
1 b x+c
bc - ac c ab - a 2
Now use R2 Æ R2 – R1, R3 Æ R3 – R1 to obtain
D = x2(x + a + b + c) b2 – ab c bc – ac
Thus, D = 0 fi x = 0 or x = – (a + b + c) D2 = ab – a 2
b b2 – ab
37. Use C1 Æ C1 – C2. bc – ac a ab – a 2
38. Show D = – a3 + 8b3 In D1 use C1 Æ C1 – (b – a)C2 to show that D1
= 0.
4.48 Complete Mathematics—JEE Main
In D2 use C3 Æ C3 – (b – a)C2 to show that D2 49. Show that the determinant equals
= 0. x3 + (a2 + b2 + c2) x = 0.
Thus, D = 0. fi x = 0 as x ΠR
45. We can write \ Number of real roots of the equation is one.
50. Note that
6i 1 1
C11 C12 C13
D = – 3iw 4 - 1 -1 = 0
D1 = C21 C22 C23
20 i i
\ x = 0, y = 0 C31 C32 C33
46. Using C1 Æ C1 + C2 + C3, we get where Cij cofactor of (i, j)th element of

1 cos x cos x È1 x x2 ˘
Í 2
˙
D = (sin x + 2cos x) 1 sin x cos x Íx x 1˙
Í 2 ˙
1 cos x sin x Îx 1 x˚
Applying R2 Æ R2 – R1, R3 Æ R3 – R1, we get \ D1 = D2 = 49
51. Applying C3 Æ C3 – C1, we get
1 cos x cos x
D = (sin x + 2cos x) 0 sin - cos x sin a cos a cos b
0
D = sin b cos a cos b =0
0 0 sin x - cos x
sin g cos a cos b
= (sin x + 2cos x) (sin x – cos x)2
52. Using R2 Æ R2 – R1 and R3 Æ R3 – R1, and
D = 0 fi tan x = – 2, tan x = 1.
p p Ê xˆ
log x – log y = log Á ˜ , we get
As – £x£ , – 1 £ tan x £ 1. Ë y¯
4 4
Thus, tan x = 1 fi x = p/4. log a log b log c
47. Applying C1 Æ C1 + iC2, we get D= log 7 log(7 ) log(73 )
2

0 i -w log(3) log(32 ) log(33 )


x + iy = 0 1 w 2 = (log 7) (log 3) D1
w - iw 2 - w2 1 log a log b log c
where D1 = 1 2 3 =0
= (w – iw2) (iw2 + w)
1 2 3
= w2 – i2w4 = w2 + w = – 1
\ D =0
fi x = – 1, y = 0.
53. C3 Æ C3 + C1 + C2, gives
48. Taking conjugate, we get
1 + cos2 q sin 2 q 1
1 e- p i / 4 e- p i / 3
2
D = (2 + 4sin4q) cos q 1 + sin 2 q 1
x – iy = ep i / 4 1 e - 2p i / 3
pi / 3 2p i / 3 2p i / 3 cos2 q sin 2 q 1
e e e
Applying R3 Æ R3 – R2, R2 Æ R2 – R1, we get
1 ep i / 4 ep i / 3
= e -p i / 4 1 e2p i / 3 1 + cos2 q sin 2 q 1
-p i / 3 - 2p i / 3 2p i / 3 D = 2(1 + 2sin4q) -1 1 0
e e e
Now, 0 -1 0
2iy = (x + iy) – (x – iy) = 2(1 + 2sin4q)
1 ep i 4 0 D = 0 fi sin4q = – 1/2
= e- p i 4
1 0 p
Now, 0 £ q £ fi 0 £ 4q £ 2p
- pi 3 - 2p i 3 - 2p i 3 2p i 3 2
e e e -e
– 2pi/3 2pi/3 7p 11p 7p 11p
= (e –e ) [1 – 1] = 0 \ 4q = , fi q= ,
fi y = 0. 6 6 24 24
Determinants 4.49
54. p q r
fi + + = 2.
( x -1)( x +1) ( x +1)2 (2 x +1)( x +1) p-a q-b r-c
P(x) = (2 x -1)( x +1) (2 x -1)( x + 3) (2 x + 3)(2 x -1) Now use
(2 x +1)(3 x - 2) (2 x -1)(3 x - 2) (4 x +1)(3 x - 2) p+a 2 p - ( p - a) 2p
= = - 1 etc.
p-a p-a p-a
x - 1 x + 1 2x + 1
57. We can write D as
= ( x + 1)(2 x - 1)(3 x - 2) x + 1 x + 3 2 x + 3
1 n + 1 (n + 1) (n + 2)
2x + 1 2x - 1 4x + 1
D = n! (n + 1)! (n + 2)! 1 n + 2 (n + 3) (n + 2)
Using C2 Æ C2 – C1, C3 Æ C3 – 2C1, we get
1 n + 3 (n + 4) (n + 3)
x -1 2 3
Applying R3 Æ R3 – R3, R2 Æ R2 – R1, we get
P(x) = ( x + 1)(2 x - 1)(3 x - 2) x + 1 2 1
1 n + 1 (n + 1) (n + 2)
2 x + 1 -2 -1
Using R2 Æ R2 – R1, R3 Æ R3 + R1, we get D = n! (n + 1)! (n + 2)! 0 1 2 ( n + 2)
0 1 2 (n + 3)
x -1 2 3
P(x) = ( x + 1)(2 x - 1)(3 x - 2) 2 0 -2 D
fi =2
3x 0 2 n!(n + 1) ! (n + 2)!

P(x) = – 4(x + 1) (2x – 1) (3x – 2) (3x + 2) 58. Use R1 Æ R1 + R3 – 2R2


59. Using C1 Æ C1 + C2 + C3, we get
\ P(x) = 0 has four distinct roots.
2p 2p 1 1- y 1- y
55. Put ar = wr where w = cos + i sin .
9 9 D = (3 – y) 1 1 + y 1 - y
1 1 1- y 1+ y
Note that w9 = 1 fi = w8. Now,
w Applying R3 Æ R3 – R2, R2 Æ R2 – R1, we get
1 w8 w7 1 1- y 1- y
D= w 3
w 2
w =0 D = (3 – y) 0 2 y 0 = (3 - y ) ( 4 y 2 )
w 6
w 5
w 4 0 -2 y 2y
as C2 and C3 are proportional. \ D = 0 fi y = 0 or y = 3.
56. As the system has a non-zero solution 60. Write D = lD1 + l¢D2, where
p b c a am + a¢ m¢ an + a ¢ n¢
D= a q c = 0 D1 = b bm + b¢ m¢ bn + b¢ n¢ and
a b r c cm + c¢ m ¢ cn + c¢ n¢
Applying R3 Æ R3 – R2, R2 Æ R2 – R1, we get a¢ am + a ¢ m¢ an + a¢ n¢
D2 = b¢ bm + b¢ m¢ bn + b¢ n¢
p b c
c¢ cm + c¢ m¢ cn + c¢ n¢
D= a- p q-b 0 =0
In D1 apply C2 Æ C2 – mC1, C3 Æ C3 – nC1 to
a- p 0 r-c
obtain
Expanding along R3, we get a a ¢ m ¢ a ¢ n¢
– (a – p) (q – b)c + (r – c) [p(q – b) – b(a – p)] D1 = b b ¢ m ¢ b ¢ n ¢ = 0
= 0.
c c ¢ m ¢ c ¢ n¢
Dividing (p – a) (q – b) (r – c), we get
Similarly D2 = 0. Thus, D = 0
c p b
+ + = 0. 61. Imitate Example 35.
r-c p-a q-b
62. We have
c-r r p b-q q
fi + + + + =0
r-c r–c p-a q-b q-b
4.50 Complete Mathematics—JEE Main

1 1 1 Using C1 Æ C1 + C2 + C3, we get


m m+3 m+6 b c
D= 1
1 1 1 q r
m (m - 1) (m + 3) (m + 2) (m + 6) (m + 5)
2 2 2 Ê a b cˆ b c
D = pqr Á 1 + + + ˜ 1 1 +
Applying C3 Æ C3 – C2, C2 Æ C2 – C1, we get Ë p q r¯ q r
1 0 0 b c
1 1+
m 3 3
q r
D= Apply R2 Æ R2 – R1, R3 Æ R3 – R1, we get
1
m (m - 1) 3 (m + 1) 3(m + 4)
2 Ê a b cˆ
D = pqr Á 1 + + + ˜ (1)
= 32(m + 4 – m – 1) = 33 Ë p q r¯
\ a+b+g=3 a b c
63. Use C2 Æ C2 – 2C1, C3 Æ C3 – 3C1 to obtain C2 \ D = 0, pqr π 0 fi + + = – 1.
p q r
and C3 are proportional.
68. The system will have a non-zero solution if
0 cos C cos B
1 a b aa +b
64. Write D = a cos C - 1 cos A
a D= b c ba + c = 0
a cos B cos A - 1
aa + b ba + c 0
Applying C1 Æ C1 + bC2 + cC3, we get
Applying R3 Æ R3 – aR1 – R2 to obtain
a cos C cos B a b aa +b
1
D= 0 - 1 cos A = sin2A D= b c ba + c
a
0 cos A - 1
0 0 – (a a 2 + 2ba + c)
65. For x π 0,
= – (aa2 + 2ba + c) (ac – b2)
cos x 1 1
Note that D = 0 if a, b, c are in G.P.
f ( x) sin x
= 2 1 2 69. The system will have a non-trivial solution if
x2 x
a a -1
tan x 1 1
D= b -1 b = 0.
1 1 1 1 0 0 -1 c c
f ( x)
lim = 2 1 2 = 2 1 2 =–1
x Æ0 x 2 Using C3 Æ C3 – C2, C2 Æ C2 – C1, we get
0 1 1 0 1 1
a 0 - (1 + a)
[using R1 Æ R1 – R3] D= b - (1 + b) (b + 1)
66. Applying C2 Æ C2 – w2C1 we find the second -1 c +1 0
column of D becomes 0.
67. The system will have a non-trivial solution if = – a( b + 1) (c + 1) – (1 + a)[b(c + 1)
– (1 + b)]
p+a b c
a b 1
D= a q+b c =0 \ D = 0 fi - - + =0
1+ a 1+ b 1+ c
a b r+c
1 1 1
Write fi + + = 2.
a b c 1+ a 1+ b 1+ c
1+ 70. The system will have a non-trivial solution if
p q r
D = 0 where
a b c l+5 l-4 1
D = pqr 1+
p q r
D= l -2 l +3 1
a b c
1+ l l 1
p q r
Applying C2 Æ C2 – lC3, C1 Æ C1 – lC3, we get
Determinants 4.51

5 -4 1 If n is odd, then |A| = – |A| fi 2|A| = 0 fi |A| = 0.


We have
D= -2 3 1 = 7 π 0
0 sin(p / 12) cos(p / 12)
0 0 1 Êpˆ
D Á ˜ = - sin(p / 12) 0 sec(p / 12) = 0
Thus, the system cannot have a non-trivial solution. Ë 4¯
71. Adding above the system of equations, we get - cos(p / 12) - sec(p / 12) 0
(a + b + c) (x + y + z) = 0 fi x + y + z = 0 [using statement-2]
\ (b + c) (– x) – ax = b – c 77. Statement-2 is true, see theory.
b-c The roots of x9 = p are p1/9 wr
fi x = – etc.
a+b+c 2p 2p
where w = cos + i sin .
72. The above system of equations will have a non- 9 9
trivial solution if value of determinant D depends on a1, a2, . . . a9.
a b c If we put ak = p1/9 w k, then
D= b c a = 0 1 w8 w7
c a b
D = (p1/9)3 1 w w2 = 0
1
But D = – (a + b + c) 1 w w2
2
[(b – c)2 + (c – a)2 + (a – b)2]
However, if a1 = p1/9, a2 = p1/9 w8,
\ D = 0, a + b + c π 0 a3 = p1/9 w7, a4 = p1/9 w, a5 = p1/9 w5
fi a = b = c.
a6 = p1/9 w4, a7 = p1/9 w2, a8 = p1/9 w3,
Thus, a : b : c = 1 : 1 : 1
a9 = p1/9 w6, then
6 6 0
1 w8 w7
73. f ¢¢¢(x) = 1 -6 4
D = (p1/9)3 w w5 w4
p p2 p3
w2 w3 w6
\ f ¢¢¢(x) is a constant.
74. Write = p1/3 (2w2 – w7 – w6) π 0.
D = D1 + pD2 where 78. Statement-2 is true.
a1 b1 + q c1 c1 + ra1 Using R2 Æ R2 – 2 R1 and R3 Æ R3 – 3 R1, we
get
D1 = a2 b2 + qc2 c2 + ra2 and
a3 b3 + qc3 c3 + ra3 6 2i 3 + 6i

b1 b1 + q c1 c1 + ra1 w= 0 3 ( 6 - 2 3 )i
D2 = b2 b2 + qc2 c2 + ra2 0 2 (2 - 3 2 )i
b3 b3 + qc3 c3 + ra3 = 6 ÎÈ2 3 – 3 6 – 2 3 + 2 6 ˘˚ i = – 6i
Now show that D1 = D and D2 = pqr D.
79. We first show statement-2 is true.
75. The system will have a non-trivial solution if
Ê nˆ Ê nˆ Ê nˆ
l +3 l +2 1 P(x) = 1 + Á ˜ x + Á ˜ x 2 + . . . + Á ˜ x n
Ë1 ¯ Ë 2¯ Ë n¯
D= 3 l + 3 1 = 0.
Ê nˆ Ê nˆ Ê nˆ
2 3 1 fi P¢(x) = Á ˜ + 2 Á ˜ x + . . . + n Á ˜ x n - 1
Ë1 ¯ Ë 2¯ Ë n¯
Using R1 Æ R1 – R2 and R2 Æ R2 – R3, we get
Ê nˆ
fi P¢(0) = Á ˜ = coefficient of x in the expan-
l -1 0 Ë1 ¯
2 sion of P(x)
D= 1 l 0 =l +1 \ Statement-2 is true.
2 3 1 Note that D(x) consists of 6 terms of the form
Note that there is no real value of l for which (1 + x)n.
D = 0. Thus, coefficients of x in D(x) = D¢(0)
76. Let A be a skew symmetric matrix of order n, then a1b1 a1b2 a1b3 1 1 1
|A¢| = (– 1)n |A| fi |A| = (– 1)n |A| But D¢(0) = 1 1 1 + a2 b1 a2 b2 a2 b3
1 1 1 1 1 1
4.52 Complete Mathematics—JEE Main

1 1 1 We can write D2 as
+ 1 1 1 =0 a1 b1 0 b1 a1 0
a3 b1 a3 b2 a3 b3 D2 = a2 b2 0 b2 a2 0 = 0
80. Statement-2 is not always true. For instance, the a3 b3 0 a3 b3 0
system of equations for all values of a1, a2, a3, b1, b2, b3 ΠR.
x + 2y + 3z = 1 a + ib c + id
2x + 3y + 4z = 2 83. As
-c + id a - ib
3x + 4y + 5z = 4 = (a + ib) (a – ib) + (c – id) (c + id)
has no solution but |A| = 0
= a2 + b2 + c2 + d2
For statement-1, let
a b c we get statement-2 is true.
Now,
D= b c a
c a b
(a12 + b12 + c12 + d12 ) (a22 + b22 + c22 + d22 )
Using C1 Æ C1 + C2 + C3, we obtain a1 + ib1 c1 + id1 a2 + ib2 c2 + id2
=
1 b c -c1 + id1 a1 - ib1 - c2 + id2 a2 - ib2
D = (a + b + c) 1 c a = 0 a + ib c + id
= = a 2 + b2 + c2 + d 2
1 a b -c + id a - ib
Also, note that x = y = z satisfies each of the three where a = a1a2 – b1b2 + c1c2 – d1d2,
equations. b = a 2b 1 + a 1b 2 + c 1d 2 + c 2d 1,
Thus, the system of equations has infinite number
c = – a 2c 1 + b 2d 1 – b 2c 1 + a 2d 1,
of solutions.
81. Let d = – b 2c 1 + a 2d 1 – b 1c 2 – b 1d 2
a b c
Level 2
D= b c a
84. We have
c a b
l1 m1 n1 l1 m1 n1
1
= - (a + b + c)[(b - c)2 + (c - a)2 + (a - b)2 ] D2 = l2 m2 n2 l2 m2 n2
2
l3 m3 n3 l3 m3 n3
If a + b + c π 0 and a2 + b2 + c2 = bc + ca + ab,
then (b – c)2 + (c – a)2 + (a – b)2 = 0 a1 b21 b31
fi a = b = c π 0 [ a + b + c π 0] = b12 a2 b32
Thus, the system of equation reduces to x + y + z = 0 b13 b23 a3
which is satisfied by infinite number of solutions.
\ Statement-1 is true. where ak = lk2 + mk2 + nk2 = 1 for k = 1, 2, 3
If a + b + c = 0 and a2 + b2 + c2 π bc + ca + and bij = li lj + mimj + ninj = 0 " i π j
ab, then at least one following is true: 1 0 0
a2 π bc, b2 π ca, c2 π ab. 2
Thus, D = 0 1 0 = 1 fi |D| = 1.
2
Suppose b – ac π 0
0 0 1
Write first two equations as:
ax + by = (a + b)z 85. Using R1 Æ aR1, R2 Æ bR2, R3 Æ cR3,
bx + cy = (b + c)z we get
Eliminating y, we get ab2 c 2
abc ab + ac
(ac – b2)x = (ac – b2)z 1 2 2
D= a bc abc bc + ab
fi x= z abc 2 2
Thus, from (1) equation, we get a b c abc ac + bc
by = bz fi y = z bc 1 ab + ac
\ x= y=z a2 b2 c 2
= ca 1 bc + ab
Therefore, statement-2 is false. abc
82. If (a1, b1), (a2, b2) (a3, b3) are non-collinear, then ab 1 ac + bc
D1 π 0. Therefore statement-2 is true.
Determinants 4.53
using C3 Æ C3 + C1, we get C2 and C3 are 88. First multiply C1 by a, C2 by b, C3 by c, followed
propotional. Thus, D = 0 by multiplying R1 by 1/a, R2 by 1/b and R3 by 1/c,
86. Using C1 Æ C1 – (x2/3)C3, C2 Æ C2 – (2x/3) C3, we get
we get
a2 + (b2 + c 2 )cos q b2 (1 - cos q )
- 5x + 3 - 5 3
D= a2 (1 - cos q ) b2 + (c 2 + a2 )cos q
D(x) = x+9 1 9
- 6 x + 9 - 6 21 a2 (1 - cos q ) b2 (1 - cos q )
Using C1 Æ C1 – xC2, we get c 2 (1 - cos q )
3 -5 3 c 2 (1 - cos q )
D(x) = 9 1 9 = a constant. c 2 + (a2 + b2 )cos q
9 -6 21
Using C1 Æ C1 + C2 + C3 and a2 + b2 + c2 = 1,
Thus, a = 0.
we get
Alternate Solution 1 b 2 (1 - cos q ) c 2 (1 - cos q )
Replacing x by 1/x, we get
D = 1 b + (1 - b ) cos q c 2 (1 - cos q )
2 2
1 Ê 1ˆ Ê 1ˆ
-5 Á ˜ + 3 2 Á ˜ - 5 3 b (1 - cos q )
2
c + (1 - c 2 ) cos q
2
x2 Ë x¯ Ë x¯ 1
Ê 1ˆ 1 Ê 1ˆ Using R2 Æ R2 – R1, R3 Æ R3 – R1, we get
3Á 2 ˜ ++9 6Á ˜ +1 9
Ëx ¯ x Ë x¯
1 b2 (1 - cos q ) c 2 (1 - cos q )
Ê 1ˆ Ê 1ˆ Ê 1ˆ
7Á 2 ˜ -6 Á ˜ + 9 14 Á ˜ - 6 21 D= 0 cos q 0 = cos2 q
Ëx ¯ Ë x¯ Ë x¯
0 0 cos q
Ê 1ˆ Ê 1ˆ Ê 1ˆ
= aÁ 3˜ + bÁ 2 ˜ + cÁ ˜ + d
Ëx ¯ Ëx ¯ Ë x¯ 89. We have
1 - 5x + 3x 2
2 - 5x 3 - sin(a + q ) sin(a + q ) 1
dA
fi 3 + x + 9 x 2 6+x 9 = a + bx + cx2 + dx3 = - sin(b + q ) sin(b + q ) 1
dq
2 - sin(g + q ) sin(g + q ) 1
7 - 6x + 9x 14 - 6 x 21
Putting x= 0, we get cos(a + q ) cos(a + q ) 1
1 2 3 + cos(b + q ) cos(b + q ) 1
a= 3 6 9 =0 cos(g + q ) cos(g + q ) 1
7 14 21 =0+0=0
[ C1 and C2 are proportional]
fi A is independent q. Thus, A(a, b, g, q ) =
87. The given system of equations will have a non- A (a, b, g , 0 )
trivial solution if
Ê p p 2p ˆ Ê p p ˆ
a+t b c fi A Á - , 0, , ˜ = A Á - , 0, , 0˜
Ë 2 2 13 ¯ Ë 2 2 ¯
D= a b+t c =0
0 -1 1
a b c+t
= 1 0 1 =2
Using C1 Æ C1 + C2 + C3, we get
0 1 1
1 b c
90. We have D = (a – b) (b – c) (c – a) = – 2.
D = (a + b + c + t) 1 b + t c =0
1 b c+t As a > b > c, a – b, b – c are positive integers and
c – a is a negative integers.
Using C2 Æ C2 – bC1, C3 Æ C3 – cC1, we get
Only possibilities are
D = (a + b + c + t)t2 = 0 fi t = 0, – (a + b + c)
a – b = 2, b – c = 1, c – a = –1 (1)
Thus, there are just two distinct values of t.
4.54 Complete Mathematics—JEE Main
or a – b = 1, b – c = 2, c – a = –1 (2) 93. For each D Œ B, there exist D1 Œ C where D1 is
obtained by interchanging 1st and 2nd row of D,
or a – b = 1, b – c = 1, c – a = –2 (3)
similarly, for each D ΠC there exists D1 ΠB.
(1) and (2) lead us to 0 = 2.
\ B and C has same number of elements.
\ a – b = 1, b – c = 1, c – a = –2.
94. |a|2 = |x|2 + |y|2 + |z|2 + x y + x y + x z + x z
Now, 3a + 7b – 10c = 3(a – c) + 7(b – c) = 13
+ yz + yz
91. Write
\ |a|2 + |b|2 + |c|2 = 3(|x|2 + | y|2 + |z|2) +
cos A sin A 0 cos P - sin P 0
D = cos B sin B 0 cos Q - sin Q 0 ( x y + x y + x z + x z + y z + y z) (1 + w + w2)
cos C sin C 0 cos R - sin R 0 | a |2 + | b |2 + | c |2
fi =3
=0 | x |2 + | y |2 + | z |2
Alternate Solution 95. Write the determinant as = abc D1 – D2

Write D = cos P D1 – sin P D2 1 a a2


cos A cos( A + Q) cos( A + R) where D1 = 1 b b2 = (a – b) (b – c) (c – a)
where D1 = cos B cos( B + Q) cos( B + R) 1 c c2
cos C cos(C + Q) cos(C + R)
a a2 1
sin A cos( A + Q) cos( A + R)
and D2 = b b2 1 = D1
and D2 = sin B cos( B + Q) cos( B + R)
c c2 1
sin C cos(C + Q) cos(C + R)
In D1, use C2 Æ C2 – cosQ C1 \ (a – b) (b – c) (c – a) (abc – 1) = 0
Since, a, b, c are distinct, we get
C3 Æ C3 – cosR C1
abc – 1 = 0 or abc = 1
cos A sin A sin Q sin A sin R
1 4 4
D1 = cos B sin B sin Q sin B sin R
96. | P |2 = |adj P| = 2 1 7 = 4
cos C sin C sin Q sin C sin R
1 1 3
=0 [ C2 and C3 are propotional]
fi | P | = ± 2
Similarly, D2 = 0 97. Using R2 Æ R2 + 5R1, R3 Æ R3 – 4R1,
Thus, D=0 we get
92. Expanding along C1, we get 1 -3 4
2 cos x 1 1 0 0 x - 13 22 =0
f(x) = 2cosx – 0 13 x - 22
1 2 cos x 1 2 cos x
= 8cos3 x – 4cos x = 4cos x cos2x fi (x – 13) (x – 22) – (13)(22) = 0

Êpˆ Êpˆ fi x(x – 35) = 0 fi x = 0, 35


fi f Á ˜ = –1, f Á ˜ = 0.
Ë 3¯ Ë 2¯
Previous Years’ AIEEE/JEE Main Questions
f¢(x) = – 4[sinx cos2x + 2cosx sin2x]
1. As ax2 + 2bx + c = 0 has equal roots,
= – 4[sin3 x + cosx sin2x]
b2 – ac = 0
Êpˆ È Êpˆ Ê 2p ˆ ˘
fi f ¢ Á ˜ = - 4 Ísin p + cos Á ˜ sin Á ˜ ˙
Ë 3¯ Î Ë 3¯ Ë 3 ¯˚ Using R3 Æ R3 – xR1 – R2, we get

=– 3 a b ax + b
D = b c bx + c
0 0 a
Determinants 4.55
where a = 0 – x(ax + b) – (bx + c) 1 2 1
- + =0
= – (ax + 2bx + c)
2 a b c

\ D = a (ac – b2) = 0 fi a, c, b are in H.P.

2. As l, m, n are pth, qth, rth terms of a G.P., log l, 5. Let R be the common ratio of the G.P. Applying
log m, log n are pth, qth, rth terms of an A.P. Let C3 Æ C3 – C2 and C2 Æ C2 – C1, we get that the
a and its common differ- given determinant
ence be d.
log an log R log R
Now, = log an +1 log R log R = 0
a + ( p - 1)d p 1 log an + 2 log R log R
D = a + (q - 1)d q 1
a + (r - 1)d r 1 6. Applying C1 Æ C1 + C2 + C3, we get

Using C1 Æ C1 – dC2 – (a – d)C3, we get 1 + 2 x + (a 2 + b 2 + c 2 ) x (1 + b 2 ) x (1 + c 2 ) x


0 p 1 f(x) = 1 + 2 x + (a 2 + b 2 + c 2 ) x 1 + b2 x (1 + c 2 )xx
D= 0 q 1=0 1 + 2 x + (a 2 + b 2 + c 2 ) x (1 + b 2 ) x 1 + c2 x
0 r 1
1 (1 + b 2 ) x (1 + c 2 ) x
3. If n is a multiple of 3, we get each element of D =1 1 + b2 x (1 + c 2 ) x
becomes 1. 1 (1 + b ) x
2
1 + c2 x
\ D=0
Applying R2 Æ R2 – R1 and R3 Æ R3 – R1, we get
If n = 3k + 1, then
1 (1 + b 2 ) x (1 + c 2 ) x
2
1 w w f ( x) = 0 1- x 0
D= w w 2
1 =0 [use C1 Æ C1 + C2 + C3] 0 0 1- x
2
w 1 w = (1 – x)2

If n = 3k + 2, then which is a polynomial of degree 2.

1 w2 w 7. If A
[use C1 Æ C1 + C2 + C3] a 1 1 1 1 1
D = w2 w 1 =0
| A | = 1 a 1 = (a + 2) 1 a 1
w 1 w2
1 1 a 1 1 a
4. As the system has a non-zero solution
[using C1 Æ C1 + C2 + C3]
1 2a a
1 3b b = 0 1 1 1
1 4c c = (a + 2) 0 a - 1 0
0 0 a -1
1/ a 2 1
1/ b 3 1 = 0 = (a + 2)(a – 1)2
1/ c 4 1 If |A| π 0, the system has a unique solution.

Using R1 Æ R1 – 2R2 + R3, we get If |A| = 0, then a = – 2 or a = 1. For a = 1, the


system of equations becomes x + y + z = 0 which
1/ a - 2 / b + 1/ c 0 0
fi 1/ b 3 1 =0 For a = – 2, the system of equation becomes
1/ c 4 1
– 2x + y + z = – 3
x – 2y + z = – 3
4.56 Complete Mathematics—JEE Main
x + y – 2z = – 3 Applying R2 Æ R2 – R1, R3 Æ R3 – R1, we get
which on adding becomes 0 = – 9 1 b c
Thus, the system has no solution if a = – 2. D1 = 0 c - b a - c
0 a-b b-c
1 1 1
8. D = 1 1 + x 1 = – (b – c)2 – (a – b)(a – c)
1 1 1+ y = – (a2 + b2 + c2 – bc – ca – ab)
Applying R2 Æ R2 – R1 and R3 Æ R3 – R1, we get 1
fi D1 = - [(b - c)2 + (c - a)2 + (a - b)2 ] < 0
2
1 1 1 As a + b + c > 0, we get
D = 0 x 0 = xy
D = (a + b + c) D1 < 0
0 0 y
which is divisible by both x and y. 11. Using C1 Æ C1 – C3 in D, we get

a +1 b +1 c -1 0 sin a cos a
9. Let D 2 = a -1 b -1 c +1 D= 0 cos a sin a
( -1)n + 2 a ( -1)n +1 b ( -1)n c 2 cos a - sin a - cos a

= 2 cos a (sin2 a – cos2 a)


a + 1 a - 1 ( -1)n + 2 a = –2 cos a cos 2a
D 2 = b + 1 b - 1 ( -1) n +1
b D = 0 for a = p/4 Π(0, p/2).
c -1 c +1 ( -1)n c This is the only value of a lying in (0, p/2) for
which D = 0.
(-1) n+2 a a + 1 a - 1
The system of linear equation will have a non-trivial
= (-1) (-1) n+1 b
2
b +1 b -1 solution if and only if
(-1) n c c -1 c +1 1 sin a cos a
Thus, D1 = 1 cos a sin a = 0
n+ 2 1 - sin a - cos a
a a + 1 a - 1 ( -1) a a + 1 a - 1
Using R2 Æ R2 + R1, we get
D = -b b + 1 b - 1 + ( -1)n+1 b b + 1 b - 1
c c -1 c +1 2 0 0
( -1)n c c - 1 c + 1
D1 = 1 cos a sin a = 0
a + ( -1)n + 2 a a +1 a -1 1 - sin a - cos a
= -b + ( -1) n +1
b b +1 b -1 fi 2(– cos2 a + sin2 a) = 0
n fi – 2 cos 2a = 0
c + ( -1) c c -1 c +1
This is true for only one value of a Π(0, p/2)
n is any odd
viz, a = p/4.
integer.
Thus, statement-1 is also true. However statement-2
\ D = 0 if n is any odd integer. is not a correct reason for statement-1.
10. Using C1 Æ C1 + C2 + C3, we get
1+1+1 1+ a + b 1+ a2 + b2
D = (a + b + c) D1
12. D = 1 + a + b 1+ a2 + b2 1 + a3 + b3
where
2 2 3 3
1+ a + b 1+ a + b 1+ a4 + b4
1 b c
D1 = 1 c a
1 a b
Determinants 4.57

1 1 1 1 1 1 we get
=1 a b 1 a b = D12 b1 x + c1 b1 a1
1 a2 b2 1 a2 b2 | A( x ) | = 2 b2 x + c2 b2 a2
b3 + c3 b3 a3
1 1 1
where D1 = 1 a b Applying C1 Æ C1 – xC2, we get
1 a2 b2 c1 b1 a1
Applying C3 Æ C3 – C2 and C2 Æ C2 – C1, we get
| A( x ) | = 2 c2 b2 a2
1 0 0 c3 b3 a3
D1 = 1 a -1 b -a
\ |B(x)| is independent of x.
1 a2 -1 b2 - a2
15. Using R3 Æ R3 – R2 and R2 Æ R2 – R1, we get
1 1
= (a - 1)(b - a )
a +1 b +a a2 b2 c2
= (a – 1) (b – 1) (b – a) D = 2 al + l 2
2 bl + l 2
2cl + l 2
-4al -4bl -4cl
Thus, D = (a – 1)2 (b – 1)2(a – b)2
Take – 4l common from R3, and applying
\ K=1
R2 Æ R2 – 2lR3, we get
13. As the given system of equations has a non-trivial
solutions. a2 b2 c2
3
D = -4l 1 1 1
a - 1 -1 -1
a b c
D = -1 b - 1 -1 = 0
-1 -1 c - 1 a2 b2 c2
2
Write = l (4l ) a b c
a -1 -1 1 -1 -1 1 1 1
D = 0 b - 1 -1 - 1 b - 1 -1
\ k = 4l2
0 -1 c - 1 1 -1 c - 1
n -1 1 n -1
1 0 0 16. As  r = n(n - 1),  (2r - 1) = (n - 1)2
r =1 2 r =1
= a[(b - 1)(c - 1) - 1] - 1 b 0
n -1 3
1 0 c and  (3r - 2) = n(n - 1) - 2(n - 1)
r =1 2
[use C2 Æ C2 + C1, C3 æ C3 + C1] 1
= (n - 1)(3n - 4)
= a (bc – b – c) – bc 2
Thus,
As D = 0, we get
ab + bc + ca = abc n(n - 1)/ 2 (n - 1)2 (n - 1)(3n - 4)/ 2
n -1
 Dr = n / 2 n -1 a
14. We have |B(x)| = |(A(x))TA(x)| r =1
n(n - 1)/ 2 (n - 1)2 (n - 1)(3n + 4)/ 2
T
= |A(x) | |A(x)|
Using R3 Æ R3 – R1, we get
= |A(x)| |A(x)| = |A(x)|2
n(n - 1)/ 2 (n - 1)2 (n - 1)(3n - 4)/ 2
Let pi(x) = ai x2 + bi x + ci (i = 1, 2, 3) n -1
 Dr = n / 2 n -1 a
1 2 r =1
Applying C1 Æ C1 - x C3 , C2 Æ C2 - xC3 . 0 0 4(n - 1)
2
4.58 Complete Mathematics—JEE Main
Expanding along R3, we get 2 2 -1
n -1 n(n - 1)/ 2 (n - 1) 2 4 3 0 = a - 12
 D r = 4(n - 1) 6 1 1
r =1 n/2 n -1
fi 12 = a – 12
Ê nˆ n -1 n -1 fi a = 24
= 4(n - 1) Á ˜ (n - 1) =0
Ë 2¯ 1 1
19. Let x be any real number and y = 1, z = 1, then
17. The system will have a non-trivial solution if x 1 1
D= 1 1 1 =0≥0
2-l -2 1
1 1 1
D= 2 -3 - l 2 =0
-1 2 -l and xyz = x can take any real value, thus, minimum
value of xyz does not exist.
Using R1 Æ R1 + R3 , R2 Æ R2 + 2R3,
1- l 0 1- l 20. The given system of equations has a non-trivial
solution if
D= 0 1 - l 2 - 2l = 0
1 l -1
-1 2 -l
l -1 -1 = 0
1 0 1 1 1 -l
fi (1 - l ) 2 0 1 2 = 0
fi l – l – l – 1 + 1 + l3 = 0
-1 2 - l
fi l (l – 1) (l + 1) = 0
Using C3 Æ C3 – C1, we get
fi l = 0, –1, 1
1 0 0
When l = 0, x = –y = z = k 0
(1 - l ) 2 0 1 2 =0
When l = –1, x = 0, y = –z = k 0
-1 2 - l + 1
When l = 1, y = 0, x = –z = k 0
fi (1 – l)2 [ – l + 1 – 4] = 0 fi l = 1, – 3
Thus, there are three values of l, for which the
Thus, the set contains two elements system of equations has a non-trivial solutions.

18. Using R2 Æ R2 – R1 – R3, we get 21. Using C1 Æ C1 + C2 + C3, we get


1 sin x sin x
x2 + x x +1 x-2 D = (2 sin x + cos x) 1 cos x sin x
D= -4 0 0 1 sin x cos x
x2 + 2 x + 3 2 x - 1 2 x - 1
Using C2 Æ C2 – (sin x) C1
Expanding along R2, we get and C3 Æ C3 – (sin x) C1,

x +1 x - 2 we get
D = -( -4) 1 0 0
2x - 1 2x - 1
Using C2 Æ C2 – C1, we get D = (2 sin x + cos x) 1 cos x - sin x 0
1 0 cos x - sin x
x + 1 -3
D=4 = (2 sin x + cos x) (cos x – sin x)2
2x - 1 0
1
= 4(3) (2x – 1) \ D = 0 fi tan x = – , 1
2
= 24x – 12 As –p/4 £ x £ p/4, –1 £ tan x £ 1
\ a = 24 and tan x is one-to-one in the interval [– p/4, p/4].
TIP : Put x = 1 to obtain Thus, there are two values of x.
Determinants 4.59
Previous Years’ B-Architecture Entrance Thus, the system has non-trivial solution for no
Examination Questions value of l.

a b c
2
(b – 1)x = 0 3. Â D k = a b g
k =1
If b π 1, x = 0. 3n - 1 4 n - 1 5n - 1
n
In this case y + z = 0, by + z = 0 where a = Â 2(3k -1 )
fi (b – 1)y = 0 fi y = 0 k =1

Thus, z = 0 2(3n - 1)
= = 3n - 1,
That is, if b π 1, then the system has only trivial 3 -1
solution. n 3(4 n - 1)
\ b=1 b = Â 3(4 k -1 ) = = 4n - 1
k =1 4 -1
n 4(5n - 1)
(a – 1) x = 0. and c = Â 4(5k -1 ) = = 5n - 1
k =1 5 -1
If a π 1, x = 0 and y + z = 0 a b c
n
- Thus, Â D k = a b g =0
tion. k =1
a b c
If a = 1, all the three equations become identical to
a11 2 -1 a12 2 -2 a13
x + y + z
of solutions. 4. det(B) = 2a21 a22 2 -1 a23
2
b = 1 fi (b – 1)2 = 0 fi b2 = 2b – 1 2 a31 2a32 a33

2. As the system has a non-trivial solution, 22 a11 2a12 a13


l +3 l +2 1 = (2 -2 )(2 -1 ) 22 a21 2a22 a23
D= 3 l +3 1 =0 22 a31 2a32 a33
2 3 1
a11 a12 a13
Using R1 Æ R1 – R3, R2 Æ R2 – R3, we get = (2 -3 )(22 )(2) a21 a22 a23
l +1 l -1 0 a31 a32 a33
D= 1 l 0 =0 = det (A) = 5
2 3 1
a b c
n
fi D = l(l + 1) – (l – 1) = 0
5. Â D r = a b g
r =1
fi (l – 1)2 = 0 fi l = 1 2 n - 1 3n - 1 5n - 1
For l = 1, the system becomes n
where a = Â 2r -1 = 2 n - 1
4x + 3y + z = 0 (i) r =1

3x + 4y + z = 0 (ii) n 2(3n - 1)
2x + 3y + z = 0 (iii) b = Â 2(3r -1 ) =
r =1 3 -1
From (i) and (iii) x = 0 = 3n – 1
From (i) and (ii) 3y + z = 0 n 4(5n - 1)
and c = Â 4(5r -1 ) =
and 4y + z = 0 r =1 5 -1
fi y = 0, z = 0 = 5n – 1
4.60 Complete Mathematics—JEE Main

a b c
abc ÊÁ - + ˆ˜ ( -1) = 0
1 2 1
n fi
Thus, Â Dr = a b g = 0 Ë a b c¯
r =1
a b c fi 2ac = ab + bc
6. Taking e common from R1, ex4 common from R2
x1
9. Using R1 Æ R1 – R2, R3 Æ R3 – R2, we get
and ex7 from R3, we get
0 a-c b-a
D = e x 1 + x 4 + x 7D 1
1 c a =0
where 0 b-c c-a
1 e3 d e6 d fi (a – c)2 + (b – a) (b – c) = 0
D1 = 1 e3d e6 d = 0 fi a2 – 2ac + c2 + b2 – ab – bc + ac = 0
3d 6d
1 e e
fi a2 + b2 + c2 – bc – ca – ab = 0

where d is common difference of A.P. fi (a – b)2 + (b – c)2 + (c – a)2 = 0

\D=0 fi a=b=c fi A = B = C

7. Taking 5 common from C2 and C3, we get fi sin A = sin B = sin C = 3 2

D = 5D1 Thus, sin A + sin B + sin C = 9/4


2 2 2

x = 2.
13 + 3 2 1
where D1 = 15 + 26 5 2 \ y – z = 1 and – 4y + 4z = a – 2

3 + 65 3 5 fi 4y – 4z + (– 4y + 4z) = 4 + (a – 2)

Applying, C1 Æ C1 - 3C2 - 13C3 , we get fi 0=a+2


Thus, the system has no solution for a π –2.
- 3 2 1
If a
D1 = 0 5 2
number of solutions, since y – z
0 3 5 number of solutions.

= - 3 (5 - 6 ) 11. Using C1 Æ C1 + C3, we get

Thus, D = 5 3 ( 6 - 5) 1 tan q + sec2 q 3


D= 0 cos q sin q
8. As the given system has a non-zero solution
0 -4 3
1 2a a
1 3b b = 0 = 3 cos q + 4 sin q
dD
1 4c c = – 3 cos q + 4 sin q
dq
1/ a 2 1 dD sin q cos q 1
= 0 fi = =
fi abc 1/ b 3 1 = 0 dq 4 3 5
1/ c 4 1 fi sin q = 4/5, cos q = 3/5
Max D = max {D(0), D(sin–1 (4/5)), D(1)}
Using R1 Æ R1 – 2R2 + R3, we get
Ï 24 ¸ 24
= max Ì3, , 4˝ =
1 2 1 Ó 5 ˛ 5
- + 0 0
a b c min D = 3.
abc 1/ b 3 1 =0 \ D Œ[3, 5]
1/ c 4 1
CHAPTER FIVE
Matrices

THE ALGEBRA OF MATRICES Addition of Matrices Two matrices A and B can be added
if and only if they are of the same size. If two matrices are
A matrix is a rectangular array of numbers. The numbers of the same size, addition is carried out term by term. For
may be real or complex. It may be represented as instance
È a b c ˘ È p q r ˘ Èa + p b + q c + r ˘
È a11 a12 ! a1n ˘ Íd e f ˙ + Í s t u ˙ = Í d + s e + t f + u ˙
Ía a22 ! a2 n ˙ Î ˚ Î ˚ Î ˚
A = Í 21 ˙
Addition is not defined for matrices of different sizes.
Í " ˙
Í ˙ The additive inverse of a matrix A, denoted by – A, is the
a
Î m1 a m2 ! a mn ˚ matrix whose elements are the negatives of the correspond-
or as A = (aij)m ¥ n. ing elements of A. For example,
A matrix with m rows and n columns is called as m ¥ n Èa b˘ È- a -b ˘
matrix and the size (or dimension) of this matrix is said be
– ÍÍ c d ˙˙ = ÍÍ -c - d ˙˙
m ¥ n.
Two matrices are said to be equal provided they are of ÍÎ e f ˙˚ ÍÎ -e - f ˙˚
the same size and corresponding elements are equal. For If A and B are two matrices of the same size, then the
example differences between A and B is defined by
Èa b c ˘ È-1 2 5 ˘ A – B = A + (– B)
Íd e f ˙ = Í 7 3 11˙
Î ˚ Î ˚ Thus, the subtraction is carried out term-by-term. For
if and only if instance
a = – 1, b = 2, c = 5, d = 7, e = 3 and f = 11. È a b c ˘ È p q r ˘ Èa - p b - q c - r ˘
Definitions A matrix A = (aij ) m ¥ n is said to be a Íd e f ˙ – Í s t u ˙ = Í d - s e - t f - u ˙
Î ˚ Î ˚ Î ˚
(i) square matrix if m = n
(ii) row matrix if m = 1 Properties of Addition If A, B and C are three matrices of
(iii) column matrix if n = 1 the same size, then
(iv) null or zero matrix if aij = 0 " i and j A+B=B+A [Commutative law]
(v) diagonal matrix if m = n and aij = 0 (A + B ) + C = A + (B + C) [Associative law]
"iπj A+O=O+A [Additive property of zero]
(vi) Scalar matrix if m = n and aij = 0
" i π j and A + ( – A) = O
aii = l "i where O is the null or zero matrix of the same size as that
(vii) Unit or identity matrix if m = n and aij = 0 of A.
" i π j and Scalar Multiplication If A is a matrix and a is a scalar,
aii = 1 " i then aA is defined as the matrix obtained by multiplying
(viii) Upper triangular matrix if m = n and aij = 0 every element of A by a. For example
"i>j
È1 - 2 5˘ È 3 - 6 15˘
(ix) Lower triangular matrix if m = n and aij = 0 3 Í ˙ = Í21 24 3 ˙
"i<j Î7 8 1 ˚ Î ˚
A matrix is said to be triangular if it is either lower or Properties of Scalar Multiplication If A and B are two
upper triangular matrix. matrices of the same size, and a, b are two scalars, then
5.2 Complete Mathematics—JEE Main
(a + b)A = aA + bA ADJOINT AND INVERSE OF A MATRIX
(ab )A = a(bA)
Let A = (aij)n ¥ n be square matrix. The adjoint of A is defined
a(A + B) = aA + aB
to be the matrix adj A = (bij)n ¥ n where bij = Aji
Matrix Multiplication Let A = (aij)m ¥ n and B = (bij)r ¥ s be where Aji is the cofactor of (j, i)th element of A.
two matrices. We say that A and B are comparable for the
product AB if n = r, that is, if the number of columns of A Properties of Adjoint
is equal to the number of rows of B. 1. A(adj A) = (adj A)A = |A|In
Definition: Let A = (aij)m ¥ n and B = (bij)n ¥ p be two matrices. 2. adj(kA) = kn–1 (adj A)
Their product AB is the matrix C = (cij)m ¥ p such that cij = ai1 b1j 3. adj(AB) = (adj B) (adj A)
+ ai2 b2j + ai3 b3j + ... + ain bnj for l £ i £ m, 1 £ j £ p. Note that Definition: A square matrix A is said to be singular if
cij, the (i, j)th element of AB, has been obtained by multiplying |A| = 0 and non-singular if |A| π 0.
ith row of A, namely (ai1 ai2 ai3 º ain) Definition: Inverse of a square matrix A = (aij)n ¥ n is the
matrix B = (bij)n ¥ n such that
Ê b1 j ˆ
Áb ˜ AB = BA = In.
Á 2j˜ 1
Áb ˜ In fact A–1 = (adj A) if |A| π 0.
Á 3j ˜ A
with the jth column of B, namely Á . ˜ = (b1j b2j…bnj)¢ Properties of Inverse
Á ˜ Let A and B be two invertible matrices of the same size.
Á . ˜
Á . ˜ 1. Inverse of a matrix if it exists is unique.
Á ˜ 2. AA–1 = A–1A = In
Ë bnj ¯
3. (A–1)–1 = A
where A¢ denotes the transpose of matrix A.
4. (kA)–1 = k–1 A–1 if k π 0
Properties of Matrix Multiplication 5. (AB)–1 = B –1 A–1 [Reversal law]
If A = (aij)m ¥ n, B = (bij)n ¥ p and C = (cij)p ¥ q then
Ê a bˆ Ê d - bˆ
6. For a matrix A = Á , adj A = Á
1. (AB)C = A(BC) [Associative law] Ë c d ˜¯ Ë - c a ˜¯
2. AIn = ImA = A
3. AB may not be equal to BA and A–1 = 1 Ê d - bˆ if ad – bc π 0
4. k(AB) = (kA)B = A(kB) where k is a scalar ad - bc ÁË - c a ˜¯
5. If A is a square matrix, then 7. If A is a triangular matrix, then A–1, if it exists is
Am An = Am + n " m, n ΠN a triangular matrix of the same kind.
(Am)n = Amn " m, n Œ N. Ê a11 0 0ˆ
6. If A is an invertible matrix then In fact if A = Á a21 a22 0 ˜ and a11a22a33 π 0,
Á ˜
(A–1 B A)m = A–1 B m A Ë a31 a32 a33 ¯
and A–m = (A–1)m " m Œ N. then
Ê a22 a33 0 0 ˆ
TRANSPOSE OF A MATRIX A–1 =
1 Á -a a a a 0 ˜
21 33 33 11
Definition: Let A = (aij)m ¥ n be a matrix. The transpose of a11a22 a33 Á ˜
Ë A13 - a32 a11 a11a22 ¯
A, denoted by A¢ or by At is the matrix A¢ = (bij)n ¥ m where
bij = aji " i and j. where A13 = cofactor of (1, 3)th element in A i.e.
By A we mean a matrix B = (bij)m ¥ n where bij = a ij where a21 a22
A13 =
a denotes the conjugate of a. By A* we mean a31 a32
A* = ( A )¢ = ( A ¢ ) 8. If A = diag (l1, l2, …, ln) then A–1 exists if and
Properties of Transpose of Matrix only if li π 0 " i and
1. (A + B)¢ = A¢ + B¢ A–1 = diag (l1–1, l2–1, …, ln–1)
Also, Am= diag (l1m, l2m…lmn) if m Œ N
2. (kA)¢ = kA¢ where k is a scalar
9. If a square matrix A satisfies the equation
3. (AB)¢ = B¢A¢ [Reversal law]
a0 + a1x + a2 x2 + … + ar xr = 0, then A is invertible
4. If A is an invertible matrix, then (A–1)¢ = (A¢)–1. if a0 π 0 and its inverse is given by
Matrices 5.3

1 In fact
A–1= – [a1 I + a2 A + … + ar Ar–1]
a0 1 1
P= (A + A¢ ) and Q = (A – A¢ ).
10. If AB = I, then BA = I and B = A –1 2 2
4. For every square matrix A, matrices A + A¢, AA¢ and
11. If AB = CA = I, then B = C.
A¢A are symmetric and matrices A – A¢ and A¢ – A
are skew-symmetric.
SOME DEFINITIONS AND RESULTS
Results 1. If A is an orthogonal matrix, then |A| π 0. In fact
A square matrix A is said to be |A| = ± 1 and A , A¢, A* are also orthogonal.
1. Symmetric if A¢ = A 2. If A and B are two orthogonal matrices, then AB
2. Skew-symmetric if A¢= – A and BA are both orthogonal matrices.
3. Hermitian if A* = A A square matrix A is said to be
4. Skew Hermitian if A* = РA 1 nilpotent if Am = O for some m ΠN.
5. Orthogonal if AA¢ = A¢A = I or if A¢ = A–1 2. idempotent if A2 = A
6. Unitary if AAq = AqA = I 3. involutory if A2 = I.

Illustration 1 Illustration 2
Ê 2 5 -1ˆ Ê 1 -1ˆ
1. A = Á 5 7 -2˜ is a symmetric matrix 1. A = Á is a nilpotent matrix as A2 = 0
Á ˜ Ë 1 -1˜¯
Ë -1 -2 0 ¯
Ê 1 -1 1ˆ
Ê 0 5 2ˆ
2. A = Á 1 -1 1˜ is an idempotent matrix as A2 = A
2. A = Á -5 0 7˜ is a skew symmetric matrix Á ˜
Ë 1 -1 1¯
Á ˜
Ë -2 -7 0¯
Ê1 / 2 1/ 2 ˆ 2
Ê 3 2 - iˆ 3. A = Á ˜ is an involutory as A = I.
3. A = Á is a Hermitian matrix Ë1 / 2 -1 / 2 ¯
Ë2 + i 7 ˜¯
Ê 2i 6 - 5iˆ
4. A = Á is a skew Hermitian matrix Remark
Ë -6 - 5i 4i ˜¯
The least value of m for which Am = O, is called the index of
Ê cos q - sin q ˆ
the nilpotent matrix.
5. A = Á is an orthogonal matrix
Ë sin q cos q ˜¯
1 Ê 2i 1 ˆ Definition: If A = (aij) n × n is a square matrix, the trace
6. A = Á ˜ is a unitary matrix. of A denoted by tr (A) is sum of all the main diagonal ele-
5 Ë 1 2i ¯ ments, i.e.
tr (A) = a11 + a22 + … + ann
TIP Properties of Trace
If A is a skew symmetric matrix, the I + A and I – A are non- Let A, B be two n × n matrices, and k Œ R.
singular matrices. 1. tr (kA) = k tr (A)
2. tr (A + B) = tr (A) + tr (B)
Some Results 3. tr (AB) = tr (BA)
1. The main diagonal elements of a skew-symmetric
matrix are zeros, i.e. aii = 0 " i. RANK OF A MATRIX
2. Determinant of a skew-symmetric matrix of odd
order is zero and determinant of a skew-symmetric If A = (aij)m ¥ n is a matrix, and B is its submatrix of order
matrix of even order is a perfect square. r, then |B|, the determinant is called as r-rowed minor of A.
3. Every square matrix can be written uniquely as a Definition Let A = (aij)m ¥ n be a matrix. A positive integer
sum of symmetric and a skew-symmetric matrix, i.e. r is said to be rank of A if
if A is a square matrix, then there exists a symmetric (i) A possesses at least one r-rowed minor which is
matrix P and a skew-symmetric matrix Q such that different from zero; and
A=P+Q (ii) every (r + 1) rowed minor of A is zero.
5.4 Complete Mathematics—JEE Main
From (ii), it automatically follows that all minors Step 2: We must get a 1 in the upper left corner. For this
of higher order are zeros. we interchange Row 1 and Row 2.
We denote rank of A by r(A)
Ê 2 -3 1ˆ Ê 1 8 6ˆ
Result The rank of a matrix does not change when the fol-
Á 1 8 6˜ [R1 ´ R2] Á 2 -3 1˜
lowing elementary row operations are applied to the matrix. Á ˜ Á ˜
(a) Two rows are interchanged (Ri ´ Rj) Ë 5 3 8¯ Ë 5 3 8¯
(b) A row is multiplied by a non-zero constant, Step 3: We must get zeros at the two remaining two places
(Ri Æ kRi, with k π 0) in the first column. For this we multiply R1 by – 2 and add
(c) a constant multiple of another row is added to a it to R2 and multiply R1 by – 5 and add it to R3.
given row (Ri Æ Ri + kRj) where i π j
Note : The arrow Æ means “replaced by”. Ê 1 8 6ˆ
Á 2 -3 1˜ [R2 Æ R2 – 2R1,
Note that the application of these elementary row operations Á ˜
does not change a singular matrix to a non-singular matrix Ë 5 3 8¯
or a non-singular matrix to a singular matrix. Therefore,
the order of the largest non-singular square submatrix is Ê1 8 6 ˆ
not affected by application of any of the elementary row R3 Æ R3 – 5R1] Á 0 -19 -11˜
Á ˜
operations. Thus, the rank of a matrix does not change by Ë 0 -37 -22¯
application of any of the elementary row operations.
Step 4: We must have 1 in the second column. This 1
A matrix obtained from a given matrix by applying any of should not be in the first row. Also, you should not be tempt-
the elementary row operations is said to be equivalent to ed to use R1 to obtain this 1. For, if we try to use R1, then
it. If A and B are two equivalent matrices, we write A ~ B. two zeros obtained in the first column will be destroyed. We
Note that if A ~ B, then r(A) = r (B). multiply R2 by – 2 and add it to R3.
By using the elementary row operations, we shall try
to transform the given matrix in the following form Ê1 8 6 ˆ Ê1 8 6 ˆ
Á 0 -19 -11˜ [R3 Æ R3 – 2R1] Á 0 -19 -11˜
Ê 1 * * *ˆ Á ˜ Á ˜
Á 0 1 * *˜ Ë 0 -37 -22¯ Ë0 1 0 ¯
Á ˜
Á 0 0 1 *˜ Step 5: We now obtain a 1 at the (2, 2)th place. For this we
Á. . . .˜ interchange R2 and R3.
Á ˜
Á. . . .˜
Á ˜ Ê1 8 6 ˆ Ê1 8 6 ˆ
Á. . . .˜ Á 0 -19 -11˜ [R2 ´ R3] Á0 1 0 ˜
ÁË 0 0 0! *˜¯ Á ˜ Á ˜
Ë0 1 0 ¯ Ë 0 -19 -11¯
where * stands for zero or non-zero element. That is, we shall
try to make aii as 1 and all the elements below aii as zero. Step 6: We must get a zero at (3, 2)th place. For this we
We illustrate the above procedure by the following multiply R2 by 19 and add it to R3.
illustration.
Ê1 8 6 ˆ Ê1 8 6 ˆ
Á0 1 0 ˜ [R3 Æ R3 + 19R2] Á0 1 0 ˜
Illustration 3 Á ˜ Á ˜
Ë 0 -19 -11¯ Ë 0 0 -11¯
Find the rank of the matrix
This matrix is in the desired triangular form. Recall if A ~ B
Ê 2 -3 1ˆ then r(A) = r(B). Thus, rank of the given matrix A is equal
A = Á 3 5 7˜ to the rank of the matrix
Á ˜
Ë 5 3 8¯
Ê1 8 6 ˆ 1 8 6
Solution
Step 1: As a first step we must get a 1 in the first column B = Á 0 1 0 ˜ . Since |B| = 0 1 0
Á ˜
of A. For this we subtract Row 1 from the Row 2. Ë 0 0 -11¯ 0 0 -11
Ê 2 -3 1ˆ Ê 2 -3 1ˆ = –11 π 0, r(B) = 3.
Á 3 5 7˜ [R2 Æ R2 – R1] Á 1 8 6˜
Á ˜ Á ˜ Hence, r(A) = 3.
Ë 5 3 8¯ Ë 5 3 8¯
Matrices 5.5

Ê x1 ˆ Ê b1 ˆ
Some Tips
Á x2 ˜ Á b2 ˜
After obtaining 1 at (1, 1)th place and zeros at the remaining
Á ˜ Á ˜
Á . ˜ Á . ˜
places in the first column, forget the first row. Do not use the X=Á and B =
. ˜ Á . ˜
first row to manipulate elements in the second or any other Á ˜ Á ˜
column. If you try to do so the zeros in the first column will Á . ˜ Á . ˜
be destroyed. Á ˜ Á ˜
Ë xn ¯ Ë bn ¯
After obtaining 1 at (2, 2)th place and zeros at the remaining
places in the second column, forget the second row. Do not By a solution of (1) we mean a set of values x1, x2, ..., xn
use it for manipulating elements in the remaining columns. such that (1) reduces to an identity.
The same remark applies to the remaining columns.
If A is a non-singular matrix of order n ¥ n, then r (A) = n. The augmented matrix for system of equations
AX = B
SYSTEM OF LINEAR EQUATIONS is the matrix (A | B). This matrix is obtained by adding
(n + 1)th column to A. The elements of this column are the
Let us consider the following m linear equations in n un- constants b1, …, bm.
knowns:
Result The system of equations
a11 x1 + a12 x2 + ... + a1n xn = b1
a11 x1 + a12 x2 + … + a1n xn = b1
a21 x1 + a22 x2 + ... + a2n xn = b2 …(1)
a21 x1 + a22 x2 + … + a2n xn = b2 …(1)
.
…………………………………
.
. am1 x1 + am2 x2 + … + amn xn = bm
am1 x1 + am2 x2 + ... + amn xn = bm is consistent (that is, possesses a solution) if and only if the
where b1, b2, ... bm are not all zero. coefficient matrix
Ê a11 a12 … a1n ˆ
Ê a11 a12 … a1n ˆ
Á a21 a22 … a2 n ˜
Áa a22 … a2 n ˜ A=Á ˜
Á 21 ˜ Á … … … … ˜
Á . . . ˜ ÁË
The m ¥ n matrix Á is called the am1 am 2 … amn ˜¯
. . . . ˜
Á ˜
Á . . . ˜ and the augmented matrix
Á ˜ Ê a11 a12 … a1n b1 ˆ
Ë am1 am 2 … amn ¯
Á a21 a22 … a2 n b2 ˜
coefficient matrix of the system of linear equations. Using (A | B) = Á ˜
it, we can now write these equations as follows: Á … … … … …˜
ÁË am1 am 2 … amn bm ˜¯
Ê a11 a12 … a1n ˆ Ê x1 ˆ Ê b1 ˆ
Áa a22 … a2 n ˜ Á x2 ˜ Á b2 ˜ have the same rank.
Á 21 ˜ Á ˜ Á ˜
Á . . . ˜ Á .˜ Á . ˜ We split the remaining result in two cases.
=
Á . . . . ˜ Á .˜ Á . ˜
Á ˜ Á ˜ Á ˜ Case 1. If the system of equations in (1) is consistent and
Á . . . ˜ Á .˜ Á . ˜ m ≥ n, then
Á ˜ Á ˜ Á ˜
Ë am1 am 2 … amn ¯ Ë xn ¯ Ë bm ¯ (i) if r(A) = r(A | B) = n, then the system of equations has a
We can abbreviate the above matrix equation as AX = B, unique solution and (m – n) equations are superfluous in the
where sense that any solution that satisfies other n equations will
Ê a11 a12 … a1n ˆ satisfy these (m – n) equations.
Áa a22 … a2 n ˜ (ii) if r(A) = r(A | B) = r < n, then the (n – r) unknowns are
Á 21 ˜ assigned arbitrary values and the remaining r unknowns can
Á . . . ˜
A=Á , be found in terms of those (n – r) unknowns which have
. . . . ˜
Á ˜ already been assigned values.
Á . . . ˜
Á ˜ Case 2. If the system of equations in (1) is consistent and
Ë am1 am 2 … amn ¯ m < n, then
5.6 Complete Mathematics—JEE Main
(i) if r(A) = r(A | B) = m, then (n – m) unknowns can be
assigned arbitrary values and the values of the remain-
Illustration 4
ing m unknowns can be found in terms of those (n – m) Consider the system of equations
unknowns which have already been assigned values. Also, x + y + z =1
(m – r) equations are superfluous. 2x + 2y + 2z = 2
(ii) if r(A) = r(A | B) = r < m then (n – r) unknowns can be 3x + 3y + 3z = k
assigned arbitrary values and the values of the remaining r Here
unknowns can be found in terms of those (n – r) unknowns Ê 1 1 1ˆ Ê1 ˆ
which have already been assigned values. Also, (m – r) A = Á 2 2 2˜ , B = Á 2˜ , and
Á ˜ Á ˜
equations are superfluous. Ë 3 3 3¯ Ë k¯
Finding Inverse by Elementary Row Operations Ê 0 0 0ˆ
To find inverse of a square matrix A we begin with the adj A = Á 0 0 0˜
Á ˜
augmented matrix [A | In]. If a sequence of elementary row Ë 0 0 0¯
operations transforms this matrix to [In | B], then B is A–1. Note that
However, if at any step we obtain all zeros in a row on the (adj A) B = O
left of the vertical line, the matrix A is not invertible. Also, note that the system of equations has
(1) infinite number of solutions if k = 3
SOLUTION OF A SYSTEM OF EQUATION (2) no solution if k π 3.
AX = B WHEN A IS A SQUARE MATRIX
Unique Solution Remark

The system of equations Note this case carefully. Mostly students commit mistake in
this case.
AX = B
has a unique solution if |A| π 0 and it is given by X = A–1 B SOLUTION OF A SYSTEM OF HOMOGE-
No Solution NEOUS LINEAR EQUATIONS AX = 0
If |A| = 0 and (Adj A)B π O, the system of equations has no The system
solution. AX = 0
has a unique solution if |A| π 0 and it is the trivial solution viz.
Infinite Number of Solution or no Solution x1 = x2 = … = xn = 0.
If |A| = 0, the system has infinite number of solutions.
If |A| = 0, and (Adj A)B = O, the system of equations has Also if AX = 0 has at least one non-zero solution, then | A| = 0.
infinite number of solutions or no solution. The following tree diagram is helpful.

AX = B

B=O B πO

|A| π 0 |A| = 0 r(A) = r (A B) r (A) = r (A B)

Infinite System of System of


Trivial number of equations is equations is
solution solutions consistent inconsistent
Matrices 5.7

SOLVED EXAMPLES
Concept-based
Straight Objective Type Questions

Èa b ˘ Example 3: Let A and B be two 3 × 3 invertible matrices.


Example 1: The number of 2 × 2 matrices A = Í ˙ If A + B = AB, then
for which Îc d ˚
(a) A–1 + B –1 = O
-1
Èa b ˘ È1 / a 1 / b ˘ (b) A–1 + B –1 = B –1 A–1
Í c d ˙ = Í1 / c 1 / d ˙ , (a, b, c, d Œ R) is (c) I – A–1 is invertible
Î ˚ Î ˚
(a) 0 (b) 1 (d) B –1 + I is invertible
(c) 2 (d) infinite Ans. (c)
Ans. (a) Solution: A + B = AB fi A = AB – B = (A – I)B.
Solution: If ad – bc π 0, then Premultiplying by A–1 we get
I = A–1A = (I – A–1)B
1 È d -b˘
A–1 = –1
fi I – A is invertible.
ad - bc ÍÎ-c a ˙˚
Example 4: Let A be a 3 × 3 matrix and
d 1
Thus, = ¤ ad = ad – bc ÏÊ xˆ ¸
ad - bc a Ô Ô
S = ÌÁ y˜ x, y, z Œ R ˝
¤ bc = 0 ¤ b = 0 or c = 0 Á ˜
ÔË z ¯ Ô
Therefore, Ó ˛
Èa b ˘
-1
È1 / a 1 / b ˘ Define f : S Æ S by
Í c d ˙ = Í1 / c 1 / d ˙ Ê xˆ Ê xˆ
Î ˚ Î ˚
is never possible. f Á y ˜ = A Á y˜
Á ˜ Á ˜
Ë z¯ Ë z¯
TIP Ê x ˆ Ê 0ˆ
If n > 1, then a n × n matrix cannot have inverse in the above Suppose f Á y˜ = Á 0˜ fi x = y = z = 0.
Á ˜ Á ˜
form. Ë z ¯ Ë 0¯
Then
Èa b ˘ (a) f is one-to-one (b) f cannot be onto
Example 2: Let A = Í ˙ , a, b, c, d Œ R (c) A is not invertible (d) A = O
Îc d ˚
5 3 Ans. (a)
If A = A + I, then A is
(a) a symmetric matrix Ê x1 ˆ Ê x2 ˆ
(b) a skew symmetric matrix Solution: Let Á y1 ˜ and Á y2 ˜ be such
Á ˜ Á ˜
(c) an invertible matrix Ë z1 ¯ Ë z2 ¯
(d) none of these Ê x1 ˆ Ê x2 ˆ
Ans. (c) that f Á y1 ˜ = f Á y2 ˜
Solution: A5 = A3 + I Á ˜ Á ˜
Ë z1 ¯ Ë z2 ¯
fi A(A4 – A2) = (A4 – A2) A = I
Ê x1 ˆ Ê x2 ˆ
fi A is invertible and A–1 = A4 – A2.
fi A Á y1 ˜ = A Á y2 ˜
Á ˜ Á ˜
Ë z1 ¯ Ë z2 ¯
TIP
Ê x1 - x2 ˆ Ê 0ˆ
If P(x) is a polynomial such that P(0) π 0 and P(A) = O, then fi A Á y1 - y2 ˜ = Á 0˜
A is invertible. Á ˜ Á ˜
Ë z1 - z2 ¯ Ë 0¯
5.8 Complete Mathematics—JEE Main

Ê x1 - x2 ˆ Ê 0ˆ Example 8: For 1 £ i, j £ 3, let


fi f Á y1 - y2 ˜ = Á 0˜ p /2
Á ˜ Á ˜
Ë z1 - z2 ¯ Ë 0¯
aij = Ú cos(ix )cos( jx ) dx
-p / 2
fi x1 – x2 = 0, y1 – y2 = 0, z1 – z2 = 0 and let A = (aij)3×3. Then
fi x 1 = x 2, y 1 = y 2, z 1 = z 2 (a) A is a singular matrix
Ê x1 ˆ Ê x2 ˆ (b) AX = B has a unique solution for every 3×3 ma-
or Áy ˜ = Áy ˜ trix B
Á 1˜ Á 2˜ (c) A is a skew-symmetric matrix.
Ë z1 ¯ Ë z2 ¯
Thus, f is one-to-one. (d) A2 = I
Ê a bˆ Ans. (b)
Example 5: Let A = Á , where a, b, c, d Œ R.
Ë c d ˜¯ Solution: For 1 £ i £ 3,
p /2
If a , b , c , d £ k, where k > 0, then
(a) det(A) ≥ 2k2 (b) det(A) ≥ k2
aii = 2 Ú cos(ix )cos(ix ) dx
0
(c) det(A) £ 2k2 (d) det(A) £ k [integrand is an even function]
Ans. (c) p /2

Solution: det(A) = ad – bc £ |ad – bc|


= Ú [1 + cos(2ix )] dx
0
fi det (A) £ | a || d | + | b || c | £ 2k2 p /2
Ê sin(2ix ) ˆ ˘ p p
Ê a bˆ = Ëx+ = +0=
Example 6: Let A = Á , where a, b, c, d Œ R. 2i ¯ ˚˙0 2 2
Ë c d ¯˜ For 1 £ i, j £ 3, i π j,
Then p /2
(a) det (A) £ a 2 + b2 c 2 + d 2 aij = 2 Ú cos(ix )cos( jx ) dx
(b) det (A) £ (a + b) (c + d) 0
p /2
(c) det (A) £ ac + bd
(d) det (A) £ (| a | – | b |) (| c | – | d |)
= Ú [cos((i + j ) x ) + cos((i - j ) x )] dx
0
Ans. (a)
sin((i + j )p / 2) sin((i - j )p / 2)
Solution: det(A) = ad – bc £ |ad – bc| = +
i+ j i- j
£ | a || d | + | b || c |
1 2
Now, \ a12 = - + 1 = = a21
3 3
(a2 + b2) (c2 + d2) – (| a || d | + | b || c |)2
a13 = 0 = a31, and
= a 2c 2 + b 2c 2 + a 2d 2 + b 2d 2 1 6
– (a2d2 + b2c2 + 2 | a || d | | b || c |) a23 = + 1 = = a32
5 5
= (| ac | – | bd |)2 ≥ 0 Ê 1 2/3 0 ˆ
fi | a || d | + | b || c | £ 2
a +b 2 2
c +d 2 Thus, A = Á 2 / 3 1 6 / 5˜
Á ˜
Example 7: Let A be a 3×3 matrix such that det(A) = –2. Ë 0 6/5 1 ¯
Then det (–2 A–1) is equal to 2
(a) 4 (b) – 4 Using C2 Æ C2 - C1 , we get
3
(c) 8 (d) – 2
1 0 0
Ans. (a)
det (A) = 2 / 3 5 / 9 6 / 5
Solution: As A–1 is a 3 × 3 matrix,
0 6/5 1
det (–2 A–1) = (–2)3 det (A–1)
5 36 199
= (–2)3 (det(A))–1 = - =- π0
9 25 225
1
= (-8) Ê- ˆ = 4
–1
As det (A) π 0, A exists. Therefore, AX = B has a unique
Ë 2¯ solution viz. X = A–1B.
Matrices 5.9
Example 9: The number of values of l for which there (a) 3 (b) 4
exist a non-zero 3×3 matrix A such that A¢ = lA is: (c) 6 (d) 8
(a) 0 (b) 1 Ans. (b)
(c) 2 (d) infinite È0 a ˘
Ans. (c) Solution: A = I + B, where B = Í ˙
Î0 0 ˚
Solution: We have As 2 r
B = O, we get B = O " r ≥ 2.
A = (A¢)¢ = (lA)¢ = lA¢ = l2A.
Thus, Ar = (I + B)r = I + rB
As A π 0, l2 = 1 fi l = ±1.
For l = 1 all non-zero symmetric matrices can work as A. È1 ra ˘
=Í ˙ " r ≥ 1.
For l = –1, all non-zero skew symmetric matrices can work Î0 1 ˚
as A. Ê 1 ˆ 1 1
fi trace Á r Ar ˜ = r (2) = r -1 " r ≥ 1.
È1 a˘ Ë2 ¯ 2 2
Example 10: Let A = Í ˙ , where a > 0. Sum of the
Î0 1˚ 1 1 2
1 1 1 \ S = 2 +1+ + +! = =4
series S = trace (A) + trace Ê Aˆ + trace ÊÁ A2 ˆ˜ + trace ÊÁ A3 ˆ˜ 22 23 1-
1
Ë2 ¯ Ë 22 ¯ Ë 23 ¯
+ … is 2

LEVEL 1

Straight Objective Type Questions

Ans. (c)
È1 4 ˘ È x y ˘
2
Example 11: If Í ˙ = Í ˙ , y < 0 then x– y + z Solution: We have
Î2 0 ˚ Î z 0 ˚
is equal to
È-i 0 ˘ È-i 0 ˘ È-1 0 ˘
(a) 5 (b) 2 A¢ A = Í ˙Í ˙=Í ˙ =–I
(c) 1 (d) – 3 Î 0 i ˚ Î 0 i ˚ Î 0 -1˚
Ans. (a) È cos a sin a ˘
Example 14: If Aa = Í ˙ , then Aa Ab is
Solution: By the equality of two matrices, x = 1, y2 = 4, equal to Î- sin a cos a ˚
z=2 (a) Aa + b (b) Aab
fi x = 1, y = – 2, z = 2 as y < 0.
(c) Aa - b (d) none of these
\ x – y + z =1 + 2 + 2 = 5
Ans. (a)
È 2˘
Example 12: If A = [1 – 2 3], B = Í-3˙ , then AB is equal Solution: We have
Í ˙
ÍÎ -1˙˚ È cos a sin a ˘ È cos b sin b ˘
to Aa Ab = Í ˙Í ˙
Î- sin a cos a ˚ Î- sin b cos b ˚
È 2˘ È 2˘
È cos a cos b - sin a sin b
(a) Í-3˙ (b) Í 6 ˙ = Í
Í ˙ Í ˙ Î- sin a cos b - cos a sin b
ÍÎ -1˙˚ ÍÎ-3˙˚
(c) [2 6 – 3] (d) none of these cos a sin b + sin a cos b ˘
Ans. (d) - sin a sin b + cos a cos b ˙˚
È 2˘ È cos (a + b ) sin (a + b ) ˘
Solution: AB = [1 -2 3] Í-3˙ = [2 + 6 - 3] = [5] = Í ˙ = Aa + b
Î- sin (a + b ) cos (a + b )˚
Í ˙
ÎÍ -1˚˙ Example 15: Let A and B be two 2 × 2 matrices.
È -i 0 ˘ Consider the statements
Example 13: If A = Í ˙ , then A¢ A is equal to
Î0 i˚ (i) AB = O fi A = O or B = O
(a) I (b) – iA (ii) AB = I2 fi A = B –1
(c) – I (d) iA (iii) (A + B)2 = A2 + 2AB + B2
5.10 Complete Mathematics—JEE Main
Then Ê cos2 q + sin 2 q cos q sin q - sin q cos q ˆ
(a) (i) is false, (ii) and (iii) are true = Á ˜
(b) (i) and (iii) are false, (ii) is true Ë cos q sin q - sin q cos q sin 2 q + cos2 q ¯
(c) (i) and (ii) are false, (iii) is true 1 0ˆ
= ÊÁ = I2
(d) (ii) and (iii) are false, (i) is true Ë 0 1˜¯
Ans. (b)
Similarly, A¢ A = I2
Solution: (i) is false.
Thus, A is an orthogonal matrix.
È0 1˘ È1 1 ˘
If A= Í
0 - 1˙ and B = Í0 0 ˙ , then Example 19: If
Î ˚ Î ˚
Èa 2 ab ac ˘ È0 c -b˘
È0 0 ˘ Í ˙
AB = Í ˙=O A = Íab b2 bc ˙ and B = Í-c 0 a ˙ then the product
Î0 0 ˚ Í ˙
Í 2 ˙ ÍÎ b - a 0 ˙˚
Thus, AB = O fi / A = O or B = O ÍÎ ac bc c ˙˚
(iii) is false since matrix multiplication is not commuta- AB is equal to
tive. (a) O (b) A
(ii) is true as product AB is an identity matrix, if and (c) B (d) I
only if B is inverse of the matrix A. Ans. (a)
È1 5˘ È-2 5˘ Solution: We have
Example 16: If A – 2B = Í ˙ and 2A – 3B = ÍÎ 0 7˙˚ ,
Î3 7˚ È0 - abc + abc a 2 c + 0 - a 2 c - a 2 b + a 2 b + 0 ˘
then matrix B is equal to Í ˙
AB = Í 0 - b2 c + b2 c abc + 0 - abc - ab2 + ab2 + 0 ˙ = O
È- 4 -5˘ È 0 6˘ Í ˙
(a) Í (b) Í 2 2 2 2
Î - 6 -7 ˙
˚ Î-3 7˚
˙ ÍÎ 0 - bc + bc ac + 0 - ac - abc + abc + 0 ˙˚

È2 -1˘ È6 -1˘ Example 20: If A is an invertible matrix and B is an


(c) Í ˙ (d) Í ˙ orthogonal matrix, of the order same as that of A, then C =
Î3 2˚ Î0 1˚
Ans. (a) A –1 BA is
(a) an orthogonal matrix
Solution: We have
(b) symmetric matrix
È - 4 - 5˘ (c) skew-symmetric matrix
B = (2 A - 3 B ) - 2 ( A - 2 B ) = Í ˙
Î - 6 - 7˚ (d) none of these
Example 17: If A and B two are 3 × 3 matrices, then Ans. (d)
which one of the following is not true: Ê cos (p / 2) sin (p / 2) ˆ Ê 0 1ˆ
Solution: Let B = Á =
(a) (A + B) ¢= A ¢ + B ¢ Ë - sin (p / 2) cos (p / 2)¯˜ ËÁ -1 0¯˜
(b) (AB)¢ = A¢ B¢
Ê 1 3ˆ Ê 1 -3ˆ
(c) det (AB) = det (A) det (B) and A= Á , A-1 = Á
Ë 0 1˜¯ Ë 0 1 ˜¯
(d) A (adj A) = |A| I3
Ans. (b) Note that B is an orthogonal matrix.
Solution: If A and B are two 3 × 3 matrices, then Ê 1 -3ˆ Ê 0 1ˆ Ê 1 3ˆ
C = A–1 BA = Á
(AB)¢ = B¢A¢ [Reversal Law] Ë 0 1 ˜¯ ÁË -1 0˜¯ ÁË 0 1˜¯
and not (AB)¢ = A¢ B¢. Ê 3 10 ˆ

Ê cos q - sin q ˆ Ë -1 -3˜¯
Example 18: If A = ÁË
sin q cos q ˜¯
, then
Note that C is neither symmetric, nor skew-symmetric and
(a) A is an orthogonal matrix nor-orthogonal.
(b) A is a symmetric matrix È cos2 a cos a sin a ˘
(c) A is a skew-symmetric matrix Example 21: Let E(a) = Í ˙ . If
(d) none of these ÍÎcos a sin a sin 2 a ˙˚
Ans. (a) a and b differs by an odd multiple of p/2, then E(a) E(b) is a
Solution: We have (a) null matrix
(b) unit matrix
Ê cos q - sin q ˆ Ê cos q sin q ˆ (c) diagonal matrix
AA¢ = Á
Ë sin q cos q ˜¯ ÁË - sin q cos q ˜¯ (d) orthogonal matrix
Matrices 5.11
Ans. (a) Ans. (c)
Solution: We have Solution: If AB = C, then B –1 A –1 = C –1
E(a) E(b) fi A –1 = BC –1
È cos2 a cos a sin a ˘ È cos2 b cos b sin b ˘ È1 1 ˘ È3 2 ˘
= Í Here AÍ ˙ = Í ˙
˙ Í ˙ Î2 0 ˚ Î1 1 ˚
ÍÎcos a sin a 2
sin a ˚ Î ˙ Í cos b sin b sin 2 b ˙˚
cos a cos b cos (a - b ) cos a sin b cos (a - b )˘ È1 1 ˘ È3 2 ˘ -1
= È fi A –1 = Í
0 ˙˚ ÍÎ1 1 ˙˚
Í sin a cos b cos (a - b ) sin a sin b cos (a - b ) ˙ Î2
Î ˚
As a and b differ by an odd multiple of p/2, a –b = È1 1 ˘ È 1 -2 ˘ È0 1 ˘
= Í =
(2n + 1)p/2 for some integer n. Thus, cos [(2n + 1) p/2] = 0 Î2 0 ˙˚ ÍÎ-1 3 ˙˚ ÍÎ2 - 4 ˙˚
\ E(a) E(b) = O Example 25: If A and B are two skew-symmetric
matrices of order n, then
È2 1 ˘ È-3 2 ˘ È1 0 ˘
(a) AB is a skew-symmetric matrix
Example 22: If Í ˙AÍ = , then
Î7 4 ˚ Î 5 - 3˙˚ ÍÎ0 1 ˙˚ (b) AB is a symmetric matrix
matrix A equals (c) AB is a symmetric matrix if A and B commute
È 7 5˘ È2 1˘ (d) none of these
(a) Í ˙ (b) Í5
Î-11 - 8˚ Î 3˙˚ Ans. (c)
È 7 1˘ È5 3˘ Solution: We are given
(c) Í ˙ (d) Í
8˙˚
A¢ = – A and B¢ = –B
Î34 5˚ Î13
Now, (AB)¢ = B¢ A¢ = (– B) (– A) = BA
Ans. (a) = AB if A and B commute.
Solution: If XAY = I, then A = X –1 Y –1 = (YX)–1 Example 26: Which of the following statements is false:
È-3 2 ˘ È2 1 ˘ È 8 5˘ (a) If |A| = 0, then |adj A| = 0
In this case YX = Í ˙ Í ˙ =Í ˙
Î 5 -3˚ Î7 4 ˚ Î-11 -7˚ (b) Adjoint of a diagonal matrix of order 3 × 3 is a
diagonal matrix
È 8 5 ˘ -1 È 7 5˘
\ (c) Product of two upper triangular matrices is a
A= Í ˙ = Í-11 -8˙
Î-11 -7˚ Î ˚ upper triangular matrix
Example 23: The matrix A satisfying (d) adj (AB) = adj (A) adj (B)
È1 5˘ È3 -1˘ Ans. (d)
AÍ ˙ =Í ˙ is
Î0 1˚ Î6 0 ˚ Solution: We have
È3 2 ˘ È3 -16 ˘ adj (AB) = adj (B) adj (A)
(a) Í ˙ (b) Í ˙ and not adj (AB) = adj (A) adj (B)
Î6 -3˚ Î6 -30 ˚
Example 27: If A and B are symmetric matrices, then
È3 -16 ˘ È3 -3˘
(c) Í ˙ (d) Í ˙ AB – BA is a
Î6 30 ˚ Î6 2 ˚ (a) symmetric matrix
Ans. (b)
(b) skew-symmetric matrix
Solution: We know that if AC = B, then A = BC –1. (c) diagonal matrix
-1
È3 -1˘ È1 5˘ È3 -1˘ È1 -5˘ (d) null matrix
\ A= Í ˙ Í ˙ = Í ˙Í ˙ Ans. (b)
Î6 0 ˚Î 0 1 ˚ Î6 0 ˚ Î0 1 ˚
Solution: We are given A¢ = A, B¢ = B
È3 -16 ˘
=Í ˙ Now (AB – BA)¢ = (AB)¢ – (BA)¢ = B¢ A¢ – A¢ B¢
Î6 -30 ˚ = BA – AB = – (AB – BA)
È1 1 ˘ È3 2 ˘ i.e. (AB – BA)¢ = – (AB – BA)
Example 24: If product A Í ˙ is the matrix Í ˙,
Î2 0 ˚ Î1 1 ˚ Hence, AB – BA is a skew-symmetric matrix.
then A–1 is given by
Example 28: Let A and B be two 3 × 3 matrices, such
È0 -1˘ È 0 -1˘ that A + B = 2B¢ and 3A + 2B = I, then
(a) Í ˙ (b) Í ˙
Î2 - 4˚ Î-2 - 4˚ (a) A – B = O (b) A + B = I
È0 1˘ (c) A – B = I (d) A + 2B = O
(c) Í ˙ (d) none of these Ans. (a)
Î 2 - 4˚
5.12 Complete Mathematics—JEE Main
Solution: A + B = 2B¢ Ans. (a)
3A + 2B = I Solution: Using C2 Æ C2 – 3C1, C3 Æ C3 – 32C1, we get
fi B = 6B¢ – I
an 2( 5 n ) 16(5n )
fi B¢ = (6B¢ – I)¢ = 6B – I n +1
det(A) = an +1 2(5 ) 16(5n +1 )
Therefore, B = 6(6B – I) – I
fi (1 – 36)B = – 7I an + 2 2(5n + 2 ) 16(5n + 2 )
1 =0 [∵ C2 and C3 are proportional]
fi B= I
5 Thus, x = 0 is a root of det (A – xI) = 0
2 3 Example 32: First row of a matrix A is [1 3 2]. If
\ 3A = I - I = I
5 5 È -2 4 a ˘
1 adj A = Í -1 2 1 ˙
fi A= I Í ˙
5 ÎÍ3a -5 -2˚˙
\ A =B
then a possible value of det(A) is
Example 29: Let A and B be two non-zero 3 × 3 matrices (a) 1 (b) 2
such that AB = O. Then (c) –1 (d) –2
(a) Both A and B are non-singular Ans. (a)
(b) Exactly one of A, B is singular
Solution: We know that
(c) Both A and B are singular
det(A) = a11 A11 + a12 A12 + a13 A13
(d) Both A + B and AB are singular
Ans. (c) = (1) (–2) + (3) (–1) + (2) (3a)
= 6a – 5
Solution: Suppose A is non-singular, then
A–1 (AB) = A–1(O) fi B = O and (det(A))2 = det (adj A) = 11a – 10
A contradiction. Thus, (6a – 5)2 = 11a – 10
\ A is singular fi 36a2 – 71a + 35 = 0
Similarly, B is singular. fi a = 1, 35/36
Example 30: Suppose A is a 3×3 skew-symmetric Therefore, a possible value of det(A) is 1.
matrix. Let B = (I + A)–1 (I – A). Then Example 33: Suppose ABC is a triangle with sides a, b,
(a) B is orthogonal (b) B is skew symmetric c and semiperimeter s. Then matrix
(c) B2 = O (d) B is a diagonal matrix È s s-c ˘
Ans. (a) Í 2 2 ˙ Ès - a ˘
A= Í s ( s - b ) ( s - a ) ( s - c )˙ Ís - b ˙
Solution: We have Í s( s - c ) Î ˚ 2 ¥1
BB¢ = (I + A)–1 (I – A) [(I + A)–1 (I – A)]¢ Î (s - a)2 ˙˚3 ¥ 2
= (I + A)–1 (I – A) (I – A)¢ ((I + A)¢]–1 È bc ˘
= (I + A)–1 (I – A) (I + A) (I – A)–1 - Íca (s - a )(s - b)˙
[∵ A¢ = –A] Í ˙
= (I + A)–1 (I + A) (I – A) (I – A)–1 ÎÍ ab (s - a) ˚˙3 ¥1
[I + A and I – A commute] is equal to:
= (I) (I) = I Èa ˘ È0˘
Thus, B is orthogonal. (a) Íb ˙ (b) Í0˙
Í ˙ Í ˙
Example 31: Let an = 3n + 5n, n Œ N, and let ÎÍc ˙˚ ÎÍ0˙˚
Ê an an +1 an + 2 ˆ Ès ˘ Ès - a ˘
A = Á an +1 an + 2 an + 3 ˜ (c) Ís˙ (d) Ís - b ˙
Á ˜ Í ˙ Í ˙
Ë an + 2 an + 3 an + 4 ¯ ÍÎs˙˚ ÍÎs - c ˙˚
Then Ans. (b)
(a) 0 is a root of the equation det (A – xI) = 0 Solution:
(b) det(A) = an an + 2 an + 4 Ès(s - a) + (s - c)(s - b) - bc ˘
(c) det(A) < 0 A = Í(s - a )(s - b) (s(s - b) + (s - a )(s - c) - ca)˙
(d) det(A) = an + an + 2 + an + 4 Í ˙
ÎÍ(s - a )(s(s - c) + (s - a )(s - b) - ab) ˚˙
Matrices 5.13
2
But s(s – a) + (s – c) (s – b) – bc Solution: As A X = 0 has a non-trivial solution,
= s(s – a) + s(s – b – c) det (A2) = 0 fi (det(A))2 = 0
= s(2s – a – b – c) = 0 etc. fi det(A) = 0
È0˘ \ AX = 0 has a non-trivial solution.
Thus, A = Í0˙ Example 36: Suppose a, b Œ R and a π b.
Í ˙
ÍÎ0˙˚
Èa b ˘
Example 34: The number of matrices Let A= Í ˙.
Îb - a ˚
Èa b ˘ Let M be a matrix such that MA = A2m for some m Œ N, then
A= Í ˙ (where a, b, c, d Œ R) M is equal to
Îc d ˚
such that A–1 = – A is : (a) (a2 + b2)m I (b) (a2 + b2)m–1 A
(a) 0 (b) 1 (c) –(a2 + b2)m–1 A (d) (a2 + b2)m A
(c) 2 (d) infinite Ans. (b)
Ans. (d) Solution: We have
Solution: As A–1 exists, det(A) = ad – bc π 0. Èa 2 + b 2 0 ˘
A2 = Í 2 2
˙ = (a + b )I
È- a - b ˘ ÍÎ 0 a + b2 ˙˚
2
Also, det (–A) = det Í ˙ = ad – bc
Î -c - d ˚
fi A2m = ((a2 + b2) I )m = (a2 + b2)m I
= det (A)
1 -1 È- a -b˘
–1
since det(A ) = , A–1 = –A implies Also, A–1 = Í ˙
det( A) a 2 + b2 Î- b a ˚
1 1 Èa b ˘
= ad – bc = Í ˙
ad - bc a 2 + b 2 Îb - a ˚
fi (ad – bc)2 = 1 fi ad – bc = ±1. Now, MA = A2m = (a2 + b2)m I
If ad – bc = 1, then A–1 = –A fi M = (a2 + b2)m A–1
gives
= (a2 + b2)m–1 A
È d - b ˘ È- a - b ˘
A–1 = Í ˙=Í ˙ Èa b ˘
Î- c a ˚ Î - c - d ˚ Example 37: Let A = Í ˙ , be a 2 × 2 matrix where
fi a + d = 0. Îc d ˚
\ a(–a) – bc = 1 a, b, c, d Œ {0, 1}. The number of such matrices which have
fi bc = – (1 + a2) fi bc π 0 and inverse is
(a) 5 (b) 6
1 + a2 (c) 7 (d) 8
b =–
c Ans. (b)
È (1 + a 2 ) ˘ Solution: det(A) = ad – bc
\ Í a - ˙
A=
Í c ˙ , where c π 0. Note that det(A) can take value –1, 0 or 1. We have
ÍÎc - a ˙˚ det(A) = 1 ¤ ad = 1, bc = 0
Thus, there are infinite number of such matrices. ¤ a = 1, d = 1 or (b = 0, c = 0, b = 0; c = 1;
b = 1, c = 0)
and det(A) = –1 ¤ ad = 0 or bc = 1
TIP This is also possible in 3 cases.
It is unnecessary to consider the case ad – bc = –1 \ A–1 exists in 6 cases.

Example 35: Let A be a 3×3 matrix with entries from Example 38: If D = diag (d1, d2, …., dn) where di π 0,
the set of real numbers, If the system of equations A2X = 0 for i = 1, 2, . . . ., n, then D –1 is equal to
has a non-trivial solution, then (a) D
(a) AX = 0 has a non-trivial solution. (b) 2D
(b) AX = 0 does not have a non-trivial solution. (c) diag (d1–1, d2–1, …., d –1 n)
(c) A is a non-singular matrix. (d) Adj D
(d) none of these. Ans. (c)
Ans. (a) Solution: See Theory.
5.14 Complete Mathematics—JEE Main
Example 39: The inverse of a symmetric matrix (if it È1 0 2 ˘
exists) is Example 43: If A = Í5 1 x ˙ is a singular matrix,
(a) a symmetric matrix Í ˙
(b) a skew-symmetric matrix then x is equal to ÍÎ1 1 1 ˙˚
(c) a diagonal matrix (a) 3 (b) 5
(d) none of these (c) 9 (d) 11
Ans. (a) Ans. (c)
Solution: Let A be an invertible symmetric matrix. Solution: As A is a singular matrix,
We have AA–1 = A–1 A = In 1 0 0
fi (AA–1)¢ = (A–1 A)¢ = (In)¢
|A| = 0 fi 5 1 x - 10 = 0
fi (A–1)¢ A¢ = A¢(A–1)¢ = In
fi (A–1)¢ A = A(A–1)¢ = In 1 1 -1
(A–1)¢ = A–1 [inverse of a matrix is unique] [using C3 Æ C3 – 2C1]
Example 40: The inverse of a skew-symmetric matrix 1 x - 10
fi =0 fi – 1 – x + 10 = 0
(if it exists) is 1 -1
(a) a symmetric matrix fi x = 9.
(b) a skew-symmetric matrix
(c) a diagonal matrix Example 44: The value of x for which the matrix
(d) none of these È2 x -1 2 ˘
Ans. (b) Í ˙
A= Í 1 x 2 x 2 ˙ is singular is
Solution: We have A¢ = – A Í 1 1x 2 ˙
Now, AA–1 = A–1 A = In Î ˚
(a) ± 1 (b) ± 2
fi (AA–1)¢ = (A–1 A)¢ = (In)¢
(c) ± 3 (d) none of these
fi (A–1)¢ A¢ = A¢(A–1)¢ = In
Ans. (a)
fi (A–1)¢ (–A) = (– A) (A–1)¢ = In
Solution: We have
Thus, (A–1)¢ = – (A–1) [inverse of a matrix is unique]
2
Ê 2ˆ x 2x 1 2 x2 1 x
Example 41: The inverse of a skew-symmetric matrix |A| = Á ˜ + +2
Ë x¯ 1 x 2 1 2 1 1x
of odd order is
(a) a symmetric matrix 2 Ê1 ˆ
(b) a skew-symmetric matrix = (0 ) + 2 - 2 x 2 + 2 Á - x ˜
x Ëx ¯
(c) diagonal matrix
2 2 2
(d) does not exist = 2 x (1 - x ) + 2(1 - x ) = 2( x + 1) (1 - x )
Ans. (d) x x
Solution: Let A be a skew-symmetric, matrix of order Now, |A| = 0 fi x = + 1.
n. By definition Example 45: If square matrix A is such that 3A3 + 2A2 +
A¢ = – A 5A + I = O, then A–1 is equal to
fi | A¢| = |– A| fi | A| = (– 1)n |A|
(a) 3A2 + 2A + 5I
fi | A| = – |A| [∵ n is odd]
(b) – (3A2 + 2A + 5I)
fi 2|A| = 0 fi |A| = 0
\ A–1 does not exist. (c) 3A2 – 2A – 5I
(d) none of these
Example 42: If A is an orthogonal matrix, then |A| is Ans. (b)
(a) 1 (b) – 1
Solution: We have A(3A2 + 2A + 5I) =–I
(c) ± 1 (d) 0
–1 2
Ans. (c) \ A = – (3A + 2A + 5I). [\ Inverse of a matrix
is unique]
Solution: As A is an orthogonal matrix,
A¢ A = AA¢ = In È1 0 ˘ È1 0 ˘
Example 46: If A = Í ˙ and I = Í ˙ then which
fi |A¢ A| = |AA¢| = |In| Î1 1 ˚ Î0 1 ˚
fi |A¢| |A| = 1 fi |A| |A| = 1 one of the following holds for all n ≥ 1, by the principle of
fi |A|2 = 1 fi |A| = + 1 mathematical induction.
Matrices 5.15
(a) An = nA + (n – 1) I È-1 0 1 ˘ È 2 6 4 ˘ È-3 -5 -5˘
An = 2n – 1 A + (n – 1) I
A–1 = Í 1 1 3˙ Í 1 0 1 ˙ = Í 0 9 2 ˙
(b)
\
(c) An = nA – (n – 1) I Í ˙Í ˙ Í ˙
(d) An = 2n – 1 A – (n – 1) I. ÍÎ 2 0 2 ˙˚ ÍÎ-1 1 -1˙˚ ÎÍ 2 14 6 ˙˚
Ans. (c) Example 49: If w is a complex cube root of unity, then
È1 0 ˘ È 1 w2 w ˘
Solution: For n = 2, A2 = Í ˙ Í ˙
Î2 1 ˚ the matrix A = Íw 2 w 1 ˙ is a
Í ˙
È3 0 ˘ 2 ÍÎ w 1 w 2 ˙˚
For n = 2, RHS of (a) = 2A + I = 3 Í ˙ πA
Î2 3 ˚ (a) singular matrix
For n = 2, RHS of (b) = 2A + I π A2 (b) non-singular matrix
So possible answer is (c) or (d). (c) skew-symmetric matrix
(d) none of these
È1 0 ˘
In fact An = Í ˙ which equals nA – (n – 1)I. Ans. (a)
În 1 ˚
Solution: We have
Alternatively. Write A = I + B
1 w2 w 1+ w2 + w w2 w
È0 0 ˘ 2 2
where B = Í ˙ |A| = w w 1 = w +w +1 w 1 =0
Î1 0 ˚ 2 2
As B2 = 0, we get Br = 0 " r ≥ 2. w 1 w w +1+w 1 w2
By the binomial theorem using C1 Æ C1 + C2 + C3 and 1 + w + w 2 = 0
An = I + nB = I + n(A – I) = nA – (n – 1)I. \ A is a singular matrix.
Example 47: Let A, B, C be three square matrices of the È0 1 2 ˘
same order, such that whenever AB = AC then B = C, if A is
Example 50: If A = Í1 2 3˙ and
(a) singular Í ˙
(b) non-singular ÍÎ3 x 1 ˙˚
(c) symmetric È1/ 2 -1/ 2 1/ 2 ˘
(d) skew-symmetric A–1 = Í -4 3 y ˙ , then
Ans. (b) Í ˙
ÍÎ5 / 2 -3 / 2 1/ 2 ˙˚
Solution: If A is non-singular, A–1 exists. (a) x = 1, y = – 1 (b) x = – 1, y = 1
Thus, AB = AC fi A–1 (AB) = A–1 (AC) (c) x = 2, y = – 1/2 (d) x = 1/2, y = 1/2
–1 –1
fi (A A) B = (A A) C fi IB = IC Ans. (a)
fi B = C. Solution: We have
È 2 6 4˘
Example 48: Let B = Í 1 0 1 ˙ . If the product BA È1 0 0 ˘ È0 1 2 ˘ È1/ 2 -1/ 2 1 / 2 ˘
Í ˙ Í0 1 0 ˙ = AA-1 = Í1 2 3˙ Í -4 3 y ˙
ÎÍ -1 1 -1 ˚˙ Í ˙ Í ˙Í ˙
ÍÎ0 0 1 ˙˚ ÍÎ3 x 1 ˙˚ ÍÎ5 / 2 -3 / 2 1/ 2 ˙˚
È -1 0 1 ˘
È 1 0 y +1 ˘
has inverse C = Í 1 1 3 ˙ , then A–1 equals
Í ˙ = Í 0 1 2( y + 1)˙
ÍÎ 2 0 2 ˙˚ Í ˙
ÎÍ4(1 - x ) 3( x - 1) 2 + xy ˙˚
È -3 -5 5 ˘ È -3 5 5 ˘
fi 1 – x = 0, x – 1= 0,
(a) Í 0 9 14 ˙ (b) Í 0 0 9 ˙
Í ˙ Í ˙ y +1 = 0, y + 1= 0, 2 + xy = 1
ÍÎ 2 2 6 ˙˚ ÍÎ 2 14 16 ˙˚ \ x = 1, y = – 1.
È -3 -5 -5 ˘ È -3 -3 -5 ˘ Èa b ˘
(c) Í 0 0 2 ˙ (d) Í 0 9 2 ˙
Example 51: Let A = Í ˙ , where a, b, c, d Œ R. If
Í ˙ Í ˙ Îc d ˚
ÍÎ 2 14 6 ˙˚ ÍÎ 2 14 6 ˙˚ A – a I is invertible for all a Œ R, then
Ans. (c) (a) bc > 0 (b) bc = 0
Solution: We have (BA)–1 = C fi A–1 B–1 = C Ê 1 ˆ
(c) bc > min Ë0, ad ¯ (d) a = 0
fi A–1 = CB 2
5.16 Complete Mathematics—JEE Main
Ans. (c) Solution: A2 – A + I = 0 fi I = A – A2 = A(I – A).
Solution: As A – a I is invertible for all a Œ R. Thus, A–1 = I – A.
det (A – a I) π 0 " a Œ R. Ê 1 -1 1 ˆ
Example 55: Let A = Á 2 1 -3˜ and
fi (a – a) (d – a) – bc π 0 " a Œ R. Á ˜
2 Ë1 1 1 ¯
fi a – (a + d)a + ad – bc π 0 " a Œ R.
Therefore Ê 4 2 2ˆ
2
(a + d) – 4(ad – bc) < 0 (10)B = Á - 5 0 a ˜
Á ˜
fi (a – d)2 + 4 bc < 0 Ë 1 2 3¯
Therefore, bc < 0. If B is the inverse of matrix A, then a equals
Also, a2 + d2 – 2ad + 4bc < 0 (a) 2 (b) –1
(c) –2 (d) 5
fi 0 £ a2 + d2 < 2ad – 4bc Ans. (d)
1 Solution: 10I = 10(AB) = A(10B)
fi bc < ad.
2
Ê 10 0 0 ˆ Ê 1 -1 1 ˆ Ê 4 2 2 ˆ
1
Thus, bc < min Ê0, ad ˆ fi Á 0 10 0 ˜ = Á 2 1 -3˜ Á -5 0 a ˜
Ë 2 ¯ Á ˜ Á ˜Á ˜
Ë 0 0 10¯ Ë 1 1 1 ¯ Ë 1 -2 3 ¯
È1 2 ˘ 2
Example 52: If A = Í ˙ , then A – 5A – I equals
Î3 4 ˚ Ê 10 0 5 - a ˆ
(a) O (b) I = Á 0 10 a - 5˜
(c) 2I (d) none of these Á ˜
Ë 0 0 5 + a¯
Ans. (b)
fi a = 5.
È1 2 ˘ È1 2 ˘ È 7 10 ˘
Solution: A2 = Í ˙Í ˙=Í ˙ Èa b ˘ 2 Èa b ˘
Î3 4 ˚ Î3 4 ˚ Î15 22 ˚ Example 56: If A = Í ˙ and A = Í b a ˙ , then
Îb a ˚ Î ˚
We have
(a) a = a2 + b2 b = 2ab
È 7 10 ˘ È1 2 ˘ È2 0 ˘
A2 – 5A = Í ˙ -5 Í ˙=Í ˙ = 2I (b) a = a 2 + b 2, b = a 2 – b 2
Î15 22 ˚ Î3 4 ˚ Î0 2 ˚
(c) a = 2ab, b = a2 + b2
fi A2 – 5A – I = I
(d) a = a2 + b2, b = –2ab
È3 2 ˘ 2 Ans. (a)
Example 53: Let A = Í ˙ satisfies A + aA + bI = O,
Î1 1 ˚ Èa b ˘ Èa b ˘ Èa + b
2 2
2ab ˘
then a, b are respectively equal to Solution: A2 = Í ˙ Í ˙ =Í ˙
Îb a ˚ Îb a ˚ ÍÎ 2ab a + b2 ˙˚
2
(a) – 4, 2 (b) –3, 3
(c) – 4, 1 (d) –3, 1 Èa b ˘ Èa 2 + b2 2ab ˘
Ans. (c) \ Íb a ˙ = Í ˙
Î ˚ ÎÍ 2ab a + b2 ˙˚
2

È3 2 ˘ È3 2 ˘ È11 8˘ fi a = a2 + b2, b = 2ab


Solution: A2 = Í ˙Í ˙=Í ˙
Î1 1 ˚ Î1 1 ˚ Î 4 3˚
Example 57: Let w π 1 be a cube root of unity and S be
Now the set of all non-singular matrices of the form
È11 + 3a + b 8 + 2 a ˘
O = A2 + aA + bI = Í È1 a b˘
Î 4+a 3 + a + b ˙˚ Í ˙
A= Íw 1 c˙
fi 3a + b = –11, 4 + a = 0, a + b = –3 Íw 2
Î w 1 ˙˚
fi a = –4, b = 1.
where a, b, c are either w or w2. Then number of distinct
Example 54: Let A2 – A + I = O, then inverse of A is matrices in the set S is
(a) I – A (b) A – I (a) 2 (b) 6
(c) A + I (d) A (c) 4 (d) 8
Ans. (b) Ans. (a)
Matrices 5.17
Solution: We have Ans. (d)
1 c w c w 1 Solution: Note that A2 = 49I
|A| = 1 -a 2 +b
w 1 w 1 w2 w fi A–1 = 1 A [\ A is non-singular]
2
= 1 – cw – aw + acw = (1 – aw)(1 – cw) 49
A is non-singular ¤ |A| π 0 ¤ a = c = w. Example 62: Let A = [5 3] and B = [3 7]
Note that b can take any value w or w2. The number of non-zero matrices C such that AC = BC is
Thus, there are two non-singular matrices. (a) 0 (b) 1
(c) infinitely many (d) none of these
Example 58: If a matrix A is both symmetric and skew-
Ans. (c)
symmetric, then
(a) A is a diagonal matrix È2x ˘
Solution: Note that every matrix of the form Í ˙ , x Œ R
(b) A is a scalar matrix
satisfies the required condition. Îx˚
(c) A is a zero matrix
(d) none of these È0 2y z ˘
Ans. (c) Example 63: If A = Í x y - z ˙ satisfies A¢ = A – 1,
Í ˙
Solution: We are given A¢ = A and A¢ = –A. then ÍÎ x - y z ˙˚
\ A = –A fi 2A = O fi A = O
(a) x = ± 1/ 6 , y = ± 1/ 6 , z = ± 1/ 3
È2 -1˘ È5 2 ˘ È 2 5˘
Example 59: Let A = Í ˙ , B = Í7 4 ˙ , C = Í 3 8 ˙ .
Î3 4 ˚ Î ˚ Î ˚ (b) x = ± 1/ 2 , y = ± 1/ 6 , z = ± 1/ 3
Let D be a matrix such that CD = AB, then D equals
(a) I (b) O (c) x = ± 1/ 2 , y = ± 1/ 6 , z = ± 1/ 3
(c) –A (d) none of these (d) x = ± 1/ 2 , y = ± 1/3, z = ± 1/ 2
Ans. (d)
Ans. (b)
Solution: CD = AB fi D = C –1 (AB)
Solution: A¢ = A – 1 ¤ AA¢ = I
È2 -1˘ È5 2 ˘ È 3 0 ˘
Now AB = Í ˙Í ˙=Í ˙ and È0 2y z ˘ È 0 x x˘
Î3 4 ˚ Î7 4 ˚ Î43 22 ˚ Í ˙ Í
Now, AA¢ = x y - z 2 y y - y˙
Í ˙ Í ˙
È 8 -5˘ ÍÎ x - y z ˙˚ ÎÍ z - z z ˙˚
C –1 = Í ˙
Î-3 2 ˚
È 4 y2 + z 2 2 y2 - z 2 - 2 y2 + z 2 ˘
È 8 -5˘ È 3 0 ˘ È-191 -110 ˘ Í ˙
\ D= Í ˙Í ˙ = Í 44 ˙˚ = Í 2 y2 - z 2 2 2 2
Î-3 2 ˚ Î43 22 ˚ Î 77 x +y +z x 2 - y2 - z 2 ˙
Í 2 2 ˙
Example 60: If A2 = A, the (I + A)4 equals ÍÎ- 2 y + z x 2 - y2 - z 2 x 2 + y 2 + z 2 ˙˚
(a) I + 15A (b) I + 7A Thus, AA¢ = I
(c) I + 8A (d) I + 11A fi 4y2 + z2 = 1, 2y2 – z2 = 0
Ans. (a)
x2 + y2 + z2 = 1, x2 – y2 – z2 = 0
Solution: As I and A commute, we can apply binomial
theorem to expand (I + A)4. We have \ x = ± 1/ 2 , y = ± 1/ 6 , z = ± 1/ 3
(I + A)4 = I + 4A + 6A2 + 4A3 + A4. Example 64: Suppose A is square matrix such that
But A3 = A2 = A, A4 = A3 = A A3 = I, then (A + I)3 + (A – I)3 – 6A equals
\ (I + A)4 = I + 4A + 6A + 4A + A = I + 15A (a) I (b) 2I
È 0 0 - 7˘ (c) A (d) 3A
Example 61: The matrix A = Í 0 - 7 0 ˙ is a Ans. (b)
Í ˙
ÍÎ- 7 0 0 ˙˚ Solution: Since AI = IA, we have
(a) scalar matrix (b) diagonal matrix (A + I)3 + (A – I)3 – 6A
(c) unit matrix (d) non-singular = 2A3 + 2(3AI2) – 6A = 2I + 6A – 6A = 2I
5.18 Complete Mathematics—JEE Main
Example 65: The number of 3 ¥ 3 matrices A whose Example 68: Let a, b and c be three real numbers
entries are either 0 or 1 and for which the system of equations satisfying
Èx˘ È2 ˘ È1 9 7 ˘
A Í y ˙ = Í0 ˙ has exactly five distinct solutions is [a b c] Í8 2 7˙ = [0 0 0] (1)
Í ˙ Í ˙ Í ˙
ÎÍ z ˚˙ ÎÍ3˚˙ ÎÍ7 3 7˙˚
(a) 0 (b) 511
If the point P(a, b, c) with reference to (1), lies on the plane
(c) 1024 (d) 5
2x + y + z = 1, then the value of 7a + b + c is
Ans. (a)
(a) 0 (b) 12
Solution: We know that the system of equation (c) 7 (d) 6
AX = B
Ans. (d)
has either (i) unique solution
or (ii) infinite number of solutions Solution: We have
or (iii) no solution. a + 8b + 7c = 0, 9a + 2b + 3c = 0, 7a + 7b + 7c = 0
Hence, there cannot exist any matrix A such that Subtracting the last equation from first and multiplying the
È x ˘ È2 ˘ last equation by (9/7) and subtracting from the second equa-
A Í y ˙ = Í0 ˙ tion we obtain – 6a + b = 0, 7b + 6c = 0
Í ˙ Í ˙
ÍÎ z ˙˚ ÍÎ3˙˚ a b c
fi = = = k (say)
has five distinct solutions. 1 6 –7
Example 66: The number of 3 ¥ 3 non-singular matrices, Thus, a = k, b = 6 k, c = – 7 k.
with four entries as 1 and all other entries as 0 is As (a, b, c) = (k, 6 k, – 7 k) lies on 2x + y + z = 1, we get k = 1.
(a) 6 (b) at least 7 Thus, 7a + b + c = 7k + 6k – 7k = 6k = 6
(c) less than 4 (d) 5
Ans. (b) Example 69: Let P and Q be 3 ¥ 3 matrices with P π Q
Solution: The matrix If P3 = Q3 and P2 Q = Q2P, then determinant of(P2 + Q2) is
equal to
È1 a b ˘ (a) 1 (b) 0
Íc 1 d ˙
Í ˙ (c) – 1 (d) –2
ÍÎe f 1 ˙˚ Ans. (d)
where exactly one of a, b, c, d, e, f is 1 and rest of them are Solution: P3 = Q3 and P2Q = Q2P gives
zeros, is invertible.
P 3 – P 2Q = Q 3– Q 2P
There are six such matrices.
È1 0 1 ˘ fi P2(P – Q) = – Q2(P – Q)
Also, the matrix Í0 1 0 ˙ is invertible. fi (P + Q2) (P – Q) = 0
2
Í ˙
ÎÍ1 0 0 ˚˙ If det (P2 + Q2) π 0, then P2 + Q2 is invertible and hence P
Thus, there are at least 7 such matrices which are invertible. = Q. Therefore, det (P2 + Q2) = 0.
Example 67: Consider the system of linear equations: Ê1 0 0ˆ
x1 +2x2 + x3 = 3 Example 70: Let A = Á 2 1 0˜ . If u1 and u2 are
2x1 +3x2 + x3 = 3 Á ˜
Ë3 2 1¯
3x1 +5x2 + 2x3 = 1
The system has Ê1 ˆ Ê 0ˆ
(a) a unique solution column matrices such that Au1 = Á 0˜ and Au2 = Á 1 ˜ , then
Á ˜ Á ˜
(b) no solution u1 + u2 equals Ë 0¯ Ë 0¯
(c) infinite number of solutions
(d) exactly 3 solutions Ê -1ˆ Ê -1ˆ
Ans. (b) (a) Á 1˜ (b) Á -1˜
Solution: Adding the first two equations and subtract- Á ˜ Á ˜
Ë -1¯ Ë 0¯
ing the third from the sum, we obtain
(x1 + 2x2 + x3) + (2x1 + 3x2 + x3) – (3x1 + 5x2 + 2x3) = 3 + 3 – 1 Ê 1ˆ Ê -1ˆ
fi 0= 5 (c) Á -1˜ (d) Á 1˜
Á ˜ Á ˜
Thus, the system of equations has no solution. Ë -1¯ Ë 0¯
Matrices 5.19
Ans. (c) Ans. (a)
Solution: We have Solution: The augmented matrix is given by
Ê1 ˆ Ê 0ˆ Ê1 ˆ Ê3 -2 1 bˆ
A(u1 + u2) = Au1 + Au2 = Á 0˜ + Á 1˜ = Á 1 ˜ (A|B) = Á 5 -8 9 3˜
Á ˜ Á ˜ Á ˜ Á ˜
Ë 0¯ Ë 0¯ Ë 0¯ Ë2 1 a -1¯
We solve the above equation for u1 + u2.
Applying R1 Æ 2R1 – R2, we get
We consider the augmented matrix
Ê1 4 - 7 2b - 3ˆ
Ê1 0 0 1ˆ
(A|B) = Á 2 1 0 1˜ (A|B) ~ Á 5 -8 9 3 ˜
Á ˜ Á ˜
Ë3 2 1 0¯ Ë2 1 a -1 ¯

Applying R3 Æ R3 – 2R2 + R1 and R2 Æ R2 – 2R1, we get Applying R2 Æ R2 – 5R1, R3 Æ R3 – 2R1, we get


Ê1 4 -7 2b - 3 ˆ
Ê1 0 01 ˆ Ê 1ˆ
Á
(A|B) ~ 0 - 28 44 18 - 10b˜
(A|B) ~ Á 0 1 0 -1˜ fi u1 + u2 = Á -1˜ Á ˜
Á ˜ Á ˜ Ë 0 -7 a + 14 5 - 4b ¯
Ë0 0 1 -1¯ Ë -1¯
Example 71: If P is a 3 ¥ 3 matrix such that The system of equations will have no solution if
P´ = 2P + I, then there exists a column matrix -28 44 18 - 10b
= π
Ê xˆ -7 a + 14 5 - 4b
X = Á y˜ π O such that fi a + 14 = 11 and 20 – 16b π 18 –10b
Á ˜
Ë z¯ fi a = –3 and b π –1/3
(a) PX = O (b) PX = X Example 74: Suppose I + A is non-singular. Let B =
(c) PX = 2X (d) PX= –X (I + A)–1 and C = I – A, then
Ans. (d) (a) BC = CB (b) BC = O
Solution: We have (c) BC = I (d) none of these
P = (P¢)¢ = (2P + I )¢ = 2P¢ + I Ans. (a)
P = 2(2P + I ) + I fi P = –I Solution: Write C = 2I – (I + A), then
Thus, there exists X π 0 such that PX = –X BC = 2(I + A)–1 – I = CB
Ê 4 3ˆ Ê a bˆ
Example 72: Let A = Á and x, y Œ R are such that Example 75: Let A = Á be such that A3 = O, but
Ë 2 5˜¯ Ë c d ˜¯
A2 + xA + yI = 0 then (x, y) equals A π O, then
(a) (–9, –14) (b) (9, –14) (a) A2 = O (b) A2 = A
(c) (–9, 14) (d) (9, 14) (c) A2 = I – A (d) none of these
Ans. (c) Ans. (a)
Ê 22 27ˆ Solution: As A3 = O, we get |A3| = 0
Solution: We have A2 = Á
Ë 18 31 ˜¯ |A|3 = 0 fi |A| fi ad – bc = 0.
2
\ A + xA + yI = O Also,
fi 22 + 4x + y = 0, 27 + 3x = 0 Ê a 2 + bc (a + d ) bˆ
A2 = Á ˜
18 + 2x = 0, 31 + 5x + y = 0 Ë (a + d )c bc + d 2 ¯
fi x = –9, y = 14
Ê a 2 + ad (a + d )bˆ
Example 73: The system of equations = Á ˜
Ë (a + d )c ad + d 2 ¯
Ê3 - 2 1ˆ Ê xˆ Ê bˆ
Á 5 - 8 9˜ Á y˜ = Á 3˜ = (a + d)A
Á ˜ Á ˜ Á ˜ If a + d = 0, we get A2 = O.
Ë2 1 a¯ Ë z¯ Ë -1¯
Suppose a + d π 0, then
has no solution if a and b are
A3 = A2A = (a + d)A2
(a) a = –3, b π 1/3 (b) a = 2/3, b π 1/3
(c) a π 1/4, b = 1/3 (d) a π –3, b π 1/3 fi O = (a + d)A2 fi A2 = O
5.20 Complete Mathematics—JEE Main

Assertion-Reason Type Questions

Example 76: Let A be a 2 ¥ 2 matrix. Statement-1: If A π I and A π – I, then det (A) = – 1.


Statement-1: adj(adj A) = A Statement-1: If A π I and A π – I, then Tr (A) π 0.
Statement-2: |adj A| = |A| Ans. (c)
Ans. (b) Ê a bˆ
Solution: Let A = Á
Èa b ˘ È d - b˘ Ë c d ˜¯
Solution: Let A = Í ˙ then adj A = Í- c a ˙
Îc d ˚ Î ˚ Now, A = I fi det (A2) = 1
2

Èa b ˘ fi (det A)2 = 1 fi det A = ± 1.


fi adj (adj A) = Í =A A = I fi A = A –1
2
Îc d ˙˚ Also,
\ Statement-1 is true. Ê a bˆ Ê d – bˆ
fi ÁË c d ˜¯ = det A ÁË – c a ˜¯
Also, |adj A| = ad – bc = |A|
\ Statement-2 is true If det A = 1, then
But statement-2 is not a correct explanation for statement-1. a = d, b = – b, c = – c fi a = d, b = c = 0.
Example 77: Let A be a 2 ¥ 2 matrix with non-zero Ê a 0ˆ
In this case A = ÁË .
entries and let A2 = I, where I is 2 ¥ 2 identity matrix. Define 0 a˜¯
Tr (A) = sum of diagonal elements of A, and
|A| = determinant of matrix A. | A| = 1 fi a2 = 1 fi a = ± 1.
Statement-1: Tr (A) = 0 \ A = I or A = – I. A contradiction.
Statement-2: |A| = 1. Thus, det (A) = – 1.
Ans. (c) Ê a bˆ Ê d - bˆ Ê - d b ˆ
\ Á =–1Á =
Èa b ˘ Ë c d ˜¯ Ë - c a ˜¯ ÁË c - a˜¯
Solution: Let A = Í ˙
Îc d ˚ fi a = – d fi Tr (A) = a + d = 0.
Now, 2 2
A = I fi |A | = |I | \ Statement -1 is true and Statement-2 is false.
fi |A|2 = 1 fi |A| = ± 1. Example 79: Let
Suppose |A| = 1.
In this case, A2 = I fi A = A – 1 Ê cos q + sin q 2 sin q ˆ
A(q) = Á ˜
Ê a bˆ Ê d - bˆ Ë - 2 sin q cos q - sin q ¯
fi ÁË c d ˜¯ = ÁË - c a ˜¯
Statement-1: A(p/3)3 = – I
fi a = d, b = 0, c = 0 Statement-2: A(q) A(j) = A(q + j)
Tr (A) = 0 fi a + d = 0
fi 2a = 0 fi a = 0 Ans. (a)
Ê a bˆ
Ê 0 0ˆ Solution: A(q) A(j) = Á where
In this case A = Á Ë -b c ˜¯
Ë 0 0¯˜
a = (cos q + sin q) (cos j + sin j) – 2 sin q sin j
Thus, if we assume statement-2 is true then we get state-
ment-2 is false. = (cos q cos j – sin q sin j) + (sin q cos j + cos q sin j)
In case |A| = – 1, then = cos (q + j) + sin (q + j)
A = – A–1 b= 2 [sin j (cos q + sin q) + sin q (sin j – cos f)]
Ê a b ˆ Ê d - bˆ
fi = -Á fi a + d = 0 = 2 sin (q + j)
ËÁ c d ¯˜ Ë -c a ¯˜
and c = – 2 sin q sin j + (cos q – sin q) (cos j – sin j)
\ |A| = – 1 fi Tr(A) = 0.
= cos q cos j – sin q sin j – (sin q cos j
Therefore statement-1 is true and statement-2 is false. + cos j sin q)
Example 78: Let A be a 2 ¥ 2 matrix with real entries. = cos (q + j) – sin (q + j)
Let I be the 2 ¥ 2 identity matrix. Denote by Tr (A), the sum Thus, A(q) A(j) = A(q + j)
of diagonal entries of A. Assume that A2 = I.
Matrices 5.21
A(q)2 = A(2q) È1 ˘ È0 ˘
Solution: Let E1 = Í ˙ , E2 = Í ˙ .
A(q)3 = A(2q) A(q) = A(3q). Î0 ˚ Î1 ˚
\ A(p /3)3 = A(p) = – I.
As X¢AX = O " X,
Ê a bˆ E1¢AE1 = O fi a11 = 0
Example 80: Suppose X = Á satisfies the equation
Ë c d ˜¯ E2¢AE2 = O fi a22 = 0.
2
X – 4X + 3I = O
Statement-1: If a + d π 4, then there are just two such Next, (E1 + E2)¢ A(E1 + E2) = O
matrices X. fi a12 + a21 = 0
Statement-2: There are infinite number of matrices X, satis-
fying X2 – 4X + 3I = 0. È 0 a12 ˘
Thus, A = Í is a skew-symmetric matrix.
Î – a12 0 ˙˚
Ans. (b)
In case A is symmetric, a12 = a21
Solution: X2 – 4X + 3I = O fi (X – I) (X – 3I) = O
\ 2a12 = 0 fi a12 = 0.
Ê ( a - 1) ( a - 3) + bc b ( a + d - 4 ) ˆ Ê 0 0ˆ Thus, A = O, in this case.
fi =
ÁË c ( a + d - 4 ) (d - 1) (d - 3) + bc˜¯ ÁË 0 0¯˜
Example 83: Suppose A, B and C are three 3×3 matrices,
If a + d π 4, then b = 0, c = 0. such that AB is invertible.
(a – 1) (a – 3) = 0, (d – 1) (d – 3) = 0. Statement-1: If AB = AC then B = C.
fi a = 1, 3, d = 1, 3 Statement-2: A is a invertible.
As a + d π 4, a = 1, d = 1 or a = 3, d = 3. Ans. (a)
Ê 1 0ˆ Ê 3 0ˆ Solution: AB is invertible
Thus, X= Á or X = Á
Ë 0 1˜¯ Ë 0 3˜¯ fi det(AB) π 0
If a + d = 4, we get fi det(A) det(B) π 0
Ê ( a - 1) ( a - 3) + bc 0 ˆ fi det(A) π 0
ÁË = O.
0 (1 - a ) (3 - a ) + bc˜¯ fi A is invertible
\ There are infinite number of matrices satisfying X2 – Premultiplying both the sides of
4X + 3I = 0 as there are infinite number of values of a, b, c AB = AC
satisfying the relation a2 – 4a + 3 + bc = 0. by A–1, we get
Ê1 0 ˆ Ê 0 2ˆ A–1 (AB) = A–1 (AC)
Example 81: Suppose A = Á and B = Á .
Ë 0 - 1˜¯ Ë 2 0˜¯ fi (A–1A)B = (A–1A)C
Let X be a 2 ¥ 2 matrix such that fi IB = IC or B = C.
X¢AX =B.
Example 84: Suppose a, b, g are real numbers such that
Statement-1: X is non-singular and det (X) = ± 2 cos(b – g ) + cos(g – a) + cos (a – b ) = –3/2 (1)
Statement-2: X is a singular matrix. Statement-1: The matrix
Ans. (c) Ê cos a sin a -2ˆ
Solution: If X = O, then X¢AX = O fi B = O. A = Á cos b sin b 1 ˜
Á ˜
A contradiction. Ë cos g sin g 1 ¯
Let det (X) = a, then det (X¢) = a is a singular matrix.
\ det (X¢AX) = det (B) Statement-2: cos a + cosb + cos g = 0
fi a(– 1)a = – 4 fi a = ± 2. and sin a + sin b + sin g = 0
As det (X) π 0, X cannot be a singular matrix. Ans. (a)

Èa a12 ˘ È x1 ˘ È y1 ˘ Solution: Write (1) as


Example 82: Let A = Í 11 ˙ ,X= Í ˙,Y= Í ˙ cos2 a + cos2 b + cos2 g
Îa21 a22 ˚ Î x2 ˚ Î y2 ˚
+ sin2 a + sin2 b + sin2 g
Statement-1: If X¢AX = O for each X, then A must be a
+ 2cos b cos g + 2 sin b sin g
skew-symmetric matrix.
+ 2cos g cos a + 2 sin g sin a
Statement-2: If A is symmetric and X¢AX = O for each X,
+ 2cos a cos b + 2 sin a sin b = 0
then A = O
Ans. (b)
5.22 Complete Mathematics—JEE Main
fi (cos a + cos b + cos g )2 Statement-2: If AX = O implies X = O, then A is non-singular.
2
+ (sin a + sin b + sin g ) = 0 Ans. (a)
fi cos a + cos b + cos g = 0 Solution: If A is not non-singular, then AX = O has
infinitely many solutions.
and sin a + sin b + sin g = 0 Thus, if AX = O fi X = O, then A must be non-singular.
Using R1 Æ R1 + R2 + R3, we get R1 consists of all zeros, Next, consider
therefore det(A) = 0. (I + A)X = O
Thus, A is a singular matrix. fi AX = – X fi (AX) = (–X)¢
fi X¢ A¢ = – X¢ fi X¢(–A) = –X¢
Example 85: Let A = (aij)3×3 be a 3 × 3 matrix with aij Œ
R for 1 £ i £ 3, 1 £ j £ 3. Let X be a 3 × 1 matrix. fi X¢ A = X¢ fi X¢AX = X¢X
Statement-1: If A is skew-symmetric, then I + A is non- fi X¢ (–X) = X¢X fi X¢X = O fi X = O.
singular. \ I + A is non-singular.

LEVEL 2

Straight Objective Type Questions

Ê 1 2ˆ Ê a 0ˆ Solution:
Example 86: Let A = Á and B = Á , a, b Œ N.
Ë 3 4˜¯ Ë 0 b˜¯ È1 0 ˘ È2 cos a 0 ˘
fi Í0 1 ˙ = Í 0
(a) Then there exists infinitely many B’s such that Î ˚ Î 2 cos a ˙˚
AB = BA 1 p
(b) there cannot exist B such that AB = BA fi 2 cos a = 1 fi cos a = = cos
2 3
(c) there exist more than one but finite number of B’s
such that AB = BA p
fi a = 2np ± ,nŒI
(d) there exists exactly one B such that AB = BA 3
Ans. (a) 0 - tan(a / 2)˘
È
Solution: If b = a, B = aI and B commutes with every Example 89: Let A = Í ˙
Î tan(a / 2 ) 0 ˚
matrix A.
a π (2n + 1) p, n Œ I, (I + A) (I – A)–1 equals
È5 5a a˘
Ècos a - sin a ˘ È cos a sin a ˘
Example 87: Let A = Í0 a 5a ˙ . If | A2 | = 25 then (a) Í (b) Í
| a | equals
Í ˙ Î sin a cos a ˙˚ Î- sin a cos a ˙˚
ÎÍ0 0 5 ˙˚
(a) 52 (b) 1 Ètan a 0 ˘ Ètan a 0 ˘
(c) Í (d) Í
(c) 1/5 (d) 5 Î 0 tan a ˙˚ Î 0 - tan a ˙˚
Ans. (c) Ans. (a)
Solution: | A2 | = 25 fi | A |2 = 25 È 1 tan(a / 2)˘
Solution: I – A = Í ˙
\ (25a)2 = 25 fi 25a2 = 1 fi | a | = 1/5 Î- tan(a / 2) 1 ˚
Ê cos a - sin a ˆ 1 È 1 - tan(a / 2)˘
Example 88: If A = Á and A + A¢ = I, then (I – A)– 1 =
Ë sin a cos a ˜¯ Í
sec 2 (a / 2) Îtan (a / 2) 1 ˙
˚
then a equals
p p Now, (I + A) (I – A)–1
(a) 2 np ± ,nŒI (b) 2np ± ,nŒI 1 - tan(a / 2)˘
2 3 1 È
= Í ˙
sec 2 (a / 2) Îtan (a / 2) 1 ˚
2p 4p
(c) 2np ± ,nŒI (d) 2np ± ,nŒI
3 3 È 1 - tan(a / 2)˘
Ítan (a / 2) 1 ˙
Ans. (b) Î ˚
Matrices 5.23

È1 - tan (a / 2) -2 tan(a / 2) ˘
2 and A3 = A2 A
Í ˙ È1 0 0 ˘ È1 0 0 ˘ È1 0 0˘
1 + tan 2 (a / 2) 1 + tan 2 (a / 2) ˙ Ècos a - sin a ˘
= Í = = Í0 -1 5 ˙ Í0 1 1 ˙ = Í0 -11 19 ˙
Í 2 tan (a / 2) 1 - tan 2 (a / 2) ˙ ÍÎ sin a cos a ˙˚ Í ˙Í ˙ Í ˙
Í ˙ ÍÎ0 -10 14 ˙˚ ÍÎ0 -2 4 ˙˚ ÍÎ0 -38 46 ˙˚
ÍÎ1 + tan 2 (a / 2) 1 + tan 2 (a / 2) ˙˚
Now,
È 3 /2 1/ 2 ˘ È1 1˘ 6I = A3 + cA2 + dA
Example 90: Let P = Í ˙, A = Í0 1˙ and
ÍÎ -1/ 2 3 / 2 ˙˚ Î ˚ 6 = 1 + c + d, 0 = 19 + 5c + d
Q = PAP¢, then P¢Q2015 P is 6 = –11 – c + d
6 = 46 + 14c + 4d, 0 = –38 – 10c – 2d
È1 -2015˘ È2015 1 ˘
(a) Í (b) Í fi d = 11, c = –6
Î0 1 ˙˚ Î 0 2015˙˚
Example 92: If a, b, c are non-zero, then number of
È1 2015˘ È2015 2015˘ solutions of
(c) Í (d) Í
Î0 1 ˙˚ Î 0 2015˙˚
2 x2 y2 z2 = 0 (1)
Ans. (c) - -
a2 b2 c2
È cos(p / 6) sin (p / 6) ˘
Solution: P = Í ˙ 2 2 2
Î- sin (p / 6) cos(p / 6)˚ – x + 2y - z = 0 (2)
2 2
a b c2
Ècos(p / 6) - sin (p / 6)˘
fi P¢ = Í ˙ 2 2 2
Î sin (p / 6) cos(p / 6) ˚ – x - y + 2z = 0 (3)
2 2 2
a b c
È1 0 ˘ –1 is
Since PP¢ = Í ˙ fi P¢ = P (a) 6 (b) 8
Î0 1 ˚
(c) 9 (d) infinite
We have Q = PAP¢ = PAP –1 Ans. (d)
fi Q2015 = (PAP –1)2015 = PA2015 P –1 Solution: From (2), (3), we get
Thus, P¢Q2015P = P –1 (PA2015 P –1)P
3 y2 3z 2 = 0 fi y2 = z 2
= (P –1 P) A2015 (P –1P) 2
-
b c2 b2 c2
È0 1 ˘
Now, A = I + B where B = Í ˙ Putting this in (1) we get
Î0 0 ˚
2 r
Since, B = O, we get B = O " r ≥ 2. 2 x2 2 y2 = 0 fi x2 y2
2
- =
a b2 a 2
b2
2015 È1 2015˘
Thus, A = I + 2015B = Í
Î0 1 ˙˚ \ x 2 = y 2 z 2 = k2 (say)
=
a2 b2 c 2
È1 0 0˘
Example 91: If A = Í0 1 1 ˙ , 6A–1 = A2 + cA + dI, fi x = ± ka, y = ± kb, z = ± kc,
Í ˙ where k Œ R.
then (c, d) is ÎÍ0 -2 4 ˚˙
(a) (– 6, 11) (b) (–11, 6) Example 93: If A and B are two matrices such that AB =
(c) (11, 6) (d) (6, 11) B and BA = A, then A2 + B2 is equal to
Ans. (a) (a) 2AB (b) 2BA
(c) A + B (d) AB
Solution: 6A–1 = A2 + cA + dI Ans. (c)
fi 6I = A3 + cA2 + dA
Solution: We have
We have
A2 + B2 = (BA)2 + (AB)2
È1 0 0 ˘ È1 0 0 ˘ È1 0 0˘
= (BA) (BA) + (AB) (AB)
A2 = Í0 1 1 ˙ Í0 1 1 ˙ = Í0 -1 5 ˙
Í ˙Í ˙ Í ˙ = B(AB)A + A(BA)B
ÍÎ0 -2 4 ˙˚ ÍÎ0 -2 4 ˙˚ ÍÎ0 -10 14 ˙˚
= B(BA) + A(AB) = BA + AB = A + B.
5.24 Complete Mathematics—JEE Main

È2 0 1 ˘ È 1 0 0˘
Example 94: If A = Í2 1 3˙ , then A2 – 5A + 6I is (c) Í- a 0 0 ˙ (d) none of these
Í ˙ Í ˙
equal to ÍÎ1 -1 0 ˙˚ ÍÎ ac b 1 ˙˚
Ans. (a)
È 1 -1 -3 ˘ È1 1 -5˘
Solution: Using the formula for inverse of a 3 × 3 trian-
(a) Í -1 -1 -10 ˙ (b) Í -1 -1 4 ˙ gular matrix given in theory, A–1 is the matrix given in (a).
Í ˙ Í ˙
ÎÍ-5 4 4 ˙˚ ÎÍ-3 -10 4 ˙˚ È3 -3 4 ˘
(c) O (d) I Example 96: If A = Í2 -3 4 ˙ then A–1 is
Í ˙
Ans. (a) ÍÎ0 -1 1 ˙˚
È2 0 1 ˘ È2 0 1 ˘ È5 -1 2 ˘ (a) A (b) A2
Solution: A2 = Í2 1 3˙ Í2 1 3 ˙ = Í9 -2 5 ˙ (c) A3 (d) A4
Í ˙Í ˙ Í ˙
ÎÍ1 -1 0 ˙˚ ÍÎ1 -1 0 ˙˚ ÍÎ0 -1 -2 ˙˚ Ans. (c)
Now, A2 – 5A + 6I Solution: To show that A–1 = B ¤ AB = I. We have
È3 -3 4 ˘ È3 -3 4 ˘ È 3 -4 4 ˘
È5 -1 2 ˘ È2 0 1 ˘ È1 0 0 ˘
Í ˙ Í ˙ Í0 1 0 ˙ A2 = Í2 -3 4 ˙ Í2 -3 4 ˙ = Í 0 -1 0 ˙
= Í 9 -2 5 - 5 2 1 3 + 6 Í ˙Í ˙ Í ˙
˙ Í ˙ Í ˙ ÎÍ0 -1 1 ˙˚ ÍÎ0 -1 1 ˙˚ ÍÎ-2 2 -3˙˚
ÍÎ0 -1 -2 ˙˚ ÍÎ1 -1 0 ˙˚ ÍÎ0 0 1 ˙˚
È3 -3 4 ˘ È 3 -4 4 ˘ È 1 * *˘
È5 - 10 + 6 -1 - 0 + 0 2 - 5 + 0 ˘ A(A2) = Í2 -3 4 ˙ Í 0 -1 0 ˙ = Í-2 * *˙
Í ˙Í ˙ Í ˙
= Í9 - 10 + 0 -2 - 5 + 6 5 - 15 + 0 ˙ ÎÍ0 -1 1 ˚˙ ÎÍ-2 2 -3˙˚ ÍÎ * * *˙˚
Í ˙
ÍÎ 0 - 5 + 0 -1 + 5 + 0 -2 - 0 + 6 ˙˚
TIP
È 1 -1 -3 ˘
Í ˙ [Need not evaluate the remaining terms as A3 π I3 ]
= Í -1 -1 -10 ˙
ÍÎ-5 4 4 ˙˚ Next
È3 -4 4˘È3 -4 4˘
È1 0 0˘
A = (A ) (A ) = Í 0 -1 0 ˙ Í 0 -1 0 ˙
4 2 2
Example 95: The inverse of the matrix A = Ía 1 0 ˙ Í ˙Í ˙
Í ˙ ÍÎ-2 2 -3˙˚ ÍÎ-2 2 -3˙˚
is ÍÎb c 1 ˙˚
È1 0 0 ˘
È 1 0 0˘ È1 0 0˘
= Í0 1 0 ˙ = I 3
(a) Í - a 1 0˙ (b) Í- a 0 0 ˙ Í ˙
Í ˙ Í ˙ ÍÎ0 0 1 ˙˚
ÍÎac - b - c 1 ˙˚ ÍÎ b -c 1 ˙˚
Thus, A–1 = A3.

EXERCISES
Concept-based
Straight Objective Type Questions
1. Let A be a 2 × 2 invertible matrix. For which of the (a) |det(A)| £ |a| + |b| + |c| + |d|
following functions det (f (A)) = f (det(A)) is not true? (b) |det(A)| £ (|a| + |b|) (|c| + |d|)
(a) f (x) = x3 (b) f (x) = x–1, x π 0
(c) |det(A)| £ (|a| + |c|) (|b| + |d|)
(c) f (x) = 1 + x (d) f (x) = x– 3, x π 0
(d) |det(A)| £ | a |2 + | b |2 | c |2 + | d |2
Èa b ˘
2. Suppose a, b, c, d Œ C, and let A = Í ˙ . Then 3. Let A, B be two 3 × 3 matrices with entries from
Îc d ˚ real numbers. Which one of the following is true?
which one of the following is not true?
Matrices 5.25
(a) (A + B)3 = A3 + 3 A2B + 3AB2 + B3 (a) C¢ = B¢ – D¢ (b) C + D = B
(b) (AB)2 = O fi AB = O (c) C¢ = B¢ + D¢ (d) none of these
(c) (A + B) (A – B) = A2 – B2 7. Suppose A, B are two 3×3 matrices such that A–1
(d) (A + B)A = BA + A2 exists. Then (A – B)A–1 (A + B) is equal to
4. Suppose A and B are two 3×3 matrices with entries (a) (A + B)A–1) (A – B) (b) A–1 B + B2
from complex numbers such that ABA = I. Which (c) (I – BAB –1) (A – B) (d) (I + BAB –1) (A + B)
one of the following is not true? 8. Let A be 3×3 matrix such that A is orthogonal and
(a) B is invertible (b) B –1 = A2 idempotent, then
(c) A is not invertible (d) A4B2 = I (a) A must be symmetric
5. Let A be a 3×3 matrix with entries from the set of (b) det(A) = –1
real numbers. Suppose the equation AX = B has a (c) A + A–1 = 1
solution for every 3×1 matrix B with entries from (d) none of these
the set of real numbers. Then 9. Suppose A and B are two orthogonal matrices of
the same size such that det(A) + det(B) = 0. Then
(a) A¢ Y = B has no solution
(b) A¢ Y = O fi Y = O (a) A + B = –I (b) A + B = I
(c) AX = O has a non-trivial solution (c) det(A + B) = 0 (d) A + B = O
(d) A is a singular matrix Ê a bˆ
10. Let A = Á , where a, b, c, d Œ R. ad π 0
È cos q i sin q ˘ Ë c d ¯˜
6. Suppose A = Í ˙ for some q Œ R. Let If (a + d)A – A2 = A¢, then
Îi sin q cos q ˚
B, C, D be three 2 × 2 matrices such that AB = AC (a) a = d (b) a = d = 1
– AD, then (c) a + d = 0 (d) a + d = 1

LEVEL 1

Straight Objective Type Questions

Ê 1 kˆ 14. Let A = (aij)3×3 be a matrix with aij Œ C. Let B be


11. Let Sk = Á , k Œ N. Then (S2)n (Sk)–1
Ë 0 1˜¯ a matrix obtained by interchanging two columns of
A. Then det(A + B) is equal to
(where n ΠN) is equal to:
(a) det(A) + det(B)
(a) S2n + k (b) S2n – k
(b) 0
(c) S n (d) S n (c) 2 det(A)
2 + k -1 2 -k

Èa b ˘ (d) det(A) – det(B)


12. Let S be the set of all 2 × 2 real matrices A = Í ˙
Îc d ˚ Ê 1 0ˆ Ê 0 1ˆ
15. If I = Á ,J= Á , and
Ë 0 1˜¯ Ë -1 0˜¯
2
such that a + d = 3 and A¢ = A – 3A. Then
(a) S contains infinite number of elements
Ê cos q sin q ˆ
(b) S = Q B= Á , then B equals
Ë - sin q cos q ˜¯
(c) S contains exactly two elements
(d) S contains exactly 24 elements (a) (cos q ) I + (sin q )J
(b) (sin q)I + (cos q )J
13. Let A = (aij)3×2 be a 3×2 matrix with real entries (b) (cos q )I – (sin q)J
and B = AA¢. Then (d) – (cos q) I + (sin q )J
(a) B –1 is a 3 × 3 matrix 16. If A is both diagonal and skew-symmetric, then
(b) B –1 is a 2 × 2 matrix (a) A is a symmetric matrix
(c) B –1 does not exist (b) A is a null matrix
(d) B –1 exists if and only if exactly one row of A (c) A is a unit matrix
consists of zeros (d) none of these matrix
5.26 Complete Mathematics—JEE Main
17. If A2 – 3A + 2I = 0, then A–1 equals 26. If A is a 2 ¥ 2 unitary matrix, then |A| is equal to
1 1 (a) 1 (b) – 1
(a) (A – 3I) (b) (3I – A)
2 2 (c) + 1 (d) none of these
(c) A + 3I (d) none of these 1 Ê -1 - 3 ˆ
27. If A = , then A–1 – A2 is equal to a
18. If A is a square matrix of order 3 such that A2 = 2 ËÁ 3 -1 ¯˜
2A, then |A|2 is equal to
(a) null matrix (b) invertible matrix
(a) 2 |A| (b) 8 |A|
(c) unit matrix (d) none of these
(c) 16 |A| (d) 0
28. If C is a 3 × 3 matrix satisfying the relation
19. If A is a square matrix then which one of the fol- C2 + C = I, then C –2 is given by
lowing is not a symmetric matrix
(a) 2 C (b) 3 C – I
(a) A + A¢ (b) AA¢
(c) C (d) 2I + C
(c) A¢A (d) A – A¢
29. If A, B and C are three square matrices of the same
20. If A = (aij)3 × 3 where aij = cos (i + j), then size such that B = CA C –1, then CA3C –1 is equal
(a) A is symmetric to
(b) A is skew-symmetric (a) B (b) B2
(c) A is a triangular matrix (c) B3 (d) B9
(d) A is a singular matrix
30. If X is a 2 × 3 matrix such that |X ¢X| π 0, and A
21. If A = (aij)3 × 3 is a matrix satisfying the equation = I – X(X ¢X)–1 X¢ then A2 is equal to
x3 – 3x + 1 = 0, then
(a) A (b) I
(a) A is a unit matrix (c) A–1 (d) none of these
(b) A is singular matrix
(c) A is non-singular matrix Ê p - qˆ
(d) none of these 31. The matrix A = Á is orthogonal if and only
Ë q p ¯˜
22. If A and B are two square matrices of the same size, if
then (A + B)2 = A2+ 2AB + B2 can hold if and only (a) p2 + q2 = 1 (b) p2 = q2
if (c) p2 = q2 + 1 (d) none of these
(a) AB = BA (b) AB + BA = O 32. The values of l for which the matrix
(c) |AB| π 0 (d) |AB| = 0 Êl 0 l ˆ
Èi 0 ˘ Èi 2 ˘ A = Á l 0 - l ˜ is orthogonal is
23. If Í Á ˜
˙ + X = Í3 4 + i ˙ – X, then X is equal
Î3 -i ˚ Î ˚ Ë0 1 0 ¯
to
(a) + 1 (b) ±1/ 3
È0 -1˘ È0 1 ˘
(a) Í ˙ (b) Í ˙ (c) + 1/2 (d) ±1/ 2
Î3 i ˚ Î0 2 + i ˚
33. The values of a for which the matrix
È1 0 ˘ Èi 2 ˘
(c) Í ˙ (d) Í ˙ Ê a a2 - 1 -3 ˆ
Î0 2 - i ˚ Î0 2 + i ˚ Á ˜
A = a +1
Á 2 a 2 + 4˜ is symmetric are
È0 -i ˘ È1 0 ˘ Á
24. If A = Í ˙ , B = Í0 -1˙ , then AB + BA is Ë -3 4a -1 ¯˜
Îi 0 ˚ Î ˚
(a) null matrix (b) unit matrix (a) –1 (b) –2
(c) invertible matrix (d) none of these (c) 3 (d) 2
Ê1 3 2ˆ
È1 2 3˘
34. Let At = Á 2 5 t ˜ , then the value(s) of t for
25. A = Í 1 2 3 ˙ , then A is a nilpotent matrix of Á ˜
Í ˙ Ë4 7 - t - 6¯
ÍÎ-1 -2 -3˙˚
which inverse of At does not exist.
index
(a) 2 (b) 3 (a) – 2, 1 (b) 3, 2
(c) 4 (d) 5 (c) 2, – 3 (d) 3, – 1
Matrices 5.27

È a + ib c + id ˘ 2 2 2 2
(a) O (b) I
35. If A = Í ˙ , where a + b + c + d (c) A (d) A2
Î-c + id a - ib ˚
È1 2 2 ˘
= 1, then A–1 is equal to
43. If 3A = Í2 1 -2 ˙ and AA¢ = I, then x + y is
Èa - ib -c + id ˘ È a - ib c - id ˘ Í ˙
(a) Í ˙ (b) Í-c - id ÍÎ x 2 y ˙˚
Î c + id a + ib ˚ Î a + ib ˙˚ equal to
Èa - ib -c - id ˘ (a) – 3 (b) – 2
(c) Í ˙ (d) none of these (c) – 1 (d) 0
Îc - id a + ib ˚
44. If the system of equations ax + y= 3, x + 2y = 3,
(
È 1 ix
Í2 e + e ) (
- ix 1 ix
2
e - e-ix )˘˙ –1
3x + 4y = 7 is consistent, then value of a is given
by
36. If A = Í ˙ then A exists (a) 2 (b) 1
ÍÎ 2( ) (
Í 1 eix - e-ix 1 eix + e-ix
2
) ˙
˙˚ (c) – 1 (d) 0
45. If the system of equations x + 2y – 3z = 1, (p +
(a) for all real x
2)z = 3, (2p + 1) y + z =2 is inconsistent, then the
(b) for positive real x only value of p is
(c) for negative real x only
(a) – 2 (b) – 1/2
(d) none of these
(c) 0 (d) 2
È ab b2 ˘ 2 46. The system of linear equations x + y + z = 2, 2x +
37. If A = Í ˙ , then A is equal
ÍÎ- a 2 - ab ˙˚ y – z = 3, 3x + 2y + kz = 4 has a unique solution
if
(a) O (b) I (a) k π 0 (b) – 1 < k < 1
(c) – I (d) none of these (c) – 2 < k < 2 (d) k = 0
38. If A is 2 × 2 matrix such that A2 = O, then tr (A) x + 2˘
È 4
is 47. If A = Í ˙ is an invertible matrix, then
Î2 x - 3 x + 1 ˚
(a) 1 (b) – 1
(c) 0 (d) none of these x cannot take value

a b˘ (a) – 1 (b) 2
39. If A = ÈÍ ˙ such that A satisfies the relation (c) 3 (d) none of these
Îc d ˚
48. If A and B are two square matrices of the same
A2 – (a + d)A = O, then inverse of A is order, then which of the following is true.
(a) I (b) A (a) (AB)¢ = A¢ B¢
(c) (a + d)A (d) none of these
(b) (AB)¢ = B¢ A¢
3 2˘
40. If A = ÈÍ –3
˙ , then A is
(c) |AB| = 0 fi |A| = 0 and |B| = 0
Î0 1 ˚ (d) none of these
1 È1 -26 ˘ 1 È-1 -26 ˘ 49. The values of a for which the system of equations
(a) (b)
27 ÍÎ0 -27 ˙˚ 27 ÍÎ 0 -27 ˙˚ x + y + z = 1, x + 2y + 4z = a , x + 4y + 10z
= a 2 is consistent, are given by
1 È1 -26 ˘ 1 È-1 26 ˘ (a) 1, – 2 (b) – 1, 2
(c) (d)
27 ÍÎ0 27 ˙˚ 27 ÍÎ 0 -27˙˚ (c) 1, 2 (d) none of these
41. If A is a skew Hermitian matrix, then the main 50. The system of homogeneous equations
diagonal elements of A are all tx + (t + 1) y + (t – 1) z = 0
(a) purely real (b) positive (t + 1) x + ty + (t + 2) z = 0
(c) negative (d) purely imaginary (t – 1) x + (t + 2) y + tz = 0
1 2 1 has a non-trivial solution for
È ˘
42. If A = Í0 1 -1˙ then A3 – 3A2 – A + 9I is equal (a) three values of t
Í ˙ (b) two values of t
ÎÍ3 -1 1 ˙˚ (c) one value of t
to
(d) infinite number of values of t
5.28 Complete Mathematics—JEE Main
51. If A and B are two 3 ¥ 3 matrices and |A| π 0, then (a) 128 B (b) 32 B
which of the following are not true? (c) 16 B (d) 64 B
(a) |AB| = 0 fi |B| = 0 Ê 2 3 - i -i ˆ
(b) |AB| π 0 fi |B| π 0 53. If A = Á 3 + i p 7 + i˜ , then A is
Á ˜
(c) |A–1| = |A|–1 Ë i 7-i e ¯
(d) |A + A| = 2|A| (a) symmetric (b) Hermitian
Ê i -i ˆ Ê 1 -1ˆ (c) skew Hermitian (d) none of these
52. If A = Á and B = Á , then A8 equals
Ë -i i ˜¯ Ë -1 1 ˜¯

Assertion-Reason Type Questions

54. Statement-1: If A and B are two 3 ¥ 3 matrices Statement-2: If | A| π 0, the system of equations
such that AB = O, then A = O or B = O. AX = B is consistent.
Statement-2: If A, B and X are three 3 ¥ 3 matrices Èa a12 ˘ È x1 ˘ È y1 ˘
such that AX = B, |A| π 0, then X = A–1 B. 58. Let A = Í 11 , X = Í ˙, Y = Í ˙
Îa21 a22 ˙˚ x
Î 2˚ Î y2 ˚
È1 p ˘ 100 È1 100p ˘
55. Statement-1: If A = Í ˙ , then A = Í0 . Statement-1: If A is symmetric, then X ¢AY = Y ¢AX
Î0 1 ˚ Î 1 ˙˚ for each pair of X and Y.
Statement-2: If B is a 2 ¥ 2 matrix such that
Statement-2: If X¢AY = Y¢AX for each pair of X
B2 = O, then (I + B)n = I + nB for each n ΠN.
and Y, then A is symmetric.
È2 1 1 ˘
56. Statement-1: As A = Í0 1 0 ˙ satisfies the equation 59. For any n ≥ 2. Let Mn (R) denote the set of all.
Í ˙ n ¥ n matrices over the set of real numbers.
ÎÍ1 1 2 ˚˙
x3 – 5x2 + 7x – 3 = 0, then A is invertible. Statement-1: If A Œ Mn (R), A π O with
Statement-2: If a square matrix A satisfies the det (A) = 0, then det (Adj A) = 0
equation a0x n + a1 x n – 1 + º + an – 1x + an = 0, Statement-2: For A Œ Mn (R),
and an π 0, then A is invertible. det (Adj A) = (det A)n–1
57. Statement-1: If a, b, c are in A.P. the system of
60. Let B be a 3 × 3 matrix such that (I – B)–1 exists.
equations
3x + 4y + 5z = a (1) Statement-1: If BX = X then X = O
4x + 5y + 6z = b (2) Statement-2: I + B + B2 + …+ Bk
5x + 6y + 7z = c (3) = (I – Bk+1) (I – B)–1 for all k Œ N.
is consistent.

LEVEL 2

Straight Objective Type Questions

- tan q ˆ Ê 1 tan q ˆ -1 Ê a -bˆ 62. If a, b, c π 0 and a + b + c = 0, then the matrix


Ê 1
61. If Á =Á ,
Ë tan q 1 ˜¯ ÁË - tan q 1 ˜¯ Ë b a ˜¯ È 1 ˘
Í1 + a 1 1 ˙
then Í 1 ˙
Í 1 1+ 1 ˙
(a) a=b=1 b
Í 1˙
(b) a = cos2q, b = sin2q Í 1 1 1+ ˙
(c) a = sin2q, b = cos2q ÍÎ c ˙˚
(d) a = 1, b = sin2q is
Matrices 5.29
(a) singular (a) A = O (b) 2A = I
(b) non-singular (c) A is orthogonal (d) none of these
(c) skew-symmetric 70 Suppose A and B are two 3 ¥ 3 non-singular ma-
(d) orthogonal trices such that
63. Suppose matrix A satisfies the equation A2 – 5A + (AB)k = AkBk
7I = O. If A8 = aA + bI, then value of a is
for k = 2015, 2016, 2017, then
(a) 1265 (b) 2599 (a) AB = O
(c) –2599 (d) 0
(b) BA = O
64. Let a, b, g be three real numbers and (c) AB = BA
È 1 cos(b - a ) cos(g - a )˘ (d) AB + BA = O
A = Ícos(a - b ) 1 cos(g - b )˙ 71. Let A be a square matrix of order 3 such that
Í ˙
ÍÎ cos(a - g ) cos(b - g ) 1 ˙˚ |Adj A | = 100, then | A | equals
then (a) ±10 (b) –100
(a) A is singular (c) 100 (d) 25
(b) A is non-singular 72. Let M be a 3 ¥ 3 matrix satisfying
(c) A is orthogonal
È0 ˘ È – 1˘ È 1 ˘ È 1˘ È1˘ È 0 ˘
(d) none of these
M Í1 ˙ = Í 2 ˙ , M Í – 1˙ = Í 1˙ , and M Í1˙ = Í 0 ˙
Ê sin q i cos q ˆ Í ˙ Í ˙ Í ˙ Í ˙ Í˙ Í ˙
65. Let A(q) = Á , then ÎÍ0 ˙˚ ÍÎ 3 ˙˚ ÍÎ 0 ˙˚ ÍÎ – 1˙˚ ÍÎ1˙˚ ÍÎ12 ˙˚
Ë i cos q sin q ˜¯
then sum of the diagonal entries of M is
(a) A(q)–1 = A(–q)
(b) A(q)–1 = A(p – q) (a) 0 (b) –3
(c) A(q)–1 does not exist (c) 6 (d) 9
(d) A(q)2 = A(2q) 73. If A, B and A + B are non-singular matrices, then
66. Let A and B be two matrices such that AB = BA, (A–1 + B –1) [A – A (A + B)–1 A]
then which one of the following does not hold for equals
each n ΠN.
(a) O (b) I
(a) An B = BAn
(c) A (d) B
(b) (AB)n = AnBn
(c) A nB = B nA 74. If A + B is a non-singular matrix, then
(d) A nB n = B nA n A – B – A (A + B)–1A + B (A + B)–1 B
È1 2 1 ˘ equals
67. Let A = Í0 1 -1˙ . The sum of all values of l (a) O (b) I
Í ˙
ÍÎ3 1 1 ˙˚ (c) A (d) B
for which there exists a column vector X π O such 75. If A and B are two matrices such that
that AX = lX, is A + B = AB, then
(a) 0 (b) 1 (a) A = I (b) B = I
(c) 2 (d) 3 (c) AB = BA (d) AB = I
68. Let a, b, c Œ R be such that a + b + c > 0 and a bˆ
abc = 2. Let 76. Let A = ÊÁ such that A3 = O, then a + d equals
Ëc d ˜¯
Èa b c ˘ (a) ad (b) bc
A = Íb c a ˙ (c) 1 (d) 0
Í ˙
ÎÍ c a b ˙˚ Ê0 x 0ˆ
If A2 = I, then value of a3 + b3 + c3 is 77. Let A = Á y 0 - x ˜ , then A3 equals
Á ˜
(a) 7 (b) 2 Ë0 y 0¯
(c) 0 (d) –1
69. If A is a 3 ¥ 3 skew-symmetric matrix with real (a) O (b) x2I
entries and trace of A2 equals zero, then (c) (x2 + y2) I (d) none of these
5.30 Complete Mathematics—JEE Main

Previous Years' AIEEE/JEE Main Questions

Èa b ˘ Èa b ˘ Ê 1 2ˆ Ê a 0ˆ
1. If A = Í and A2 = Í ˙ , then 7. Let A = Á and B = Á a, b Œ N. Then
˙
Îb a ˚ Îb a ˚ Ë 3 4¯˜ Ë 0 b¯˜

(a) a = a2 + b2, b = 2ab (a) there exist infinitely many B’s such that AB = BA
(b) a = a 2 + b 2, b = a 2 – b 2 (b) there cannot exist B such that AB = BA
(c) a = 2ab, b = a2 + b2 (c) there exist more than one but finite number of
(d) a = a2 + b2, b = 2ab [2003] B’s such that AB = BA
(d) there exists exactly one B such that AB = BA
Ê0 0 -1ˆ
[2006]
2. Let A = Á 0 -1 0 ˜ . The only correct statement
Á ˜ È5 5a a˘
Ë -1 0 0¯
8. Let A = Í0 a 5a ˙ . If | A2 | = 25 then | a |
about matrix A is Í ˙
ÍÎ0 0 5 ˙˚
(a) A–1 does not exist equals
(b) A = (–1)I, where I is a unit matrix (a) 52 (b) 1
(c) A is a zero matrix (c) 1/5 (d) 5 [2007]
(d) A2 = I [2004]
9. Let A be a 2 ¥ 2 matrix with real entries. Let I be
Ê 1 -1 1 ˆ Ê 4 2 2ˆ the 2 ¥ 2 identity matrix. Denote by tr(A), the sum
3. Let A = Á 2 1 -3˜ and (10)B = Á -5 0 a ˜ . of diagonal entries of A. Assume that A 2 = I.
Á ˜ Á ˜
Ë1 1 1 ¯ Ë 1 -2 3 ¯ Statement-1: If A π I and A π – I, then
If B is the inverse of matrix A, then a is det (A) = – 1.
Statement-2: If A π I and A π – I, then tr(A) π O.
(a) 2 (b) –1
(c) –2 (d) 5 [2004] [2008]
2
4. If A – A + I = O, then the inverse of A is 10. Let A be a 2 ¥ 2 matrix.
(a) A – I (b) I – A Statement-1: adj (adj A) = A
(c) A + I (d) A [2005] Statement-2: |adj A| = |A| [2009]
È1 0 ˘ È1 0 ˘ 11. Let A be a 2 ¥ 2 matrix with non-zero entries and
5. If A = Í ˙ and I = Í0 1 ˙ , then which one of let A2 = I, where I is 2 ¥ 2 identity matrix. Define
Î1 1 ˚ Î ˚
the following holds for all n ≥ 1, by the principle Tr(A) = Sum of diagonal elements of A,
of mathematical induction |A| = determinant of matrix A.
(a) An = nA + (n – 1)I Statement-1: Tr (A) = 0
(b) An = (2n – 1) A + (n – 1)I Statement-2: |A| = 1 [2010]
(c) An = nA – (n – 1)I 12. The number of 3 ¥ 3 non-singular matrices with
(d) An = (2n – 1) A – (n – 1)I [2005] four entries as 1 and all other entries 0, is
6. If A and B are square matrices of size n ¥ n such (a) 6 (b) at least 7
that (c) less than 4 (d) 5 [2010]
A2 – B2 = (A – B) (A + B), then which of the fol- 13. Consider the system of linear equations
lowing will be always true?
x1 + 2x2 + x3 = 3
(a) either A or B is an identity matrix
(b) A=B 2x1 + 3x2 + x3 = 3
(c) AB = BA 3x1 + 5x2 + 2x3 = 1
(d) either A or B is a zero matrix [2006] The system has
Matrices 5.31
(a) a unique solution È1 a 3˘
(b) no solution 20. If P = Í1 3 3 ˙ is the adjoint of a 3 × 3 matrix
(c) infinite number of solutions Í ˙
(d) exactly 3 solutions [2010] ÍÎ2 4 4 ˙˚
A and |A| = 4, then a is equal to
14. The number of values of k for which the linear
equations (a) 11 (b) 5
4x + ky + 2z = 0 (c) 0 (d) 4 [2013]
kx + 4y + z = 0 21. The number of values of k for which the system of
equations
2x + 2y + z = 0
(k + 1)x + 8y = 4 k
possess a non zero solution is kx + (k + 3)y = 3 k – 1
(a) 0 (b) 3 has no solution, is
(c) 2 (d) 1 [2011] (a) 1 (b) 2
Èw 0 ˘ (c) 3 (d) infinite [2013]
15. If w π 1 is cube root of unity and H = Í ˙ 22. If the system of linear equations
70
then H equals to Î0 w ˚
x1 + 2x2 + 3x3 = 6
(a) O (b) –H x1 + 3x2 + 5x3 = 9
(c) H2 (d) H [2011]
2x1 + 5x2 + ax3 = b
16. If trivial solution is the only solution of the system
is consistent and has infinite number of solutions,
of linear equations:
then
x – ky + z = 0 (a) a = 8, b can be any real number
kx + 3y – kz = 0 (b) b = 15, a can be any real number
3x + y Рz = 0 (c) a ΠR Р{8} and b ΠR Р{15}
then set of all values of k is (d) a = 8, b = 15 [2013, online]
23. If p, q, r are 3 real numbers satisfying the matrix
(a) R – {2, –3} (b) R –{2} equation,
(c) R –{–3} (d) {2, –3} [2011]
È3 4 1 ˘
17. Let A and B be two symmetric matrices of order 3.
[ p q r ] Í3 2 3˙ = [3 0 1]
Statement-1: A(BA) and (AB)A are symmetric Í ˙
matrices ÎÍ2 0 2 ˚˙
then 2p + q – r equal
Statement-2: AB is symmetric if matrix multiplica-
tion of A with B is commutative [2011] (a) –3 (b) –1
(c) 4 (d) 2 [2013, online]
18. Let P and Q be 3 ¥ 3 matrices such that P π Q. 24. Consider the system of equations:
If P3 = Q3 and P2Q = Q2P, then det (P2 + Q2) is x + ay = 0, y + az = 0, and z + ax = 0.
equal to Then the set of all values of a for which the system
(a) 1 (b) 0 has a unique solution is
(c) –1 (d) –2 [2012] (a) R – {1} (b) R – {–1}
Ê 1 0 0ˆ (c) {1, –1} (d) {1, 0, –1}
19. Let A = Á 2 1 0˜ . If u1 and u2 are column ma- [2013, online]
Á ˜
Ë 3 2 1¯ ÏÊ a a12 ˆ ¸
Ê 1ˆ Ê 0ˆ 25. Let S = ÌÁ 11 aij Œ{0, 1, 2}, a11 = a22 ˝
˜
trices such that Au1 = Á 0˜ and Au2 = Á 1˜ , then ÓË a21 a22 ¯ ˛
Á ˜ Á ˜ Then the number of non-singular matrices in the
u + u is equal to Ë 0¯ Ë 0¯
1 2 set S is
Ê -1ˆ Ê -1ˆ (a) 27 (b) 24
(a) Á 1˜ (b) Á -1˜ (c) 10 (d) 20 [2013, online]
Á ˜ Á ˜
Ë -1¯ Ë 0¯ 26. The matrix A2 + 4A – 5I, where I is the identity
Ê 1ˆ È1 2 ˘
Ê -1ˆ matrix and A = Í ˙ , equals:
(c) Á -1˜ (d) Á 1˜ [2012] Î4 -3˚
Á ˜ Á ˜
Ë -1¯ Ë 0¯
5.32 Complete Mathematics—JEE Main

È2 1 ˘ È0 -1˘ È1 2 2˘
(a) 4 Í ˙ (b) 4 Í ˙
Î2 0 ˚ Î2 2 ˚ 33. If A = Í 2 1 -2 ˙ is a matrix satisfying the equa-
Í ˙
È2 1 ˘ È1 1 ˘ ÍÎ a 2 b ˙˚
(c) 32 Í ˙ (d) 32 Í ˙
Î2 0 ˚ Î1 0 ˚ tion AAT = 9I, where I is 3 ¥ 3 identity matrix, then
the ordered pair (a, b) is equal to:
[2013, online]
(a) (2, –1) (b) (–2, 1)
27. Let A, other than I or –I, be a 2 × 2 real matrix
such that A2 = I, I being the unit matrix. Let Tr(A) (c) (2, 1) (d) (–2, –1) [2015]
be the sum of diagonal elements of A. 34. If A is 3 ¥ 3 matrix such that |5adj A| = 5, then |A|
Statement-1: Tr(A) = 0 is equal to:
Statement-2: det(A) = –1 [2013, online] 1
(a) ± (b) ± 5
28. If A is an 3 × 3 non-singular matrix such that AA¢ 5 1
= A¢A and B = A–1 A¢, then BB¢ equals: (c) ± 1 (d) ± [2015, online]
25
(a) I (b) B –1 0 -1˘
35. If A = È , then which one of the following
–1
(c) (B )¢ (d) I + B [2014] Í 1 0˙
Î ˚
29. If B is a 3 × 3 matrix such that B2 = O, then statement is not correct?
det [(I + B)50 – 50B] is equal to: (a) A4 – I = A2 + I (b) A3 – I = A(A – I)
(c) A2 + I = A(A2 – I) (d) A3 + I = A(A3 – I)
(a) 1 (b) 2 [2015, online]
(c) 3 (d) 50 [2014, online]
È5a -b ˘
30. Let A be a3 × 3 matrix such that 36. If A = Í ˙ and Aadj A = AA¢, then 5a + b
Î3 2˚
È1 2 3˘ È 0 0 1˘ is equal to
A Í0 2 3˙ = Í 1 0 0˙ (a) –1 (b) 5
Í ˙ Í ˙
ÎÍ0 1 1˚˙ ÎÍ0 1 0 ˙˚ (c) 4 (d) 13 [2016]
then A–1 is : È 3 1 ˘
Í ˙ È1 1˘
2 2 ˙
È3 1 2 ˘ È3 2 1 ˘ 37. If P = Í ,A= Í ˙ and Q = PAP ¢, then
Í 1 ˙ Î0 1˚
(a) Í3 0 2 ˙ (b) Í3 2 0 ˙
3
Í ˙ Í ˙ Í- ˙
Î 2 2 ˚
ÎÍ1 0 1 ˙˚ ÎÍ1 1 0 ˙˚
P ¢Q2015 P is:
È 0 1 3˘ È 1 2 3˘ È0 2015˘ È2015 0 ˘
(c) Í0 2 3˙ (d) Í0 1 1˙ (a) Í
0 ˙˚
(b) Í
2015˙˚
Í ˙ Í ˙ Î0 Î 1
ÎÍ1 1 1˙˚ ÎÍ0 2 3˙˚
È1 2015˘ È2015 1 ˘
[2014, online] (c) Í (d) Í
Î0 1 ˙˚ Î 0 2015˙˚
y
È ˘
È1 2 x ˘ [2016, online]
31. If A = Í ˙ and B = Í x ˙ be such that AB
Î3 -1 2 ˚ Í ˙ È-4 -1˘
ÎÍ1 ˚˙ 38. If A = Í ˙ , then the determinant of the matrix
È6 ˘ Î3 1˚
= Í ˙ , then:
Î8 ˚ (A2016 – 2A2015 – A2014) is:
(a) y = 2x (b) y = –2x (a) –175 (b) 2014
(c) y = x (d) y = –x [2014, online] (c) 2016 (d) –25 [2016, online]
39. Let A be a 3 ¥ 3 matrix such that A2 – 5A + 7I = 0.
32. Let A and B be any two 3×3 matrices. If A is sym-
1
metric and B is skew-symmetric, then the matrix Statement-1: A–1 = (5I – A)
AB – BA is: 7
Statement-2: The polynomial A3 –2A2 – 3A + I can
(a) skew-symmetric be reduced to 5(A + 4I). Then
(b) symmetric (a) Both the statements are true.
(c) neither symmetric nor skew-symmetric (b) Both the statements are false.
(d) I or –I, where I is an identity matrix. (c) Statement-1 is true, but Statement-2 is false.
[2014, online] (d) Statement-1 is false, but Statement-2 is true.
[2016, online]
Matrices 5.33

Previous Years' B-Architecture Entrance


Examination Questions

1. If A and B are square matrices of the same order È1 k ˘


then which one of the following is always true? 8. Let Sk = Í ˙ k Œ N, where N is the set of
(a) (A + B)–1 = A–1 + B –1 Î0 1 ˚
(b) adj (AB) = (adj B) (adj A) natural numbers, then (S2)n (Sk)–1, for n Œ N is:
(c) A and B are non-zero and |AB| = 0 ¤ |A| = 0 and (a) S2n + k –1 (b) S2n + k -1
|B| = 0
(d) (AB)–1 = A–1 B –1 [2006] (c) S2n - k (d) S2n – k [2014]
È1 1 0 ˘ 1 1˘ È b1 b2 ˘
2. Let A = Í0 1 0 ˙ , and let I denote the 3 × 3 9. Let A = ÈÍ and B = Íb b ˙ . If 10A +
10
Í ˙ Î 0 1˙˚ Î 3 4˚
ÍÎ0 0 1 ˙˚
Adj(A10) = B, then b1 + b2 + b3 + b4 is equal to:
identity matrix. Then 2A2 – A3 =
(a) A + I (b) A – I (a) 91 (b) 92
(c) I – A (d) A [2008] (c) 111 (d) 112 [2015]
3. Let A and B be 2 × 2 matrices with real entries.
Let I be the 2 × 2 identity matrix. Denote by tr(A) È 1 -2 4˘
the sum of diagonal entries of A. 10. If for a matrix A, |A| = 6 and adj A = Í 4 1 1 ˙ ,
Í ˙
Statement-1: AB – BA π I then k is equal to ÎÍ-1 k 0 ˙˚
Statement-2: tr(A + B) = tr(A) + tr(B) and
(a) 0 (b) 1
tr(AB) = tr(BA) [2009]
(c) 2 (d) –1 [2016]
Èa b ˘
4. Let A = Í ˙ be a 2 × 2 real matrix. If A – a I
Îc d ˚
is invertible for every real number a, then
Answers
(a) bc > 0 (b) bc = 0 Concept-based
(c) bc < min Ê0, 1 ad ˆ (d) a = 0 [2010] 1. (c) 2. (a) 3. (d) 4. (c)
Ë 2 ¯
5. (b) 6. (c) 7. (a) 8. (a)
5. Let A and B be two 2 × 2 matrices.
9. (c) 10. (b)
Statement-1: A (adj A) = |A| I2
Statement-2: adj(AB) = (adj A)(adj B)
Level 1
[2011] 11. (b) 12. (b) 13. (c) 14. (b)
Èa b ˘ 15. (a) 16. (b) 17. (b) 18. (b)
6. Let A = Í ˙ , be a 2 × 2 matrix, where a, b,
Îc d ˚ 19. (d) 20. (a) 21. (c) 22. (a)
c, d take values 0 or 1 only. The number of such 23. (b) 24. (a) 25. (a) 26. (d)
matrices which have inverses is: 27. (a) 28. (d) 29. (c) 30. (a)
(a) 8 (c) 7
31. (a) 32. (d) 33. (d) 34. (c)
(c) 6 (d) 5 [2012]
35. (c) 36. (a) 37. (a) 38. (c)
Èa b ˘ 39. (d) 40. (c) 41. (d) 42. (a)
7. Let S be the set of all real matrices A = Í ˙
Îc d ˚ 43. (a) 44. (a) 45. (a) 46. (a)
2
such that a + d = 2 and A¢ = A – 2A. Then S: 47. (d) 48. (b) 49. (c) 50. (c)
(a) is an empty set 51. (d) 52. (a) 53. (b) 54. (d)
(b) has exactly one element
55. (a) 56. (a) 57. (b) 58. (b)
(c) has exactly two elements
(d) has exactly four elements [2013] 59. (a) 60. (b)
5.34 Complete Mathematics—JEE Main
Level 2 5. Let X1, X2 X3 be solutions of AX = B, when

61. (b) 62. (b) 63. (a) 64. (a) Ê1 ˆ Ê 0ˆ Ê 0ˆ


B = Á 0˜ , Á1 ˜ , Á 0˜ respectively. Let C be the 3 × 3
65. (b) 66. (c) 67. (d) 68. (a) Á ˜ Á ˜ Á ˜
Ë 0¯ Ë 0¯ Ë1 ¯
69. (a) 70. (c) 71. (a) 72. (d)
matrix [X1 X2 X3], then AC = I fi A is invertible.
73. (b) 74. (a) 75. (c) 76. (d)
fi A¢ is invertible.
77. (a)
Thus A¢Y = O fi Y = O.
Previous Years' AIEEE/JEE Main Questions 6. det(A) = cos2q + sin2q = 1 π 0
1. (a) 2. (d) 3. (d) 4. (b) \ A–1 exist.
Now, AB = A(C – D)
5. (c) 6. (c) 7. (a) 8. (c)
fi A–1(AB) = A–1 (A(C – D))
9. (c) 10. (b) 11. (c) 12. (b)
B = C – D fi C = B + D
13. (b) 14. (c) 15. (d) 16. (a) fi C¢ = (B + D)¢ = B¢ + D¢
17. (b) 18. (b) 19. (c) 20. (a)
7. (A – B) A–1 (A + B)
21. (a) 22. (d) 23. (a) 24. (b)
= (I – BA–1) (A + B)
25. (d) 26. (a) 27. (a) 28. (a)
= A + B – BA–1 A – BA–1 B
29. (a) 30. (a) 31. (a) 32. (b) = A – BA–1 B
33. (d) 34. (a) 35. (c) 36. (b) = A + B – BA–1 A – BA–1 B
37. (c) 38. (d) 39. (a) = (A + B) – BA–1 (A + B)
Previous Years' B-Architecture Entrance = (A + B) (I – BA–1)
Examination Questions = (A + B)A–1 (A – B)
1. (b) 2. (d) 3. (a) 4. (c) 8. AA¢ = I and A2 = I
2
5. (c) 6. (c) 7. (a) 8. (d) \ AA¢ = A fi A–1 (AA¢) = A–1A2
9. (d) 10. (c) fi A¢ = A
9. As A, B are orthogonal matrices, AA¢ = BB¢ = I,
Hints and Solutions det(A) π 0, det(B) π 0. Let C = A + B.

Concept-based C ¢ = A¢ + B ¢ fi AC ¢ = AA¢ + AB ¢ = I + AB ¢
fi AC ¢B = B + AB ¢B = B + A
1. If f (x) = 1 + x, then det f (A) = 1 + det(A) may not
hold. Take A = –I. Now,
det(A + B) = det(AC ¢B) = det(A) det(C ¢) det(B)
2. |det(A)| = |ad – bc| £ |a||d| + |b||c|
= det(A) det(C) (– det(A))
£ (|a| + |b|) (|c| + |d|)
fi det(C) = – (det(A))2 det(C) = – det(C)
Also, |det(A)| £ (|a| + |c|) (|b| + |d|)
fi det(C) = 0
For truth of (c), see Example 6

[ Ê a b ˆ Ê a b ˆ Ê a + bc b(a + d )ˆ
2
È 5 -1˘ ˘ =Á
(a) does not hold in general. Take A = Í 10. A2 = Á ˜ Á ˜
˙˙
Î10 2 ˚ ˚ Ë c d ¯ Ë c d ¯ Ë c(a + d ) bc + d 2 ˜¯
3. (a), (b), (c) cannot hold in general unless AB = BA. \ (a + d)A – A2 = A¢
4. det(ABA) = 1 Ê ad - bc 0 ˆ Ê a bˆ
fi = fi ad = a, d
fi det(A) det(B) det(A) = 1 ËÁ 0 ad - bc¯˜ ÁË c d ¯˜
fi det(A) π 0, det(B) π 0 As ad π 0, we get a = d = 1.
fi Both A–1 and B –1 exist
Level 1
Also ABA = I fi BA = A–1 fi B = A–1A–1 = A–2
Ê 1 2ˆ
fi B2 = (A–2)2 = A– 4 fi A 4B 2 = I 11. S2 = Á =I+B where
Ë 0 1˜¯
Matrices 5.35

Ê 1 0ˆ Ê 0 2ˆ \ B –1 does not exist.


I= Á and B = Á
Ë 0 1˜¯ Ë 0 0˜¯ 14. In A + B two rows are identical.
2
Note that B = O, therefore, B = O r
" r ≥ 2. 15. B = (cos q)I + (sin q)J.
16. A is a diagonal matrix, aij = 0 " i π j
Thus, (S2)n = (I + B)n = I + nB
As A is a skew-symmetric matrix,
Ê 1 2 nˆ aii = 0 " i

Ë 0 1 ˜¯
\ A=O
Ê 1 -k ˆ 17. A2 – 3A + 2I fi 2I = A(3I – A)
Also, Sk-1 = Á
Ë 0 1 ˜¯

Ê 1 2 nˆ Ê 1 - k ˆ
\ (S2)n(Sk)–1 = Á
fi I= A { 1
2 }
(3I - A)

Ë 0 1 ˜¯ ÁË 0 1 ˜¯ 1
fi A –1 = (3I – A)
2
Ê 1 2n - k ˆ
=Á = S2 n - k 18. | A | = | 2A | = 23 | A | fi | A |2 = 8 | A |
2
Ë0 1 ˜¯
19. (A – A¢)¢ = A¢ – (A¢)¢ = A¢ – A = – (A – A¢)
Èa b ˘ Èa b ˘ \ A – A¢ is skew-symmetric matrix.
12. A2 = Í ˙Í ˙
Îc d ˚ Îc d ˚ Ê cos 2 cos 3 cos 4ˆ
È a 2 + bc b(a + d )˘ 20. A = Á cos 3 cos 4 cos 5˜ = A¢
= Í Á ˜
˙ Ë cos 4 cos 5 cos 6¯
ÍÎc(a + d ) bc + d 2 ˙˚
21. A3 – 3A + I = 0 fi I = A(3I – A2)
Èa 2 + bc 3b ˘ fi A is invertible and hence non-singular.
= Í ˙
ÍÎ 3c bc + d 2 ˙˚ 22. (A + B)2 = (A + B) (A + B) = A2 + BA + AB + B2
Èa 2 - 3a + bc 0 ˘ \ (A + B)2 = A2 + 2AB + B2 if and only if
2
\ A – 3A = Í 2
˙ AB = BA
ÍÎ 0 d - 3d + bc ˙˚
Èi 2 ˘ È i 0 ˘ È0 2 ˘
Since A¢ = A2 – 3A, we get 23. 2X = Í ˙-Í ˙=Í ˙
Î3 4 + i ˚ Î3 -i ˚ Î0 4 + 2i ˚
Èa c ˘ Èa - 3a + bc ˘
2
0
Íb d ˙ = Í ˙ È0 1 ˘
Î ˚ ÍÎ 0 2
d - 3d + bc ˙˚ fi X= Í ˙
Î0 2 + i ˚
fi b = 0, c = 0, a = a2 – 3a, d = d2 – 3d
È0 -i ˘ È1 0 ˘ È0 i ˘
fi a (a – 4) = 0 and d(d – 4) = 0 24. AB = Í ˙Í ˙=Í ˙
Î i 0 ˚ Î0 -1˚ Î i 0 ˚
fi a = 0 or a = 4 and d = 0 or d = 4
È1 0 ˘ È0 -i ˘ È 0 -i ˘
But then a + d cannot be 3. and BA = Í ˙Í ˙=Í ˙
Î0 -1˚ Î i 0 ˚ Î-i 0 ˚
\ S =f
Thus, AB + BA = O
13. Let
Ê a pˆ È 1 2 3 ˘ È 1 2 3˘ È0 0 0 ˘
A = Á b q ˜ , then 25. A2 = Í 1 2 3 ˙ Í 1 2 3˙ = Í0 0 0 ˙
Í ˙Í ˙ Í ˙
Á ˜
Ëc r¯ ÎÍ-1 -2 -3˙˚ ÍÎ-1 -2 3˙˚ ÍÎ0 0 0 ˙˚
\ A is a nilpotent matrix of index 2
Ê a 2 + p2 ab + pq ac + pr ˆ
Á ˜ Èa b ˘ Èa c ˘
B = AA¢ = Á ab + pq b2 + q 2 bc + qr ˜ 26. Let A = Í ˙ , A* = Íb d ˙
Îc d ˚ Î ˚
ÁË ac + pr bc + qr c 2 + r 2 ˜¯
AA* = I fi | A | | A*| = 1
a p 0 a b c fi (ad – bc) (a d - b c) = 1
det (B) = b q 0 p q r = 0 fi | ad – bc |2 = 1
c r 0 0 0 0 fi ad Рbc = eiq for some q ΠR.
5.36 Complete Mathematics—JEE Main

Ê -1 / 2 - 3 / 2ˆ Ê cos(4p / 3) sin (4p / 3) ˆ È ab b2 ˘ È ab b2 ˘


27. A = Á ˜ =Á ˜ 37. A2 = Í ˙Í ˙
Ë 3 / 2 -1 / 2 ¯ Ë - sin (4p / 3) cos(4p / 3)¯ ÍÎ- a 2 - ab ˙˚ ÍÎ- a 2 - ab ˙˚
3 2 –1
Note that A = I fi A – A = O Èa 2 b2 - a 2 b2 ab3 - ab3 ˘
28. C2 + C = I fi C(C + I) = I = Í ˙ =O
ÍÎ - a3 b + a3 b - a 2 b2 + a 2 b2 ˙˚
fi C –1 = C + I
Thus, C –2 = (C + I)2 = C2 + 2C + I Èa b ˘ Èa 2 + bc ab + bd ˘
38. Let A = Í ˙ , then A 2
= Í 2
˙
= I – C + 2C + I = 2I + C Îc d ˚ ÎÍ ac + cd bc + d ˙˚
29. CA3C–1 = (CAC –1) (CAC –1) (CAC –1) = B3 Now, A2 = O
30. A2 = (I – X(X¢X) –1 X¢)2 fi a2 + bc = 0, (a + d) b = 0
= I – 2X (X¢X)–1 X¢ + X(X¢X)–1 X¢ X(X¢X)–1 X¢ (a + d) c = 0, bc + d2 = 0
= I – 2X(X¢X)–1 X¢ + X(X¢X)–1 X¢ Suppose a + d π 0, then b = 0, c = 0.
= I – X(X¢X)–1 X¢ = A
\ a2 = 0, d2 = 0 fi a = 0, d = 0
31. AA¢ = I Contradiction.
p - q ˆ Ê p q ˆ Ê 1 0ˆ
fi ÊÁ = Thus, a + d = 0 we get tr(A) = 0
Ë q p ˜¯ ÁË -q p˜¯ ÁË 0 1˜¯ 39. Suppose A–1 exists, then
Ê p2 + q 2 0 ˆ Ê 1 0ˆ A–1 A2 – (a + d)A–1 A = O
fi Á =Á
Ë 0 p +q ¯
2 2˜ Ë 0 1˜¯ fi A – (a + d)I = O

\ p2 + q2 = 1 È- d b ˘
fi Í ˙ =O
32. AA¢ = I Î c -a˚
Êl 0 l ˆ Êl l 0ˆ Ê 1 0 0ˆ fi a = b = c = d = 0 fi | A | = 0
A contradiction. Thus, A–1 does not exist.
fi Á l 0 - l ˜ Á 0 0 1˜ = Á 0 1 0˜
Á ˜Á ˜ Á ˜ 1 È1 -2 ˘
Ë 0 1 0 ¯ Ël - l 0¯ Ë 0 0 1¯ 40. We have A–1 =
3 ÍÎ0 3 ˙˚
Ê 2l 2 0 0 ˆ Ê 1 0 0ˆ
Á ˜ 1 È1 -2 ˘ È1 -2 ˘ È1 -2 ˘
fi Á 0 2l 2
0 ˜ = Á 0 1 0˜ fi A–3 = Í0 3 ˙ Í0 3 ˙ Í0 3 ˙
Á ˜ 27 Î ˚Î ˚Î ˚
Á 0 0 1˜¯ Ë 0 0 1¯
Ë 1 È1 -8˘ È1 -2 ˘ 1 È1 -26 ˘
= =
\ 2l2 = 1 fi l = ± 1/ 2 27 ÍÎ0 9 ˙˚ ÍÎ0 3 ˙˚ 27 ÍÎ0 27 ˙˚
33. A¢ = A
41. A* = –A = fi ( A)¢ = –A fi aii = –aii " i
Ê a a + 1 -3ˆ Ê a a2 - 1 -3 ˆ
Á ˜ fi 2Re(aii) = 0 fi aii is purely imaginary.
Á a2 - 1 2 ˜
4a = Á a + 1 2 2
a + 4˜
fi Á ˜ È1 2 1 ˘ È1 2 1˘ È4 3 0˘
ÁË -3 a 2 + 4 -1˜¯ ÁË -3 4a -1 ˜¯
42. A = Í0 1 -1˙ Í0
2
1 -1˙ = Í-3 2 -2 ˙
Í ˙Í ˙ Í ˙
fi a + 1 = a2 – 1 and 4a = a2 + 4 ÍÎ3 -1 1 ˙˚ ÎÍ3 -1 1 ˙˚ ÍÎ 6 4 5 ˙˚
fi a + 1 = 4a – 4 – 1 fi 6 – 3a fi a = 2. È 4 3 0 ˘ È1 2 1˘ È4 11 1 ˘
34. Since At–1 does not exist, | At | = 0 A3 = Í-3 2 -2 ˙ Í0 1 -1˙ = Í-9 -2 -7˙
fi (–30 – t(7 – t)) – 3(–12 – 4t) + 2(14 – 2t – 20) Í ˙Í ˙ Í ˙
ÍÎ 6 4 5 ˙˚ ÍÎ3 -1 1 ˙˚ ÍÎ 21 11 7 ˙˚
=0
fi t2 + t – 6 = 0 fi t = –3, 2 Now, A3 – 3A2 – A + 9I
Èa - ib -c - id ˘ È 4 11 1 ˘ È4 3 0˘
35. A–1 = Í ˙
Îc - id a + ib ˚ = Í-9 -2 -7˙ - 3 Í-3 2 -2 ˙
Í ˙ Í ˙
1 ÍÎ 21 11 7 ˙˚ ÍÎ 6 4 5 ˙˚
36. | A | = [(eix + e–ix)2 – (eix – e–ix)2]
4 È1 2 1 ˘ È 9 0 0 ˘
1
= (4eix e–ix) = 1 π 0 – Í0 1 -1˙ + Í0 9 0 ˙ = O
4 Í ˙ Í ˙
Thus, A–1 exists for all x Œ R. ÎÍ3 -1 1 ˙˚ ÎÍ0 0 9 ˙˚
Matrices 5.37

È1 2 x ˘ t t +1 t -1
43. (3A)¢ = Í2 1 2 ˙ D = t +1 t t+2
Í ˙
ÍÎ2 -2 y ˙˚ t -1 t + 2 t
Now, 9I = (3A) (3A)¢ Applying R3 Æ R3 – R2, R2 Æ R2 – R1, we get
È 1 2 2 ˘ È1 2 x ˘ t t +1 t -1 2t + 1 t + 1 t - 1
= Í2 1 -2 ˙ Í2 1 2 ˙ D= 1 -1 3 = 0 -1 3
Í ˙Í ˙
ÎÍ x 2 y ˙˚ ÎÍ2 -2 y ˙˚ -2 2 -2 0 2 -2

È 9 0 x + 4 + 2y ˘ [using C1 Æ C1 + C2]
Í ˙ = (2t + 1) (2 – 6) = – 4(2t – 1)
= Í 0 9 2 x + 2 - 2 y˙
Note that D = 0 for t = –1/2 i.e., for just for one
Í x + 4 + 2 y 2 x + 2 - 2 y x 2 + 4 + y2 ˙
Î ˚ value of t.
fi x + 2y + 4 = 0 51. Choice (d) is not true as | A + A | = | 2A | = 8| A |
2x – 2y + 2 = 0 52. A = iB
x2 + y2 + 4 = 9 Ê 1 -1ˆ Ê 1 -1ˆ
fi A2 = i2B2 = (–1) Á = –2B
fi x = –2, y = –1 Ë -1 1 ˜¯ ÁË -1 1 ˜¯
44. Solving x + 2y = 3, 3x + 4y = 7, we get fi A4 = (–2B)2 = 4B2 = 4(2B) = 8B
x = 1, y = 1
\ a + 1 = 3 fi a = 2. fi A8 = (8B)2 = 64B2 = 64(2B) = 128B
53. We have
-1 3
45. If p π –2, then z = Ê 2 3+i i ˆ
2 p+2
A¢ = Á 3 - i p 7 - i˜
1 Á ˜
and y = corresponding to this value we can Ë -i 7 + i e ¯
get x. p +2
Ê 2 3 - i -i ˆ
If p = – 1/2, we get z = 2. fi A*= ( A¢ ) = Á 3 + i p 7 - i˜ = A
Corresponding to which we get infinite number of Á ˜
Ë i 7+i e ¯
value of x and y.
Thus A is a Hermitian matrix.
For p = –2, the system of equations is inconsistent. 54. Statement-2 is true since |A| π 0, implies A–1 exists.
46. The system of equations will have a unique solution \ AX = B
if
fi A– 1 (AX) = A– 1 B
1 1 1 1 0 0 fi (A–1 A)X = A–1 B
2 1 -1 π 0 fi 2 -1 -3 π 0 fi IX = A–1 B
3 2 k 3 -1 k - 3 fi X = A–1 B
[using C1 Æ C1 – C1, C3 Æ C3 – C1] That statement-1 is false can be seen by the fol-
fi (k – 3) (–1) –3 π 0 fi k π 0. lowing example.
47. | A | π 0 fi 4(x + 1) – (2x – 3) (x + 2) π 0. È0 0 1 ˘ È0 1 0 ˘
fi 2x2 – 3x – 10 π 0 Let A = Í0 0 0 ˙ and B = Í0 0 0 ˙ then
Í ˙ Í ˙
ÍÎ0 0 0 ˙˚ ÍÎ0 0 0 ˙˚
fi x π 3 ± 89
4 AB = O but neither A = O nor B = O
48. See Theory. 55. Since B commutes with I, we can use binomial
theorem to obtain
49. Subtracting first equation from the second and third,
we get Ê nˆ Ê nˆ Ê nˆ
(I + B)n = I + Á ˜ B + Á ˜ B2 + º + Á ˜ Bn
y + 3z = a – 1 Ë1 ¯ Ë 2¯ Ë n¯
3y + 9z = a2 – 1 Since B2 = O, we get Br = O " r ≥ 2.
Thus,
\ a2 – 1 = 3(a – 1) fi a = 1 or a = 2
(I + B)n = I + nB
50. The system of equations will have a non-trivial
Now,
solution if D = 0 where
5.38 Complete Mathematics—JEE Main

È0 p ˘ \ Statement-2 is true
A = I + B where B = Í ˙ Also, BX = X fi (I – B)X = O
Î0 0 ˚
2
Since B = O, we get fi X = (I – B)–1 O = O
È1 100p ˘ Thus, both the statements are true but statement-2
A100 =I + 100B = Í
Î0 1 ˙˚ is not a correct reason for it.
n
56. Since a0 A + a1 A n –1
+ º + an – 1 A + an I = O, Level 2
and an π 0, we get AB = I where
Ê a -bˆ Ê 1 - tan q ˆ Ê cos q - sin q cos q ˆ
2

a a an – 1 61. Á ˜ =Á ˜ Á ˜
B = – 0 An – 1 – 1 An – 2 – ! – I Ë b a ¯ Ë tan q 1 ¯ Ë sin q cos q cos2 q ¯
an an an
Ê cos 2q - sin 2q ˆ
fi B = A–1. =Á
Ë sin 2q cos 2q ˜¯
57. If |A| π 0, A is invertible and we can write AX = B
as X = A–1 B. fi a = cos2q, b = sin2q
\ AX = B has a unique solution and hence is con- 1 + 1/ a 1 1
sistent. 62.
1 1 +1 / b 1
Subtracting (2) from (3) and (1) and (2), we get 1 1 1 + 1/ c
the system of equations as
3x + 4y + 5z = a (4) a +1 b c
1
x+y+z= b–a (5) = a b +1 c
x+y+z= c–b (6) abc
a b c +1
As a, b, c are in A.P. b – a = c – b
\ the last two equations are identical. Using C1 Æ C1 + C2 + C3, we get
From (4) and (5) we obtain 1 b c
x = 4b – 5a + k 1
| A| = (a + b + c + 1) 1 b + 1 c
y = 4a – 3b – 2k abc
1 b c +1
z= k
where k is an arbitrary complex number. Thus, the Using R2 Æ R2 – R1, R3 Æ R3 – R1, we get
system of equations in statement-1 is consistent.
1 b c
58. Suppose A is symmetric, then A¢ = A 1
Since X¢AY is a 1 ¥ 1, matrix, | A| = (a + b + c + 1) 0 1 0
abc
X¢AY = (X¢AY)¢ = Y¢A¢(X¢)¢ = Y¢AX 0 0 1
1
Next, suppose X¢AY = Y¢AX for each pair of X and =(a + b + c + 1) π 0
Y. abc
È1 ˘ È0 ˘ 63. A2 = 5A – 7I
Let E1 = Í ˙ , E2 = Í ˙ .
Î0 ˚ Î1 ˚ fi A4 = (A2)2 = (5A – 7I ) (5A – 7I )
= 25 A2 – 70A + 49I
Taking X = E1 and Y = E2, then
= 25 (5A – 7I ) – 70A + 49I
E1¢AE2 = E2¢AE1 fi a12 = a21
= 55A – 126I
Thus, A is symmetric.
59. For truth of statement-2, see theory fi A8 = (A4)2 = (55A – 126I ) (55A – 126I )
As det (Adj A) = (det A)n – 1 = O if det A = O = 3025 A2 – 13860A + 15876 I
Thus, statement-1 is true and statement-2 is correct
reason for it. = 3025 (5A – 7I) – 13860A + 158766I
60. We have
(I + B + B2 + … + Bk) (I – B) = 1265A – 5299 I
= (I – B) + (B – B2) + (B2 – B3) + … Thus, a = 1265.
+ (Bk – Bk + 1) 1 cos ( b - a ) cos (g - a )
k +1
=I – B
64. |A| = cos (a - b ) 1 cos (g - b )
fi I + B + B2 + … + Bk = (I – Bk + 1) (I – B)–1 cos (a - g ) cos ( b - g ) 1
Matrices 5.39

cos a sin a 0 cos a sin a 0 We have


= cos b sin b 0 cos b sin b 0 =0 a3 + b3 + c3 – 3abc
cos g cos g 0 cos g sin g 0 = (a + b + c) (a2 + b2 + c2 – bc – ca – ab)
fi A is singular. =1
fi a3 + b3 + c3 = 7
65. | A(q)| = sin2q – i2 cos2q = 1.
Èa b c˘
Ê sin q - i cos q ˆ
\ A(q) – 1 = Á 69. Let A = Íb d e˙
Ë - i cos q sin q ˜¯ Í ˙
ÍÎ c e f ˙˚
= A(p – q )
Trace of A2 = (a2 + b2 + c2) + (b2 + d2 + e2)
66. An B = An – 1 (AB) = An – 1 (BA)
+ (c2 + e2 + f 2)
= An – 2 (AB) A
fi a2 + 2b2 + 2c2 + d2 + 2e2 + f 2 = 0
= An – 2 (BA) A = An – 2 BA2
As a, b, c, d, e, f are real numbers, we get
Continuing in this way, we get
a = b = c = d = e = f = 0.
AnB = BAn
n n n n n \A=0
Similarly, AB = B A, (AB) = A B
70. Let k = 2015. We are given
and AnBn = BnAn
n n (AB)k = Ak Bk and (AB)k + 1 = Ak +1
Bk +1
However, A B = B A does not hold.
We have
67. AX = l X
Ak + 1 B k+1
= (AB)k + 1 = (AB)k (AB) = (AkBk) (AB)
fi (A – l I) X = 0 fi Ak + 1 Bk + 1 = Ak (Bk A) B
As X π 0, |A – l I | = 0 As A, B are non-singular matrices, we get
1- l 2 1
ABk = Bk A.
fi 0 1- l -1 = 0
Similarly, using (AB)k + 1 = Ak + 1 Bk + 1
3 1 1- l
and (AB)k + 2 = Ak + 2 Bk + 2
1- l -1 2 1 we get AB k + 1 = B k + 1 A
fi (1 – l) +3 =0
1 1- l 1- l -1 Now, AB k + 1 = B k + 1 A = B(B k A)
2
= (1 – l) [(1 – l) + 1] + 3[– 2 – (1 – l)] = 0 = B(AB k)
3
= (1 – l) – 2(1 – l) – 6 = 0 fi (AB) B k = (BA) B k
3 2
fi l – 3l + 5l + 7 = 0 (1) As B is invertible, we get AB = BA.
If l1, l2, l3 are roots of (1), then 71. |adj A| = |A|2 = 100 = |A|2 fi |A| = ± 10
l1 + l2 + l3 = 3. È1 ˘ Ê È 1 ˘ È0 ˘ˆ
Èa b c ˘ Èa b c ˘ Èa b b˘ 72. M Í0 ˙ = M Á Í-1˙ + Í1 ˙˜
Í ˙ Á Í ˙ Í ˙˜
68. A2 = ÍÍb c a ˙˙ ÍÍb c a ˙˙ = Í b a b˙ ÍÎ0 ˙˚ Ë ÍÎ 0 ˙˚ ÍÎ0 ˙˚¯
Í ˙
ÎÍ c a b ˙˚ ÎÍ c a b ˙˚ ÍÎ b b a ˙˚ È-1˘ È 1 ˘ È0 ˘
where a = a2 + b2 + c2, = Í 2 ˙ + Í 1 ˙ = Í3 ˙
Í ˙ Í ˙ Í ˙
b = bc + ca + ab. ÎÍ 3 ˚˙ ÍÎ-1˙˚ ÍÎ2 ˙˚
2
As A = I, we get È0 ˘ Ê È1˘ È1 ˘ È0 ˘ˆ
2 2 2 and M Í0 ˙ = M Á Í1˙ - Í0 ˙ - Í1 ˙˜
a +b +c =a=1 Í ˙ Á Í ˙ Í ˙ Í ˙˜
ÍÎ1 ˙˚ Ë ÍÎ1˙˚ ÍÎ0 ˙˚ ÍÎ0 ˙˚¯
bc + ca + ab = 0
È 0 ˘ È0 ˘ È-1˘ È 1 ˘
Now, (a + b + c)2 = a + 2b = 1
= Í 0 ˙ - Í3 ˙ - Í 2 ˙ = Í-5˙
fi a + b + c = 1 Í ˙ Í ˙ Í ˙ Í ˙
ÎÍ12 ˙˚ ÍÎ2 ˙˚ ÍÎ 3 ˙˚ ÍÎ 7 ˙˚
5.40 Complete Mathematics—JEE Main

È1 ˘ È 0 ˘ Ê 0 0 -1ˆ Ê 0 0 -1ˆ
M ÍÍ0˙˙ = ÍÍ3˙˙ fi m11 = 0 2. We have A2 = Á 0 -1 0 ˜ Á 0 -1 0 ˜ = I
Á ˜Á ˜
ÍÎ0˙˚ ÍÎ0˙˚ Ë -1 0 0 ¯ Ë -1 0 0 ¯

È0˘ È-1˘ 3. As B is inverse of A, AB = I


Similarly, M ÍÍ1˙˙ = ÍÍ 2 ˙˙ fi m22 = 2 Ê 1 -1 1 ˆ Ê 4 2 2 ˆ
ÍÎ0˙˚ ÍÎ 3 ˙˚ \ Á 2 1 -3˜ Á -5 0 a ˜ = 10 I
Á ˜Á ˜
Ë 1 1 1 ¯ Ë 1 -2 3 ¯
and m33 = 7
Ê10 0 5 - a ˆ Ê10 0 0ˆ
Thus, m11 + m22 + m33 = 9
fi Á 0 10 a - 5˜ = Á 0 10 0˜
73. (A–1 + B –1) [A – A (A + B)–1 A] Á ˜ Á ˜
Ë 0 0 5 + a ¯ Ë 0 0 10¯
= B –1 (B + A)A–1 A[I – (A + B)–1 A] fi a=5
–1 –1
= B (A + B) (A + B) [A + B – A] 4. A – A + I = O fi I = A – A2 = A(I – A)
2

= B –1 I B = I fi A–1 = I – A

74. A – B – A (A + B)–1 A + B (A + B)–1 B 5. For n = 1, all (a), (b), (c) and (d) hold good. We
have
= A[I – (A + B)–1 A] – [I – B (A + B)–1] B
È1 0 ˘ È1 0˘ È1 0˘
A2 = Í ˙Í =
= A(A + B)–1[A + B – A] Î1 1 ˚ Î1 1 ˙˚ ÍÎ 2 1 ˙˚
– [A + B – B] (A + B)–1 B If (a) holds for some n ≥ 1, then

= A (A + B)–1 B – A (A + B)–1 B An + 1 = AnA = [nA + (n – 1)I] A

=O = nA2 + (n – 1)A

75. A + B = AB È1 0 ˘ È1 0˘
= nÍ ˙ - (n - 1) Í1 1 ˙
Î2 1˚ Î ˚
fi I – (A + B) + AB = I
È 2n - 1 0 ˘
fi (I – A) (I – B) = I = Í ˙
fi (I – A)–1 = I – B Î 3n - 1 2n - 1˚
fi (I – B) (I – A) = I È1 0˘ È1 0˘
π (n + 1) Í ˙ + n Í0 1˙ = (n + 1) A + nI
fi I – (B + A) + BA = I Î1 1 ˚ Î ˚
fi A + B = BA Similarly (b) does not hold for n + 1.
Thus, AB = BA For (c),
76. Use An +1 = AnA = [nA – (n – 1)I ]A

O = A3 = (a + d)2 A = nA2 – (n – 1)A


Ê xy 0 - x2 ˆ È1 0 ˘ È1 0˘
= nÍ ˙ - (n - 1) Í1 1˙
77. A = Á 0
2
0 0 ˜ Î 2 1 ˚ Î ˚
Á ˜
ÁË y 2 0 - xy ˜¯ È 1 0˘
=
Í n + 1 1 ˙ = (n + 1) A - nI
A3 = O Î ˚
Thus, (c) holds.
Previous Years' AIEEE/JEE Main Questions
Alternatively
Èa b ˘ Èa b ˘ Èa + b 2ab ˘
2 2
1. A2 = Í =Í ˙ È0 0˘ 2
˙ Í ˙ Put B = Í ˙ and note that A = I + B and B =
Î b a ˚ Î b a ˚ Î 2ab a + b2 ˚
2
Î1 0˚
\ a = a2 + b2, b = 2ab O. As I and B commute.
Matrices 5.41
An = (I + B)n = I + nB [∵ B2 = O] a b˘
11. Let A = ÈÍ ˙
= n(B + I) – (n – I) I = nA – (n – I)I Îc d ˚
6. A2 – B2 = (A – B)(A + B) Now, A2 = I fi |A2| = |I|
2 2
= A – BA + AB – B fi |A|2 = 1 fi |A| = ± 1.
fi BA = AB
Suppose |A| = 1.
Ê a 0ˆ
7. Each matrix of the form Á commutes with In this case, A2 = I fi A = A–1
A. Ë 0 a ˜¯
Ê a b ˆ Ê d -b ˆ
8. |A| = 25a fi |A2| = |A|2 = 25 fi Á =
Ë c a ˜¯ ÁË -c a ˜¯
fi (25a)2 = 25 fi a = d, b = 0, c = 0
1
fi a = Tr(A) = 0 fi a + d = 0
5
Ê a bˆ fi 2a = 0 fi a = 0
9. Let A = Á
Ë c d ˜¯
Ê 0 0ˆ
In this case A = Á
Now, A2 = I fi det(A2) = 1 Ë 0 0˜¯
fi (det A)2 = 1 fi det A = ± 1. Thus, in this case, if we assume statement-2 is true
then we get statement-2 is false.
Also, A2 = I fi A = A–1
In case |A| = –1, then
Ê a bˆ Ê d -b ˆ A = – A–1
fi Á = det A Á
Ë c d ˜¯ Ë -c a ˜¯
If det A = 1, then Ê a b ˆ Ê d -b ˆ
fi Á = fi a + d = 0
Ë c d ˜¯ ÁË -c a ˜¯
a = d, b = – b, c = – c fi a = d, b = c = 0.
\ |A| = –1 fi Tr(A) = 0.
Ê a 0ˆ
In this case A = Á Therefore Statement-1 is true and Statement-2 is
Ë 0 a ˜¯ false.
2
|A| = 1 fi a = 1 fi a = ± 1.
12. The matrix
\ A = I or A = –I. A contradiction.
È1 a b˘
Thus, det (A) = –1. Íc 1 d˙
Í ˙
Ê a bˆ Ê d -b ˆ Ê - d bˆ ÍÎ e f 1 ˙˚
\Á = -1Á =
Ë c d ˜¯ Ë -c a ˜¯ ÁË c - a ˜¯
where exactly one of a, b, c, d, e, f is 1 and rest
of them are zeros, is invertible.
\ a = –d fi Tr(A) = a + d = 0
There are six such matrices.
\ Statement-1 is true and Statement-2 is false.

Ê a bˆ È1 0 1˘
10. Let A = Á
Ë c d ˜¯ Also, the matrix Í 0 1 0 ˙ is invertible.
Í ˙
Ê d -b ˆ ÎÍ 1 0 0 ˙˚
adj A = Á
Ë -c a ¯˜ Thus, there are at least 7 such matrices which are
|adj A| = ad – bc = |A| invertible.

\ Statement-2 is true. 13. Adding the first two equations and subtracting the
third from the sum, we obtain
Ê a bˆ
adj (adj A) A = Á =A (x1 + 2x2 + x3) + (2x1 + 3x2 + x3) – (3x1 + 5x2 +
Ë c d ˜¯ 2x3) = 3 + 3 – 1
Statement-1 is also true but Statement-2 is not a
correct reason for it. fi 0 = 5
5.42 Complete Mathematics—JEE Main
Thus, the system of equation has no solution. 19. We have
14. The system of equations will have a non-zero solu- Ê 1 ˆ Ê 0ˆ Ê 1 ˆ
tion if and only if A(u1 + u2 ) = Au1 + Au2 = Á 0˜ + Á 1˜ = Á 1˜
Á ˜ Á ˜ Á ˜
4 k 2 Ë 0¯ Ë 0¯ Ë 0¯
k 4 1 =0 We solve the above equation for u1 + u2.
2 2 1
We consider the augmented matrix
4 1 k 1 k 4 Ê 1 0 0 1ˆ
fi 4 -k +2 =0
2 1 2 1 2 2 ( A / B) = Á 2 1 0 1˜
Á ˜
Ë 3 2 1 0¯
fi 4(2) – k(k – 2) + 2(2k – 8) = 0
Applying R3 Æ R3 – 2R2 + R1 and R2 Æ R2 – 2R1,
fi k2 – 6k + 8 = 0 fi k = 2, 4 we get
15. H = wI fi H70 = w70I Ê 1 0 0 1ˆ Ê 1ˆ
( A / B) ∼ Á 0 1 0 -1˜ fi u1 + u2 = Á -1˜
But w70 = (w3)23w = w Á ˜ Á ˜
Ë 0 0 1 -1¯ Ë -1¯
\ H70 = wI = H
20. |P| = |A|2 = 16
16. The system of equations will have only trivial solu-
tion if and only if fi (1) (0) – a (4 – 6) + (3)(4 – 6) = 16
fi 2a – 6 = 16 fi a = 11
1 -k 1
k 3 -k π 0 21. As the system of equations has no solution
3 1 -1 k +1 8 4k
= π
3 -k k -k k 3 k k + 3 3k - 1
¤ (1) +k + (1) π0 k +1 8
1 -1 3 -1 3 1 But = fi k 2 + 4k + 3 = 8k
k k +3
fi 2(k2 + k – 6) 0 fi k –3, 2
fi (k – 1) (k – 3) = 0 fi k = 1, 3
17. We have A¢ = A and B¢ = B
8 4k
For k = 1, =
Now, (A(BA))¢ = (BA)¢ A¢ = (A¢B¢)A¢ k + 3 3k - 1
= (AB)A = A(BA) and for k = 3,
8 8 4 4k 3
= = and =
[∵ matrix multiplication is associative] k +3 6 3 3k - 1 2
fi A(BA) is symmetric. Thus, there is exactly one such value of k.

Similarly, (AB) A is a symmetric matrix 22. Adding first two equations and subtracting from
Also, (AB)¢ = B¢A¢ = BA third, we get

Note that AB is symmetric if and only if AB = BA. (a – 8)x3 = b – 15


Thus, both Statement-1 and Statement-2 are true If a 8, we get value of x3, and substituting in
but Statement-2 is not a correct explanation for the first two we get values of x1 and x2. When a 8,
statement-1. we get the system has a unique solution.
18. P3 = Q3 and P2Q = Q2P gives Thus, a = 8 and hence b = 15.
P 3 – P 2Q = Q 3 – Q 2P
When a = 8 and b = 15, we solve first two equa-
fi P2(P – Q) = – Q2(P – Q) tions in terms of x3 to obtain infinite number of
fi (P2 + Q2) (P – Q) = 0 solutions.

If det (P2 + Q2) 0, then P2 + Q2 is invertible and 23. 3p + 3q + 2r = 3 (i)


hence P = Q. Therefore, det (P2 + Q2) = 0. 4p + 2q = 0 (ii)
Matrices 5.43
p + 3q + 2r = 1 (iii) a bˆ
27. Let A = ÊÁ
From (i) and (iii) 2p = 2 or p = 1 Ë c d ˜¯
A2 = I fi det (A)2 = 1
When p = 1, q = –2 [from (ii)]
fi det (A) = ± 1.
Now, 2p + q – r
1 Now, A2 = I fi A–1 = A
= 2 p + q - [1 - p - 3q ]
2 Ê d - bˆ Ê a b ˆ
fi det( A) Á = (1)
1
= 2(1) + ( -2) - [1 - 1 - 6] = -3 Ë -c a ˜¯ ÁË c d ˜¯
2 If det (A) = 1, then
24. As the system of equation has a unique solution.
a = d, –b = b, –c = c
1 a 0 fi a = d, b = 0, c = 0.
D= 0 1 a π0
\ det (A) = 1 fi a2 = 1 fi a = ± 1.
a 0 1
Using C1 Æ C1 + C2 + C3, we get Thus, in this case A = I or A = –I.

1 a 0 But A π I, –I.
D = (1 + a ) 1 1 a Therefore det(A) = –1.
1 0 1
fi Statement-2 is true.
Using R2 Æ R2 – R1, R3 Æ R3 – R1, we get
Now, from (1)
1 a 0
Ê d -b ˆ Ê a b ˆ
D = (1 + a ) 0 1 - a a ( -1) Á =
Ë -c a ˜¯ ÁË c d ˜¯
0 -a 1
fi a + d = 0 fi tr(A) = 0
= (1 + a) (1 – a + a2) = 1 + a3
Thus, Statement-1 is true and Statement-2 is correct
D 0 fi a –1 [∵ a Œ R] reason for it.
a bˆ BB¢ = (A–1A¢ )(A–1A¢ )¢
25. Let A = ÊÁ ; a, b, c Œ {0, 1, 2}. 28.
Ë c a ˜¯ = (A–1A¢ )[(A¢ )¢(A–1)¢]
2
|A| = a – bc = A–1(A¢A)(A–1)¢
When a = 0, bc 0 fi b, c Œ {1, 2} = A–1(AA¢ ) (A¢ )–1
= (A–1A)[A¢(A¢)–1]
There are four such values of b, c, viz. (b, c) =
(1, 1), (1, 2), (2, 1) and (2, 2) = (I)(I) = I
2
When a = 1, both b and c cannot be 1. In this case 29. As B = 0, Br = 0 " r ≥ 2.
there are 8 such pairs of (b, c). Now, (I + B)50 = I + 50
C 1B + 50
C 2B 2 + " +
50
When a = 2, both b and c cannot be 2. In this case C50 B50
also, there are 8 such pairs of (b, c).
= I + 50B [∵ Br = 0 " r ≥ 2]
È1 2˘ È 1 2˘ È 9 - 4˘
26. A2 = Í fi (I + B)50 – 50B = I
˙Í ˙=Í ˙
Î 4 - 3˚ Î 4 - 3˚ Î - 8 17 ˚
fi det(I + B)50 – 50B = 1
A2 + 4A – 5I
30. Applying C2 ´ C3, we get
È 9 - 4˘ È 4 8˘ È - 5 0 ˘
=Í ˙ +Í ˙+Í ˙
Î - 8 17 ˚ Î16 - 12 ˚ Î 0 -5˚ È1 3 2˘ È0 1 0˘
A Í 0 3 2˙ = Í1 0 0˙
È8 4 ˘ È2 1˘ Í ˙ Í ˙
=Í ˙ = 4 Í 2 0˙ ÍÎ 0 1 1 ˙˚ ÍÎ 0 0 1 ˙˚
Î8 0 ˚ Î ˚
5.44 Complete Mathematics—JEE Main
Applying C1 ´ C2, we get fi A3 = –A and A4 = I

È3 1 2 ˘ È1 0 0˘ Now, A4 – I = O = A2 + I
A Í3 0 2 ˙ = Í 0 1 0˙ and A3 – I = –A – I = –A + A2 = A(A – I)
Í ˙ Í ˙
ÍÎ1 0 1 ˙˚ ÍÎ 0 0 1 ˙˚
and A3 + I = –A + I = A4 – A = A(A3 – I)
È3 1 2˘
fi A = Í3 0
-1
2˙ But A2 + I = O and A(A2 – I) = – 2A.
Í ˙
ÎÍ1 0 1 ˙˚ \ A2 + I A(A2 – I).
È6 ˘ 36. We know A(Adj A) = |A|I2
31. AB = Í ˙
Î8 ˚ \ AA¢ = |A|I2
fi y + 2x + x = 6 È5a -b ˘ È 5a 3˘ È1 0 ˘
fi Í ˙ Í-b 2˙ = (10a + 3b) Í0 1˙
3y – x + 2 = 8 Î 3 2 ˚Î ˚ Î ˚
fi 3x + y = 6 È ˘ È1 0 ˘
fi Í25a + b 15a - 2b ˙ = (10a + 3b) Í
2 2
˙
–x + 3y = 6 Î 15a - 2b 13 ˚ Î0 1 ˚

fi 3x + y = –x + 3y fi 25a2 + b2 = 10a + 3b, 15a – 2b = 0, 10a + 3b

fi y = 2x = 13
45
Now, b = (15/2)a 100 + a = 13
32. Let C = AB – BA, then 2
C¢ = (AB – BA)¢ = (AB)¢ – (BA)¢ fi a = 2/5, b = 3
\ 5a + b = 2 + 3 = 5
= B¢A¢ – A¢B¢
Ê cos a sin a ˆ
37. P = Á where a = p/6
= (–B)A – A(–B) Ë - sin a cos a ˜¯
= –BA + AB = C Ê cos a - sin a ˆ
P¢ = Á
fi AB – BA is symmetric. Ë sin a cos a ˜¯
33. AAT = 9I Ê 1 0ˆ
We have PP¢ = Á
È 1 2 2 ˘ È1 2 a ˘ È1 0 0˘ Ë 0 1˜¯
fi P¢ = P –1
fi Í 2 1 -2 ˙ Í 2 1 2 ˙ = 9 Í 0 1 0 ˙
Í ˙Í ˙ Í ˙ Thus, Q = PAP–1 fi P–1QP = A
ÍÎ a 2 b ˙˚ ÍÎ 2 -2 b ˙˚ ÍÎ 0 0 1 ˙˚
fi A2 = (P–1QP)2 = P–1Q2P
È 9 0 a + 4 + 2b ˘ È 9 0 0 ˘
Í ˙ fi A3 = P–1Q3P
fi Í 0 9 2a + 2 - 2b ˙ = Í 0 9 0 ˙
Í ˙
Í a + 4 + 2b 2a + 2 - 2b a + 4 + b ˙ ÍÎ 0 0 9 ˙˚
2 2 Continuing like this we get,
Î ˚
fi a + 2b = –4, a – b = –1, a2 + b2 = 5 A2015 = P–1Q2015P = P¢Q2015P
Ê 0 1ˆ
fi (a, b) = (–2, –1) We have A = I + B where B = Á
Ë 0 0˜¯
34. |5 adj A| = 5
Ê 0 1ˆ Ê 0 1ˆ Ê 0 0ˆ
fi 53 |adj A| = 5 Also, B2 = Á
Ë 0 0˜¯ ÁË 0 0˜¯
= Á
Ë 0 0˜¯
fi |adj A| = 1/25
Ê 0 0ˆ
fi Br = Á "r≥2
fi |A|2 = 1/25 fi |A| = ± 1/5 Ë 0 0˜¯
È 0 -1˘ È 0 -1˘ Ê 1 2015ˆ
35. A = Í ˙ = -I Thus, A2015 = I + 2015B = Á
2
˙Í
Î 1 0˚ Î 1 0˚ Ë0 1 ¯˜
Matrices 5.45
38. Let B = A2016 – 2A2015 – A2014 Note that B2 = O.
= A2014 (A2 – 2A – I) A2 = (I + B)2 = I2 + 2IB + B2
= A2014 (A – ( 2 + 1)I)(A + ( 2 – 1)I)
= I + 2B
-4 -1 A3 = AA2 = (I + B)(I + 2B)
We have |A| = = –1
3 1 = I2 + BI + 2(IB) + 2B2
-4 - ( 2 + 1) -1 = I + 3B [∵ B2 = O]
|A – ( 2 + 1)I| = 2 3
3 1 - ( 2 + 1) Thus, 2A – A = 2(I + 2B) – (I + 3B)
= I + B = A.
= (–4 – ( 2 + 1)(1 – ( 2 + 1) + 3
3. Statement-2 is true. [See Theory]
= ( 2 + 1)2 + 3( 2 + 1) – 1 = 5 + 5 2
tr(AB – BA) = tr(AB) – tr(BA) = 0
-4 + ( 2 - 1) -1 But tr(I) = 2.
|A + ( 2 – 1)I| =
3 1 + ( 2 - 1)
\ AB – BA I.
= (–4 + ( 2 – 1))(1 + ( 2 – 1)) + 3 Thus, both statements are true and Statement-2 is
2
= ( 2 – 1) – 3( 2 – 1) – 1 a correct explanation for Statement-1.

=5–5 2 4. As A – aI is invertible for all a Œ R.

Thus det (B) = (–1) 2014


(5 + 5 2 )(5 Р5 2 ) det (A РaI) 0 " a ΠR.
= –25
fi (a Рa)(d Рa) Рbc 0 " a ΠR.
39. A2 – 5A = 7I = O
2
fi 7I = A(5I РA) = (5I РA)A fi a Р(a + d)a + ad Рbc 0 " a ΠR.

I = AB = BA Therefore
1
where B = (5I – A) (a + d)2 – 4(ad – bc) < 0
7
fi (a – d)2 + 4bc < 0
1
\ A–1 = B = (5I –A) Therefore, bc < 0.
7
fi Statement-1 is true. Also, a2 + d2 – 2ad + 4bc < 0
We have fi 0 £ a2 + d2 < 2ad – 4bc
A – 2A – 3A + I
3 2

fi 1
= A(5A – 7I) – 2(5A – 7I) – 3A + I bc < ad .
2
= 5A2 – 7A – 10A + 14I – 3A + I
Thus, bc < min ÊÁ 0, ad ˆ˜
1
= 5(5A – 7I) – 20A + 15I Ë 2 ¯
= 25I – 35I – 20A + 15I 5. Statement-2 is false as
= 5(A – 4I)
adj(AB) = (adj B) (adj A)
Thus, Statement-2 is also true.
(adj A) (adj B)
Previous Years' B-Architecture Entrance
But Statement-1 is true as
Examination Questions
A(adj A) = |A|I2. [See Theory]
1. adj(AB) = (adj B) (adj A) [See Theory]
6. det (A) = ad – bc
2. A = I + B
Note that det (A) can take value –1, 0 or 1. We
È0 1 0˘
where B = Í 0 0 0 ˙ have
Í ˙
det(A) = 1 ¤ ad = 1, bc = 0
ÎÍ 0 0 0 ˙˚
5.46 Complete Mathematics—JEE Main
¤ a = 1, d = 1 or (b = 0, c = 0, b = 0; c = 1; -1 Ê 1 -k ˆ
Also, Sk = ÁË
b = 1, c = 0) 0 1 ˜¯
and det(A) = –1 ¤ ad = 0 or bc = 1 Ê 1 2 nˆ Ê 1 - k ˆ
\ ( S2 ) n ( Sk ) -1 = Á
Ë 0 1 ˜¯ ÁË 0 1 ˜¯
This is also possible in 3 cases.
\ A–1 exists in 6 cases. Ê 1 2n - k ˆ
= Á = S2n - k
Ë0 1 ˜¯
a b ˘ Èa b ˘
7. A2 = ÈÍ ˙Í ˙ È0 1˘
Îc d ˚ Îc d ˚ 9. Let A = I + C where C = Í ˙
Î0 0˚
È a 2 + bc b(a + d ) ˘ AS C2 = O, we get Cr = 0 " r > 2.
=Í ˙
ÍÎ c(a + d ) bc + d 2 ˙˚ Now,

È a + bc
2
2b ˘ A10 = (I + C)10
=Í ˙ 10
ÍÎ 2c bc + d 2 ˙˚ = I + 10C + C2 (C2) + … + 10
C10(C10)

È1 10˘
Èa 2 - 2a + bc 0 ˘ = I + 10C + O = Í ˙
\ A2 - 2 A = Í ˙ Î0 1 ˚
ÍÎ 0 a 2 - 2d + bc ˙˚
È 1 -10 ˘
fi Adj( A10 ) = Í ˙
Since A¢ = A2 – 2A, we get Î0 1 ˚

È a c ˘ È a - 2a + bc ˘ b b2 ˘
2
As È 1
0
Íb d ˙ = Í ˙ Íb = 10 A10 + adj( A10 ) , we get
Î ˚ ÍÎ 0 a - 2d + bc ˙˚
2
Î 3 b4 ˙˚
È b1 b2 ˘ È10 100 ˘ È 1 -10 ˘
fi b = 0, c = 0, a = a2 – 2a, d = d2 – 2d fi Í = +
Îb3 b4 ˙˚ ÍÎ 0 10 ˙˚ ÍÎ 0 1 ˙˚
fi a(a – 3) = 0 and d(d – 3) = 0
È11 90 ˘
= Í ˙
fi a = 0 or a = 3 and d = 0 or d = 3 Î 0 11 ˚
fi b1 = b4 = 11, b2 = 90, b3 = 0,
But then a + d cannot be 2.
\ b1 + b2 + b3 + b4 = 112
\ S=f
10. |adj A| = |A|2 = 36
Ê 1 2ˆ
8. S2 = Á = I + B where 1 -2 4
Ë 0 1˜¯
fi 4 1 1 = 36
Ê 1 0ˆ Ê 0 2ˆ
I =Á and B = Á
Ë 0 1˜¯ Ë 0 0˜¯ -1 k 0
2
Note that B = O, therefore, Br = O " r ≥ 2. -2 4 1 4
n n
fi (–1) –k = 36
Thus, (S2) = (I + B) = I + nB 1 1 4 1
[Expanding along R3]
È 1 2n ˘
=Í ˙
Î0 1 ˚ fi (–1)(–2 – 4) – k(1 – 16) = 36
fi 6 + 15k = 36 fi k = 2

You might also like